Systems-Based Emergency Medicine Course Objectives: Cardiovascular Emergencies PA 605 ER

Réussis tes devoirs et examens dès maintenant avec Quizwiz!

For each of the following cardiovascular conditions, students should describe the underlying pathophysiology, presenting signs and symptoms, basic epidemiology, modifiable (when applicable) and non-modifiable risk factors, differential diagnosis, appropriate diagnostic studies, clinical intervention, treatment guidelines, pharmaceutical therapies, and health maintenance concerns: Hypotension

shock in PPP (look up)

Tx: sodium nitroprusside = drug of choice

-Potential for developing cyanide toxicity -> treat with sodium thiosulfate which combines with cyanide ion to form thiocyanate (nontoxic) -Nitro should be avoided in setting of renal failure due to risk of cyanate and thiocyanate toxicity

Question: What is the pathophysiology of Osler nodes?

Answer: Sequelae of vascular occlusion from microthrombi.

Aortic regurgitation (ROSH)

Aortic regurgitation occurs when valve leaflets fail to close fully, causing blood to flow from the aorta back into the left ventricle during diastole. Increasing stroke volume followed by a rapid pressure drop during diastole results in a wide pulse pressure. Patients with acute aortic regurgitation present most commonly with dyspnea and pulmonary edema. If associated with a sudden onset of chest pain, especially if tearing in quality, one must evaluate for aortic dissection as the acute cause. Patients with chronic regurgitation typically present with exertional dyspnea, fatigue, or worsening heart failure. Chest pain may occur from ischemia related to low diastolic pressures. If febrile, or there is a history of IV drug use, endocarditis should be suspected. On physical exam, there will be a classic, blowing diastolic murmur that is best heard in the second or third intercostal space at the left sternal border. The diagnosis is confirmed with echocardiography. If aortic dissection is suspected, a CT angiogram should be performed. Treatment of acute aortic regurgitation requires surgical intervention. Afterload reduction with vasodilators such as nitroprusside in combination with inotropic agents such as dobutamine may be required to stabilize and bridge the patient on the way to the operating room.

CAUSES OF SYNCOPE: "HEAD HEART AND VESSELS"

Cerebral malfunctioning can occur with loss of metabolic substrates seen in hypoxemia or hypoglycemia. Patients may be hypoxic due to toxins such as carbon monoxide, ventilation/perfusion mismatch or hypercapnic respiratory failure (which might be worsened by supplemental oxygen often given to people who have TLOC). Pulse oximetry should be checked on all patients suffering loss of consciousness with the realization that pulse oximetry is not accurate in carbon monoxide poisoning as carboxyhemoglobin is bright red, like arterial blood. Possible sources of carbon monoxide need investigation. Multiple victims, exposure history and headache, confusion with nausea and vomiting may precede loss of consciousness and all suggest possible carbon monoxide poisoning. Patients with severe lung disease develop resistance to hypercarbia and high-flow oxygen can decrease respiratory drive creating loss of consciousness. A venous blood gas might indicate respiratory acidosis. Pulmonary hypertension presents as dyspnea on exertion, hypoxia, and signs of right ventricular failure (pulmonary artery dilation, echocardiographic changes, elevated B-type natriuretic peptide (BNP) without pulmonary edema). Persistent altered mental status requires evaluation for hypoglycemia with a fingerstick glucose. Symptoms are usually adrenergic with sweating, hunger, and tachycardia. Continued coma is rare and should resolve with glucose correction. Seizures may be confused with syncope given that some patients exhibit asynchronous brief myoclonic jerks with their loss of consciousness. Epilepsy, rather than syncope, is suggested by tonic-clonic activity, prolonged episode more than 15 seconds, automatisms, lateral tongue biting, and a postictal state. Bladder incontinence and trauma are seen in both syncope and seizures. Nonconvulsive status epilepticus usually presents as persistent coma rather than syncope. Anxiety may cause syncope or pseudoseizures. Hyperventilation (perceived or not by the patient) due to anxiety can cause syncope. Many lethal diseases are associated with anxiety and it is a diagnosis of exclusion. Eyes are generally open with seizures and syncope, and resist opening in anxiety. Bizarre behaviorisms also may occur. A dysfunctional diencephalon (vertebrobasilar TIA) presents as vertigo, dizziness, and/or ataxia, often with crossed cranial neuropathies. This is a rare cause of TLOC. Drop attacks (cataplexy, narcolepsy) are characterized by sudden loss of muscle tone but no change in sensorium. There are no reports of syncope as a manifestation of carotid artery origin stroke. Heart attack (acute coronary syndrome) is a potentially lethal cause of syncope. As a sole symptom of ACS, syncope is unusual, but initial symptoms include syncope in 5-15% of patients particularly in the elderly. High-risk conditions include preload-dependent right ventricular infarction (usually inferior myocardial infarction [MI] associated with right coronary artery occlusion), sick sinus syndrome, atrioventricular (AV) nodal block (perhaps due to adenosine hypersensitivity) associated with inferior MI, and finally pump failure due to dysrhythmias or muscle loss which is more often seen in anterior ischemia. Diaphoresis, dyspnea, angina, and hypotension warrant an ECG and cardiac marker evaluation. Cardiac dysfunction is a potentially lethal cause of syncope and should remain a prime focus of investigation. Syncope is the initial presentation of PE in 10% of patients and is associated with poor outcomes. Dyspnea, chest pain, and risk profile analysis (eg, the pulmonary embolism rule-out criteria [PERC] rule, Wells rule) can help in diagnosis. Evaluation for PE is discussed in Chapter 33. Chronic pulmonary emboli as well as chronic obstructive pulmonary disease (COPD)/hypoxia can cause pulmonary hypertension. Syncope in pulmonary hypertension is ominous and requires admission and specialized care. The ischemic right ventricle handles volume overload poorly, so treatment requires low-volume resuscitation with pulmonary dilators and vasopressors. Intubation with increased intrathoracic pressure and decreased venous return is a well-known cause of fatal hemodynamic collapse and noninvasive ventilation with continuous positive pressure is preferred. Aortic obstruction from hypertrophic obstructive cardiomyopathy (HOCM) is characterized by asymmetric left ventricular (LV) hypertrophy and disproportionate septal thickening, which causes syncope either by dynamic outflow obstruction or dysrhythmias. HOCM is a genetically transmitted disease of myocardial sarcomeres and can present as syncope and sudden death. One in 500 adults have the disease in various degrees of penetration and a family history exploration is important. About 20% of HCM patients present with syncope. A systolic murmur that increases with standing, Valsalva, or nitroglycerin is pathognomonic. ECG is abnormal in 90% of patients, with septal Q waves in inferior and lateral precordial leads and signs of left ventricular hypertrophy (LVH). Echocardiography should be done with cardiology consultation. Classic aortic stenosis (AS) presents as exertional angina, congestive heart failure (CHF), and syncope. Listen for a systolic harsh murmur at the aortic window. A narrow pulse pressure may also be present. ECG changes include LVH, and echocardiography is diagnostic. Patients with AS and syncope need cardiology admission. Multiple dysrhythmias, both slow and rapid, produce syncope. Sudden onset, family history of sudden death, exertional precipitant, palpitations during syncope, and history of channelopathies are high-risk indicators of an arrhythmic cause. Most patients should be placed on a cardiac monitor at intake, which may "catch" the cause. An ECG is indicated in most of the patients with syncope. A history of palpitations and/or structural heart disease should be ascertained. Ventricular tachycardia (VT) usually occurs in individuals with structural heart disease, while catecholaminergic polymorphic ventricular tachycardia (CPVT) occurs in young adults and children during exertion or emotional stress without structural cardiac abnormalities. CPVT is a genetic disorder of calcium handling with an association with an early depolarization pattern in the inferior/lateral leads on ECG. Many tachyarrhythmias will self-terminate but some degenerate into ventricular fibrillation. Brugada syndrome is a type of channelopathy, due to an autosomal dominant gene mutation for the sodium channel, which predisposes to ventricular tachycardia and sudden death. The ECG has an rSR pattern with ST elevation in lead V1 (a pseudo right bundle branch block [RBBB] that does not have the S wave seen in I and lateral precordial leads) (Figure 18-2). Long QT syndrome may be congenital (with or without deafness) or acquired. Many drugs, as well as electrolyte disturbances (hypokalemia, hypomagnesemia), cause long QT (Figure 18-3). Persistent symptomatic sinus bradycardia, Mobitz II second-degree or third-degree block, alternating left and right BBB, torsades de pointes, and ventricular tachycardia require cardiology evaluation. Arrhythmogenic right ventricular dysplasia (ARVD) is a genetic disorder in which there is a fibrofatty infiltration of myocardial tissue targeting the right ventricle causing aneurysmal dilation and electrical instability. ECG may show epsilon waves and greater than 80% have persistent T-wave inversion V1-V3 not due to RBBB (Figure 18-4). Echocardiography is diagnostic. Vasovagal faint (the common faint) is a condition precipitated by emotional distress or orthostatic stress from prolonged standing. Adrenergic symptoms of sweating, nausea, vomiting, and pallor associated with prodromal symptoms are typical. Treatment is supine positioning. Vasovagal faint should be rarely considered as a cause of syncope in the elderly. Ectopic pregnancy is reminder to search for volume or blood loss. Intravascular volume loss, either blood or fluids, can precipitate syncope. Hematocrit and a search for bleeding or volume loss (overt or occult) is an important investigation. Third spacing, such as with ascites, should also be remembered. Situational syncope is characterized by loss of consciousness after micturition, defecation, cough, brass instrument playing, or Valsalva maneuver with reduction of cardiac preload. Labeling syncope as situational should not dissuade one from pursuing other causes. Gastrointestinal bleeding may present as "defecation syncope." Subclavian steal, a rare cause of syncope, is suspected by arm use dizziness, vertebral artery flow reversal, and a blood pressure difference in the arms of greater than 40 mm Hg. ENT causes include trigeminal neuralgia and more often glossopharyngeal neuralgia, which are rarely associated with transient symptomatic bradycardia, occasionally requiring a pacer. Low systemic vascular resistance (SVR) remains a frequent final common pathway to cause syncope. Medications that reduce systemic vascular resistance such as nitrates, antihypertensives, or sildenafil can be implicated, as well as diabetic neuropathy with reduction of SVR on standing due to loss of small vessel innervation. Addison disease causes hypotension, hypoglycemia, and syncope. Orthostatic vital signs are recommended by some guidelines but are controversial. The best marker is recurrence of near syncope upon standing rather than a specific number change. Carotid sinus hypersensitivity (CSH) is classically associated with neck movement or constriction (shaving for example), and is an exaggeration of carotid sinus baroreceptor stimulation. Unfortunately, criteria for diagnosis and use of carotid sinus massage with or without tilt table testing is not standardized. Carotid sinus massage is not risk free, generally not part of the initial investigations and the technique is described elsewhere.

ACLS

If a patient does not respond to initial CPR and defibrillation, further interventions are required. Furthermore, in cases where initial interventions are successful in restoring a perfusing rhythm, the patient may still require ALS (ie, airway, medications, and further evaluation) to optimize outcome. Research devoted to ALS is ongoing; however, this chapter reflects the AHA's most current recommendations. ADVANCED AIRWAY MANAGEMENT During a resuscitation, the risks and benefits of advanced airway management must be considered. To place a definitive airway requires time that may result in interruption of effective chest compressions. In addition, there is a risk of esophageal intubation that is highest in inexperienced providers. Once a definitive airway is in place, the risk of aspiration is significantly reduced and more effective ventilation can be performed. Current advanced cardiac life support (ACLS) recommendations emphasize the use of effective bag-valve-mask ventilations rather than definitive airway placement during a resuscitation to minimize prolonged interruptions to effective chest compressions. Practice in most emergency departments still involves early placement of an advanced airway, as the skill level and training of most emergency medicine physicians will typically allow rapid, accurate insertion. Advanced airway techniques include endotracheal intubation, the use of the laryngeal mask airway, or the use of a double-lumen tube such as the esophageal-tracheal combitube or pharyngotracheal lumen airway. Endotracheal intubation provides the best means of securing the airway; however, intubation should be attempted only by providers who are skilled in the procedure and who perform it at least 6-12 times per year. Placement of the tube should be confirmed by colorimetric ETCO2 detectors (although unreliable in prolonged cardiac arrest), esophageal detector devices, or capnography in addition to more traditional techniques. The details of endotracheal intubation are discussed in Chapters 7 and 10. When unable to secure an airway or to ventilate the patient using a bag-mask, the trained provider must perform a surgical or needle cricothyroidotomy. VASCULAR ACCESS While peripheral IVs are still commonly used for prehospital and in-hospital vascular access to deliver medications in resuscitations, intraosseous devices are becoming more prevalent in both EMS and emergency room practice that can provide more rapid and equally effective vascular access in the setting of arrest for drug and fluid delivery. Devices are available that use either powered or manual insertion techniques and can be inserted in a variety of sites in the adult patient. Central venous catheters may also be inserted in the hospital for more definitive access and central administration of medications. SPECIFIC RHYTHMS Ventricular Fibrillation or Pulseless Ventricular Tachycardia As mentioned previously, rapid defibrillation is essential in preventing progression of VF or pulseless VT to asystole. Recently, emphasis has been given to minimizing the interruption of CPR. In an unwitnessed arrest, it is now recommended that CPR be performed until the defibrillator is applied to the patient and charged. It is felt that this will increase myocardial oxygen and energy substrates, increasing the likelihood of a restoration of spontaneous circulation (ROSC) to the myocardium following defibrillation. If the arrest was witnessed and a defibrillator is present, deliver the first shock immediately. The first shock is delivered at 360 or 200 J with a monophasic defibrillator or biphasic defibrillator, respectively. CPR is immediately resumed after the first shock. Vasopressors (ie, epinephrine) may be given before or after the first shock, being careful not to interrupt CPR or defibrillation. Epinephrine is given at 0.5-1.0 mg IV every 3-5 minutes. Following the 2 minutes of CPR, a second shock should be performed if indicated. The pulse should only be checked if there is an organized rhythm on the monitor. Again, 360 or 200 J with a monophasic defibrillator or biphasic defibrillator, respectively, is delivered. If VF or pulseless VT persists after two to three shocks, an antiarrhythmic (eg, amiodarone) should be administered. Magnesium should be considered for torsades de pointes. The cycle of CPR, defibrillation, and vasopressors is continued until there is ROSC, a change in rhythm, or the resuscitation efforts are deemed futile. Pulseless Electrical Activity PEA is the condition where there is an organized rhythm on the monitor in the absence of cardiac output. The key to successful resuscitation is identifying and treating the underlying cause. Determining the offending etiology can be facilitated by history, vital signs including temperature, electrocardiogram (ECG), blood gas analysis, electrolytes, and a focused physical examination. Initial interventions can be performed concurrently to address hypoxemia and hypovolemia. In addition to effective CPR, epinephrine (1 mg IV every 3-5 minutes) is given. Asystole Asystole is usually a preterminal rhythm. As with PEA, the underlying causes must be addressed. Transcutaneous pacing may be considered early on, but it should be used only if the patient was witnessed entering asystole from a perfusing rhythm. Epinephrine (1 mg IV every 3-5 minutes) is administered. If an asystolic patient does not respond to ALS interventions in the field, resuscitation efforts may be terminated in the field without urgent transport. Tachydysrhythmia with a Pulse If a patient has a tachydysrhythmia with evidence of poor perfusion (in which the tachydysrhythmia is the likely source), prompt cardioversion is indicated. Chest pain, dyspnea, altered level of consciousness, hypotension, and new-onset CHF are all indications of instability. Quickly perform synchronized cardioversion. Give the patient analgesic and sedative agents if circumstances permit. In a hemodynamically stable patient with a tachydysrhythmia, clinical evaluation is needed to determine the source of the dysrhythmia. History and ECG are the two key diagnostic aids. The tachycardias are classified as narrow or wide complex. The most difficult differentiation is with the regular wide-complex tachycardias. The diagnostic dilemma is to decide whether the rhythm is supraventricular in origin with aberrant conduction or a true VT (Chapter 34). If the rhythm cannot be confirmed as supraventricular tachycardia (SVT), the patient should be presumed to have a VT, and appropriate treatment should be rendered. Treating a VT as an SVT can have disastrous results. Cardioversion As mentioned above, the treatment for unstable tachycardia, which is not VF or pulseless VT, is synchronized cardioversion. Synchronization prevents the shock being delivered during the refractory period of the cardiac cycle, which may trigger VF. Escalate the dose of the second and subsequent shocks if ineffective. For unstable atrial flutter and other SVTs, the initial charge is 50-100 J. Escalate the dose of energy if ineffective. For unstable atrial fibrillation, the biphasic energy delivered is 120-200 J. For unstable VT with a pulse, the morphology and rate determine the amount of energy which should be delivered in the first synchronized shock. If the VT is monomorphic, an initial charge of 100 J is delivered, and increased to 200, 300, and 360 J if ineffective. For polymorphic VT, treat in the same manner as VF. Bradycardia As with tachydysrhythmia, a slow rhythm with signs of inadequate perfusion needs immediate intervention, regardless of the underlying pathology. A relative bradycardia with good perfusion may be observed while preparations are made to treat the underlying cause. Atropine (0.5 mg IV every 3-5 minutes with up to 3 mg total) can be beneficial if parasympathetic tone is excessive. This is typically the case in depressed sinus node automaticity and in atrioventricular (AV) block secondary to acute myocardial infarction. Bradycardias secondary to degeneration of the conduction system (or disruption, as in a transplanted heart), or due to pharmacologic or metabolic causes, generally do not respond to atropine. Transcutaneous or transvenous pacing can be used as a bridge to a more permanent pacing mechanism. In addition, epinephrine or dopamine infusions may be required if the patient fails treatment with atropine or pacing. PHARMACOLOGIC THERAPY See Table 9-1. Vasoactive Agents A. Epinephrine Epinephrine is a mixed α- and β-agonist that has been shown to increase diastolic aortic blood pressure, coronary perfusion pressure, as well as cerebral blood flow. As such, it has been the mainstay of pharmacologic therapy in the pulseless patient, regardless of the underlying rhythm. The dose and frequency are described above. Epinephrine may be given through an endotracheal tube if IV access cannot be obtained. The endotracheal dose is 2.0-5.0 mg in 10 mL of normal saline. High-dose epinephrine in adults is no longer recommended. B. Vasopressin Vasopressin (antidiuretic hormone) is an endogenous hormone that causes peripheral vasoconstriction, as well as vasoconstriction of the coronary, cerebral, and renal vasculature. Recent studies have indicated that vasopressin has no benefit over epinephrine in cardiac arrest and should not be routinely used. C. Dopamine Dopamine is endogenous neurotransmitter that exhibits both α- and β-adrenergic effects, as well as dopaminergic effects that manifest in a dose-dependent fashion. Doses of greater than 20 μg/kg/min may have adverse effects on splanchnic perfusion and therefore should be avoided. Dopamine can also provoke tachydysrhythmias. If hypotension persists after optimization of filling pressures, dobutamine (as an inotropic agent) or norepinephrine (as a vasopressor) should be considered. D. Norepinephrine Norepinephrine is an adrenergic agent that affects α-receptors in the vasculature and β1-receptors of the heart resulting in peripheral vasoconstriction and increased heart rate and contractility. It is used in states of severe shock and is recommended for management of hypotension when systolic pressures are less than 70 mm Hg. The starting infusion rate is 0.5-1.0 μg/min and titrated to effect, with a maximal infusion rate of 30 μg/min. E. Dobutamine Dobutamine is a synthetic catecholamine with potent β1-agonist properties with little β2- or α-adrenergic effects. As such, it can improve myocardial contractility and increase cardiac output. It is the initial agent of choice in patients with systolic blood pressure of 70-100 mm Hg. Paradoxically it can worsen hypotension in patients with inadequate preload. In addition, dobutamine can provoke tachyarrhythmias. It is run as an IV infusion of 2-20 mg/kg/min. Antidysrhythmics A. Adenosine Adenosine is an endogenous nucleoside that is a potent but short-lived AV nodal blocking agent. Along with vagal maneuvers, it is considered first-line therapy in paroxysmal supraventricular tachycardia (PSVT) secondary to a reentrant-type conduction defect. Adenosine should be considered only when a supraventricular rhythm is suspected. It should not be used as an aid in differentiating between PSVT with aberrant conduction and VT. It is associated with a prolonged sinus pause. The initial dose is 6 mg, given as a rapid bolus. If this fails to resolve PSVT, the dose is increased to 12 mg IV. If there is no response, this dose may be repeated in 1-2 minutes. B. Amiodarone Amiodarone is predominantly a class III antidysrhythmic (potassium channel blocker), but it also has some properties of class I (sodium channel blockade), class II (β-blockade), and class IV (calcium channel blockade) antidysrhythmics. This wide variety of effects makes amiodarone useful in treating both supraventricular and ventricular tachydysrhythmias. For VF and pulseless VT, the initial dose is 300 mg IV, which can be followed with a second dose of 150 mg if the arrhythmia persists. While amiodarone has been found to increase the rate of ROSC and prehospital survival from cardiac arrest, it has not been shown to increase survival to hospital discharge. C. Atropine Atropine is an anticholinergic agent useful in the treatment of symptomatic bradycardias that are due to increased parasympathetic tone. It should not be used when infranodal pathology is suspected such as with second-degree Mobitz type II AV blocks. Atropine is no longer indicated in the setting of asystole and bradycardic PEA. It is ineffective in the setting of previous heart transplant and may worsen ischemia during a myocardial infarction. Dosing is described in Table 9-1. D. β -Adrenergic Blockers β-Blockers (ie, atenolol, metoprolol, esmolol) are indicated for SVT for rate control in patients with preserved left ventricular function. Atenolol and metoprolol are β1-blocking agents (cardioselective) available in both IV and oral formulations. Esmolol is a short-acting β1-agent that must be given in a bolus and then maintained through a continuous infusion. This may be advantageous in patients who may respond negatively to β2-blockade (eg, patients with chronic obstructive pulmonary disease). If an adequate response is not achieved after 5 minutes, the loading dose may be repeated and the infusion rate doubled. E. Calcium Channel Blockers Calcium channel blockers (ie, diltiazem and verapamil) are also indicated for rate control in SVT. They slow AV nodal conduction and prolong the AV nodal refractory period. Calcium channel blockers are contraindicated in atrial fibrillation or atrial flutter with rapid ventricular response when an accessory pathway such as Wolff-Parkinson-White syndrome exists, because it could lead to a life-threatening increase in the ventricular heart rate. Diltiazem is better tolerated in patients with impaired left ventricular function. F. Lidocaine Lidocaine is a class IB antidysrhythmic. It is commonly used for ventricular rhythms, both stable and unstable. Its use has largely been replaced by amiodarone and should only be used when amiodarone is not available. The initial dose in VF and pulseless VT is 1.0-1.5 mg/kg. Half of this dose may be repeated every 5-10 minutes, with a maximal total dose of 3 mg/kg. If successful in terminating the offending rhythm, a maintenance infusion of 3-5 mg/min may be administered. G. Magnesium Magnesium is indicated for patients who are known or suspected to have a low magnesium level, recurrent ventricular dysrhythmias, or for those with torsades de pointes. H. Procainamide Procainamide is a class IA antidysrhythmic, which is capable of suppressing both atrial and ventricular arrhythmias. It may be used in treatment of atrial fibrillation and atrial flutter (even in the presence of Wolff-Parkinson-White syndrome) in patients with preserved left ventricular function. In addition, procainamide may be used in SVT when vagal maneuvers and adenosine are ineffective. Its use should be avoided in patients with long QT intervals or when drugs that prolong the QT interval, such as amiodarone, have already been administered. FIBRINOLYTIC THERAPY Most cases of sudden cardiac death are secondary to an intravascular thrombosis; majority of these cases are related to either an intracoronary thrombus or massive pulmonary embolus. The primary goal in treating these patients is restoring perfusion immediately. In the case of myocardial infarction, the two methods to restore coronary blood flow are fibrinolytic therapy and percutaneous coronary intervention (PCI). Multiple studies have found that in centers that can provide rapid PCI, there is improved outcome and a decreased rate of reocclusion when compared to fibrinolytic therapy. There is evidence that PCI improves outcomes in patients with and without ST elevation upon initial presentation. However, the time from initial presentation to PCI should occur within 90 minutes. Many facilities are unable to perform PCI, and transfer the patient to centers with catheterization capabilities. If the patient is unable to undergo PCI in the 90-minute period, fibrinolytic therapy should be initiated if not contraindicated. If fibrinolytic therapy is to be initiated, it should ideally occur within 30 minutes of the patient's presentation for maximum benefit. Benefit is seen in those patients who present within 3 hours from the onset of symptoms. Fibrinolytic administration by prehospital providers was studied in order to reduce time to administration. No improvement in outcomes was found. Fibrinolytic therapy is currently not recommended for administration by prehospital providers. POSTRESUSCITATION STABILIZATION Following successful ROSC after cardiac arrest, mortality is often associated with refractory cardiac damage, central nervous system (CNS) injury, and sepsis. As described by the AHA, there are four objectives of postresuscitation stabilization after the patient has been transported to the emergency department: (1) optimize cardiopulmonary function and systemic perfusion, (2) identify the precipitating cause of the arrest, (3) prevent recurrence, and (4) begin measures to improve long-term survival and neurologic function. Initial data gathering is directed to address the patient's history, underlying condition, and current physiologic status. The history may be obtained from family, friends, medical records, and prehospital care providers. Evaluate the patient's status by physical examination, laboratories, radiologic studies, and continuous hemodynamic monitoring. Routine postresuscitation studies include complete blood count, electrolytes (including glucose and magnesium), cardiac enzymes, toxicology, arterial blood gas, and a portable chest X-ray. Additional studies might include bedside echocardiography, pulmonary artery catheterization, heart catheterization, and computed tomography. Direct initial interventions are toward maintaining stable hemodynamic parameters, with the primary goal to restore adequate perfusion. Attach full monitoring equipment, if not done previously. Place a Foley catheter to monitor urine output. Postresuscitation hypotension should be initially treated with small-volume (250-500 mL) boluses of crystalloid solution. Pulmonary artery catheterization or central venous pressure monitoring may aid in determining the patient's fluid status. Consider other causes of hypotension, such as pneumothorax and pericardial tamponade. If bolus fluid therapy fails, add vasoactive (eg, norepinephrine) or inotropic (eg, dopamine) agents. Hyperthermia increases the cerebral metabolic rate, which creates an imbalance between oxygen delivered and demanded. This can lead to anoxic cell death and initiate a subsequent systemic inflammatory response. Hypothermia, in contrast, can be beneficial to the patient. Studies using induced mild hypothermia (32-36°C or 89.6-96.8°F) maintained for 24 hours have demonstrated increased survival to discharge and improved neurologic recovery with comatose patients after ROSC. Many institutions have hypothermia protocols in place for postresuscitation patients. Hyperglycemia is associated with worsening prognosis following global ischemia from cardiac arrest. Tight glucose control is felt to increase survival and reduce the incidence of developing infectious complications in the postresuscitation period. TERMINATION OF RESUSCITATION Ideally, termination of resuscitation efforts should occur when there is no further chance of survival or meaningful neurologic recovery. Efforts to resuscitate the victim should be stopped after 20-30 minutes of appropriate ACLS protocol, with no ROSC. In addition, bedside echocardiography can be used to evaluate for cardiac kinetic motion. Wall motion suggests the possibility that PEA has been inadequately resuscitated, whereas the absence of activity is associated with death. One exception to the standard guidelines for termination of resuscitation is hypothermic cardiac arrest, which may occur from severe exposure or cold water drowning. Because of the protective effects of hypothermia, several cases of good neurologic recovery after prolonged resuscitation from hypothermic cardiac arrest have been documented. In general, continue resuscitation efforts until the patient has been rewarmed to a core temperature of 32-36°C (89.6-96.8°F).

For each of the following cardiovascular conditions, students should describe the underlying pathophysiology, presenting signs and symptoms, basic epidemiology, modifiable (when applicable) and non-modifiable risk factors, differential diagnosis, appropriate diagnostic studies, clinical intervention, treatment guidelines, pharmaceutical therapies, and health maintenance concerns: Vascular disease

PAD •Atherosclerotic disease of the peripheral arteries, especially the lower extremities. CLINICAL MANIFESTATIONS •Classic claudication most common symptom - intermittent lower extremity pain or cramping with ambulation, exercise, or movement and relieved with rest. •Atypical extremity pain: - 1) leg pain/carry on (exertional leg pain that does not cause the patient to stop walking) and 2) leg pain on exertion and rest is exertional leg pain that may occur at rest. •Ischemic rest pain: in advanced disease. Often described as a constant intractable, burning pain in the forefoot (soles of the feet), most common at night, aggravated by elevation, and relieved with foot dependency (the minor increase in blood flow due to gravity temporarily diminishes the otherwise intractable pain). PHYSICAL EXAMINATION: •Pulses: Decreased, weak, or absent pulses, bruits, Decreased capillary refill. Usually no edema. •Skin: Atrophic skin changes - muscle atrophy; skin that is thin, dry, and shiny; hair loss or slowed hair growth, thickened nails with slow growth, cool limbs to the touch, and areas of necrosis. •Nonhealing wound/ulcer: often starts as a traumatic wound that fails to heal). They are commonly found on the tips of the toes and between the digits. Ischemic ulcers also form at sites of increased focal pressure (eg, lateral malleolus and metatarsal heads). •Color: Foot & leg pallor on elevation. Dependent rubor dusky red color of the leg when it is placed in the dependent position (increase in blood flow due to gravity temporarily diminishes the pain). DIAGNOSIS •Ankle-brachial index: most useful screening test (simple, quick noninvasive). Normal ABI 1-1.3. -+PAD if ABI <0.90 (0.50 is severe). Rest pain if 0.2- 0.4. Tissue loss (ulcer, gangrene): 0 - 0.4. - >1.3 = possible noncompressible (calcified) vessels - may lead to a false reading. •Duplex ultrasonography: Performed to evaluate the further extent of vascular disease. A classic finding of PAD is low-velocity flow through the affected arteries. •Arteriography: gold standard. Usually only performed if revascularization is planned. MANAGEMENT •Supportive: First-line therapy - risk reduction (eg, smoking cessation associated with greatest benefit, hyperlipidemia, DM), exercise therapy (for those who can comply), and possibly pharmacologic therapy, rather than initial vascular intervention. Risk factor reduction: with lifestyle modifications foot care. Smoking cessation most important modifiable CAD risk factor. •Cilostazol most effective medical therapy to improve symptoms and increase walking distance in patients with claudication. Cilostazol is an ADP inhibitor (prevents platelet aggregation), phosphodiesterase-3 inhibitor that promotes direct arterial vasodilation, increases red blood cell flexibility, and suppresses the proliferation of vascular smooth muscle cells. •Pentoxifylline improves oxygen delivery (not as good as Cilostazol). •Platelet inhibitors: Aspirin or Clopidogrel to reduce the risk of Myocardial infarction and Stroke. Statins: indicated for all patients with PAD to reduce risk of MI and Stroke. •Revascularization: Percutaneous transluminal angioplasty (first-line revascularization procedure), bypass grafts, endarterectomy (last-line). VARICOSE VEINS • pilation of superficial veins due to failure of the venous valves in the saphenous veins, leading to retrograde flow, venous stasis, and pooling of blood. RISK FACTORS • Family history, female gender, increased age, standing for long periods, obesity, increased estrogen (eg, OCP use, pregnancy), chronic venous insufficiency. CLINICAL MANIFESTATIONS • Most are asymptomatic but may present due to cosmetic issues. • Dull ache or pressure sensation. Pain is worse with prolonged standing or sitting with the leg dependent and is relieved with elevation. • Physical examination: dilated visible veins, telangiectasias, swelling, discoloration, Venous stasis ulcers: severe varicosities resulting in skin ulcerations. + Mild ankle edema. MANAGEMENT • Conservative; compression stockings, leg elevation, pain control. • Ablation; catheter-based endovenous thermal ablation (laser or radiofrequency). • Ligation and stripping, sclerotherapy. CHRONIC VENOUS INSUFFICIENCY •Changes due to venous hypertension of the lower extremities as a result of venous valvular incompetency. •Most commonly occurs after superficial thrombophlebitis, after DVT or trauma to the affected leg. CLINICAL MANIFESTATIONS •Leg pain worsened with prolonged standing, prolonged sitting with the feet dependent •Leg pain improved with ambulation and leg elevation. •Pain classically described as a burning, aching, throbbing, cramping or "heavy leg". PHYSICAL EXAM FINDINGS •Stasis Dermatitis: itchy eczematous rash (inflammatory papules, crusts or scales), excoriations, weeping erosions & brownish or dark purple hyperpigmentation of the skin (hemosiderin deposition). •Venous stasis ulcers (especially at the medial malleolus) may be seen. •Dependent pitting leg edema, increased leg circumference, varicosities & erythema with normal pulse and temperature. •Atrophie blanch; atrophic, hypopigmented areas with telangiectasias & punctate red dots. MANAGEMENT •Nonoperative; initial management of choice for most includes compression stockings (cornerstone of treatment), leg elevation, skin care, exercise (improves the calf muscle pump), & weight management. •Treat the underlying cause. •Ulcer management; compression bandaging systems (eg, zinc impregnated gauze/Unna boots), wound debridement if needed, Aspirin (accelerates ulcer healing). •Surgical intervention usually reserved for patients not responsive to conservative therapy.

Aortic Stenosis RAPID REVIEW

Risk factors: advancing age, diabetes, hypertension Sx: dyspnea, chest pain, syncope PE: crescendo-decrescendo systolic murmur that radiates to the carotids, paradoxically split S2, S4 gallop. Murmur decreases with Valsalva Most commonly caused by degenerative calcification Treatment: aortic valve replacement

COMPLICATIONS OF MYOCARDIAL INFARCTION 4. Myocardial Rupture

The chief cause of myocardial rupture is mechanical failure of an infarcted ventricular wall. Clinical Findings Myocardial rupture is an uncommon cause of sudden death during acute myocardial infarction; it is responsible for only about 5% of deaths. Myocardial rupture is suggested by abrupt onset of hypotension with increased venous pressure (ie, cardiac tamponade). Pulseless electrical activity often occurs. Treatment and Disposition If echocardiography or bedside emergency ultrasound demonstrates a pericardial effusion, pericardiocentesis is indicated and can be performed under ultrasound guidance (see Chapter 6). When emergent ultrasound is not available to assess a patient for possible pericardial tamponade, blind pericardiocentesis may be lifesaving. Obtain emergency cardiac surgical consultation for immediate cardiac surgery. This is successful in the few cases in which rupture has been minimal with slow intrapericardial hemorrhage.

Coronary angiography

•Definitive diagnosis/gold standard. "Cath" outlines the coronary artery anatomy. Angiography also defines location & extent of coronary artery disease (CAD). Indications: 1. Confirm/exclude CAD in patients with symptoms consistent with CAD. 2. Confirm/exclude CAD in patients with negative noninvasive testing for CAD. 3. Patients who may possibly need revascularization (PTCA or CABG).

Anterior and Lateral Wall MI

•Complete occlusion of the left anterior descending artery (anterior) or left circumflex (lateral). ECG •Anterior MI: ST elevations > 1 mm in at least 2 contiguous anterior leads (V1 through V4) with or without reciprocal changes (ST depressions) in the inferior leads (I, III, aVF). •Lateral MI: ST elevations > 1 mm in at least 2 contiguous lateral leads (I, aVL, V5, V6) with or without reciprocal changes (ST depressions) in the inferior leads (Il, III, aVF). •Left bundle branch block (LBBB): Left bundle branch block, especially when new (or not known to be old), in a patient with symptoms of an acute MI is considered to be a "STEMI equivalent" and should be treated as a STEMI. Patients with a pre-existing left bundle branch block can be further evaluated using Sgarbossa's criteria: ST-segment elevation of 1 mm or more that is concordant with (in the same direction as) the QRS complex; ST-segment depression of 1 mm or more in lead V1, V2, or V3; ST-segment elevation of 5 mm or more that is discordant with (in the opposite direction) the QRS complex. Cardiac enzymes: positive. MANAGEMENT Initial: •Aspirin 325 mg (non-enteric coated) to be chewed and swallowed for faster absorption. •Nitroglycerin: 3 sublingual NTG tablets (0.4 mg) one at a time, spaced 5 minutes apart, or one aerosol spray under tongue every 5 minutes for 3 doses if patient has persistent chest discomfort, hypertension, or signs of heart failure and there is no sign of hemodynamic compromise (eg, right ventricular infarction) and no use of phosphodiesterase inhibitors (eg, for erectile dysfunction); add IV nitroglycerin for persistent symptoms. •Oxygen as needed if hypoxic to maintain O2 saturation >90%. Establish IV access. •Morphine for unacceptable, continued discomfort or anxiety related to myocardial ischemia after IV Nitrate administration when Nitrates fail to relieve the pain. Morphine is rarely used. •High-intensity statin (eg, Atorvastatin 80 mg daily or Rosuvastatin 20-40 mg daily) should be initiated as early as possible in all patients with STEMI. •Risk stratification - those with high-risk factors (eg, new Mitral regurgitation) should undergo immediate coronary angiography and revascularization because they are at extremely high risk of an adverse cardiovascular event in the short term. Acute management of STEMI: •Beta blockers (eg, Metoprolol tartrate 25 mg orally) to prevent further ischemia or life-threatening arrhythmias if no contraindications (eg, hypotension, cardiogenic shock, bradycardia, severe airway disease). •Dual antiplatelet therapy P2Y12 receptor antagonist (eg, Ticagrelor, Prasugrel, Clopidogrel) in addition to Aspirin to all patients. Consider adding a GP IIb/IIIa inhibitor (either Eptifibatide or Tirofiban) in patients not treated with an invasive approach. •Anticoagulant therapy: eg, Unfractionated Heparin, Enoxaparin, Bivalirudin, Fondaparinux. •ACE inhibitors indicated for all patients following STEMI who do not have contraindications. Reperfusion therapy: •Percutaneous coronary intervention (strongly preferred) or Fibrinolytics most important component of therapy and should be done as soon as possible, with certain exceptions. •Percutaneous coronary intervention (PCI) ideally within 90 minutes of ED presentation of PCI-capable hospital (or transfer to a PCI capable hospital within the first 120 minutes) & within 12 hours of chest pain onset. Restoration of blood flow for myocardial salvage & reduce mortality •Thrombolytics within 30 minutes of ED presentation is an alternative to catheterization if PCI is not possible & within 12 hours of chest pain onset.

Giant cell (temporal) arteritis

•Immune-mediated large- & medium-vessel granulomatous vasculitis of the extracranial branches of the carotid artery (eg, temporal, occipital, ophthalmic, & posterior ciliary arteries). PATHOPHYSIOLOGY: T-cells and monocytes are recruited to the vessel wall and result in an inflammatory response. •If not recognized & treated early, ischemic complications may cause permanent vision loss (15-25%). EPIDEMIOLOGY: •Same clinical spectrum as Polymyalgia Rheumatica - PMR & GCA frequently overlap; 20% of patients with PMR will get diagnosed with GCA later; In GCA, PM features are present in up to 50%. RISK FACTORS: •Women, >50 years of age - incidence rises steadily thereafter, peaking between the ages of 70 - 79 (80% of patients are 70 years and older). Northeastern Europeans and Caucasians. Smoking. CLINICAL MANIFESTATIONS: •Headache - new-onset headaches or change in baseline headaches is the most common systemic symptom (75%). Headache is usually temporal but can be occipital, periorbital, or non-focal as well. Headaches have an insidious onset and gradually progress over time, although they may spontaneously resolve rarely, even in the absence of treatment. Scalp tenderness while combing or brushing hair is frequent and can be focal in the temporal areas or diffuse. •Jaw claudication - mandibular pain, discomfort, or fatigue brought on by chewing, talking, or using the jaw, relieved by stopping (exertional ischemia) seen in ~50%. Jaw Claudication occurs secondary to ischemia of the masseter muscle, which is supplied by the maxillary artery. •Visual changes: ocular involvement may include eye pain, monocular (rarely binocular) vision loss, diplopia, & Amaurosis fugax (transient monocular vision loss). •Abnormal superficial temporal artery: absent or decreased pulsation, local tenderness to palpation (including scalp tenderness), localized erythema, beading (nodularity), or thickening. A clinically normal temporal artery does not rule out GCA. •Constitutional symptoms: fever (usually low-grade), fatigue, weight loss, anorexia, malaise. DIAGNOSIS: •Elevated ESR & CRP •Temporal artery biopsy: Criterion standard - histopathology reveals inflammatory infiltrate surrounding a fragmented internal elastic lamina within the media of an arterial wall. The infiltrate consists predominantly of mononuclear cell infiltration or granulomatous inflammation, or multinucleated giant cells. Biopsy may be normal as there may be skip lesions of normal tissue. •Scheduling of temporal artery biopsy should never interfere with or delay initiation of treatment in a patient with a high likelihood of GCA, since delay can put the patient at risk for complications (eg, vision loss). The yield of biopsy is still very high up to 2 weeks after initiation of corticosteroids. MANAGEMENT: •High-dose systemic glucocorticoids; initiated once GA is suspected to prevent blindness and suppress disease activity (do not delay treatment to biopsy or for biopsy results). IV Methylprednisolone pulses for 3-5 days prior to oral may be used in some with visual changes. •Low-dose Aspirin: Helps to reduce the risk of vision loss, transient ischemic attacks, & Strokes in GCA and can be considered as adjuvant therapy if no contraindications exist. •IL-6 inhibitors: Tocilizumab, an IL-6 inhibitor, may be an alternative in patients who are intolerant to corticosteroids. IL-6 levels are significantly elevated in patients with GA, and IL-6 is thought to play a pathogenic role in GCA by activating the T-cells and promote IFN-gamma release from the T-cells.

aortic regurgitation

•Incomplete aortic valve closure, leading to regurgitation of blood through the aortic valve into the left ventricle during diastole. ETIOLOGIES •Chronic AR: usually a slowly progressive disease over decades before becoming symptomatic. Causes include congenital bicuspid valve, Rheumatic heart disease, Hypertension, autoimmunity, Syphilis, Collagen vascular disease: Ehlers-Danlos syndrome, Marfan syndrome. •Acute AR: acute onset is usually a medical emergency due to the inability of the left ventricle to rapidly adapt to the abrupt increase in end-diastolic volume caused by the regurgitant flow. The two most common causes of acute AR of a native aortic valve are Endocarditis and Aortic dissection. If not surgically corrected, acute severe AR commonly results in cardiogenic shock. CLINICAL MANIFESTATIONS: •Chronic AR: Patients may be asymptomatic for decades until severe left ventricular dysfunction has developed - exertional dyspnea, orthopnea, paroxysmal nocturnal dyspnea, decreased exercise tolerance, fatigue. Angina, palpitations. •In acute severe AR, compensatory mechanisms of the LV do not develop rapidly enough to adapt to the regurgitant volume load, increased preload & afterload. LV diastolic pressures rise rapidly leading to acute pulmonary edema (pulmonary capillary wedge pressure) and cardiogenic shock. PHYSICAL EXAMINATION •High-pitched, blowing (soft), decrescendo or sustained, diastolic murmur best heard over Erb's point (third or fourth intercostal space at the left sternal border). •Increased murmur intensity: sitting up while leaning forward, holding breath in end-expiration, increased venous return (eg, squatting, supine, and leg raise), increased afterload (eg, handgrip), isometric exercise. •Decreased murmur intensity: decreased venous return (eg, Valsalva, standing), inspiration. Decreased afterload (eg, Amyl nitrate). •Austin-Flint murmur: mid-late diastolic rumble at the apex secondary to retrograde regurgitant jet competing with antegrade flow from the left atrium into the left ventricle. •Bounding pulses due to increased stroke volume. Pulsus bisferiens can be seen (especially with combined AS + AR or severe AR). Wide pulse pressure (increased SBP and decreased DBP) Classic Signs of WIDENED PULSE PRESSURE in AR/AI (seen ONLY with chronic AR/AD) SIGN DESCRIPTION Water Hammer pulse Swift upstroke & rapid fall of radial pulse accentuated with wrist elevation. Corrigan's pulse Similar to water hammer pulse but referring specifically to the carotid artery. Hill's sign Popliteal artery systolic pressure > brachial artery by 60mmHg (most sensitive). Duroziez's sign Gradual pressure over femoral artery - systolic and diastolic bruits. Traube's sound (pistol shot) Double sound heard @ femoral artery c partial compression of femoral artery. De Musset's sign Head-bobbing with each heartbeat (Low sensitivity). Müller's sign Visible systolic pulsations of the uvula. [Quincke's pulses Visible fingernail bed pulsations with light compression of fingernail bed. DIAGNOSTIC STUDIES •Echocardiogram: Primary diagnostic test of choice to diagnose AR - regurgitant jet. •Cardiac catheterization: definitive diagnosis (may be used prior to surgery). MANAGEMENT •Medical therapy: afterload reduction improves forward flow (ACE, ARBs, Nifedipine, Hydralazine). •Surgical therapy: definitive management. Indicated in symptomatic AR, asymptomatic with LV decompensation (EF < 55% - patients with AR normally have higher than normal ejection fraction). The management for acute severe AR is emergency aortic valve replacement or repair.

mitral regurgitation

•Incomplete closure of the mitral valve apparatus, leading to retrograde blood flow from the left ventricle (LV) into the left atrium (LA) during systole. PATHOPHYSIOLOGY • Abnormal, retrograde blood flow from the left ventricle into the left atrium, leading to increased left atrial pressure, subsequent LV volume overload & preload, & increased pulmonary pressure. • With chronic MR, left atrial dilation and left ventricular hypertrophy are compensatory consequences to better tolerate the regurgitant volume. Later it decompensates. ETIOLOGIES • Leaflet abnormalities: Mitral valve prolapse most common cause in the US (Rheumatic fever in developing countries). Endocarditis, valvulitis, annulus or LV dilation, Marfan syndrome. • Papillary muscle dysfunction: myocardial ischemia or infarction, cardiomyopathy. • Ruptured chordae tendineae: collagen vascular disease, dilated cardiomyopathy. CLINICAL MANIFESTATIONS • Chronic: Many are asymptomatic. Heart failure symptoms (eg, exertional dyspnea most common fatigue), atrial fibrillation, hemoptysis, hypertension. • Acute: Significant dyspnea at rest, pulmonary edema, hypotension, tachypnea, hypoxemia, cyanosis, decreased cardiac output, and possible cardiogenic shock. Usually due to either papillary muscle in Acute myocardial infarction or in the setting of Infective endocarditis. PHYSICAL EXAMINATION • High-pitched, blowing, holosystolic murmur best heard at the apex, often with radiation to the left axilla, subscapular region, or the upper sternal borders. • Increased murmur intensity: left lateral decubitus position, expiration, isometric exercise, increased venous return (eg, squatting, leg raise, lying supine); increased afterload (eg, handgrip) due to increased atrial pressure. • Decreased murmur intensity; decreased venous return (eg, Valsalva, standing), inspiration, decreased afterload (eg, Amyl nitrate). Widely split S2, laterally displaced PMI, S3. Soft (diminished) S1 if severe. • With MVP, the murmur may appear in mid-late systole with or without mid-systolic click. DIAGNOSIS • Echocardiogram: primary and most commonly used noninvasive test to diagnose MR - allows for assessment of MR etiology, morphology, severity, assessment of ventricular size & function, and the need for mitral valve repair. Hyperdynamic LV, regurgitant jet in MR. • ECG: nonspecific - left atrial enlargement, LVH, atrial fibrillation. • Chest radiograph: nonspecific - left atrial enlargement, LVH, pulmonary edema MANAGEMENT • Hypertension + MR: afterload reducers eg, ACE inhibitors (ACEs), Angiotensin receptor blockers (ARBs), Hydralazine, Nitrates. ACE & ARBs have been used to delay MR progression. Surgical • Repair is generally preferred over replacement when possible (decreased recurrence of MR after repair, lower rate of mortality, & better late outcome). Replacement if extensive tissue destruction. • Indications: left ventricular dysfunction even in the absence of symptoms (eg, ejection fraction 60% or less or left ventricular end systolic dimension is 40 mm or greater), refractory to medical therapy, moderate to severe symptomatic MR. Due to abrupt hemodynamic decompensation, acute MR typically requires urgent surgical or percutaneous intervention.

Myocarditis

•Inflammation of the heart muscle resulting in myocardial necrosis & degeneration. ETIOLOGIES: •Idiopathic: 50% of cases. In some, viral particles may be found in biopsy specimens. •Infectious: Viral most common infectious cause, especially the enteroviruses [Echoviruses & Coxsackieviruses (Coxsackie B most common viral cause; Parvovirus B-19, Adenoviruses, Herpes viruses (eg, CMV, Epstein Barr virus, HHV 6), Influenza virus, HCV, HIV. Bacterial: Borrelia burgdorferi (Lyme disease), Mycoplasma pneumoniae. Corynebacterium diphtheriae. Parasitic: Trypanosoma cruzi (Chagas disease), Toxoplasma gondii, Trichinella spiralis. •Autoimmune: SLE, RA, Wegener's granulomatosis, Kawasaki disease, Giant cell arteritis. •Medications: Clozapine, Methyldopa, antibiotics, Isoniazid, Cyclophosphamide, Indomethacin, Phenytoin, sulfonamides, Lithium, TCA antidepressants, Doxorubicin. •Other: Cocaine, alcohol, and uremia. EPIDEMIOLOGY: •Most common in young previously healthy adults, typically between 20-50 years. CLINICAL MANIFESTATIONS: Clinical presentation of myocarditis is highly variable and may include any of the following: •Viral prodrome - flu-like illness that may consist of fever, myalgias, malaise, vomiting, and/or diarrhea for 7-14 days. •New onset or worsening HF: syndrome of Heart failure and Dilated cardiomyopathy in the absence of CAD and known causes of HF (most common in adults) - over 2 weeks to 3 months after respiratory or GI infection with symptoms including dyspnea, fatigue, exercise intolerance, peripheral edema, chest discomfort, and arrhythmias (sinus tachycardia most common) with echocardiographic evidence of global or regional LV and/or RV dysfunction. •ACS-like: patients present similar to Acute coronary syndrome with acute chest pain 1-4 weeks after respiratory or GI infection (may be recurrent), with ST/T changes (including ST segment elevation or depression and T wave inversions), elevated cardiac biomarkers (g, Troponin) echocardiographic evidence of global or regional LV and/or RV dysfunction, with no evidence of significant coronary artery disease on angiography. •Pericarditis: at least two of the following - typical chest pain (eg, pleuritic, postural), pericardial friction rub, classic ECG changes (widespread ST segment elevation), and/or new or worsening pericardial effusion. •Myopericarditis: diagnosis of Acute pericarditis PLUS suggestive symptoms (g, palpitations, dyspnea, chest pain) & certain ECG features more than normal variants not documented previously [ST/T abnormalities, atypical ECG changes such as localized ST-elevation (inferolateral or anterolateral) and T-wave inversion before ST-segment normalization, cardiac arrhythmias - supraventricular or ventricular tachycardia or frequent ectopy, atrioventricular block)] OR focal or diffuse decreased LV function of undetermined age by an imaging study); 1 of the following features: elevated cardiac biomarkers (creatine kinase-MB fraction, or troponin I or T), OR new onset of focal or diffuse decreased LV function by an imaging study, OR abnormal imaging consistent with myocarditis (MRI with gadolinium, gallium-67 scanning, anti-myosin antibody scanning); absence of any other cause. •Life-threatening condition: in the absence of CAD and known causes of HF with one or more of the following - life-threatening arrhythmias, cardiogenic shock occurring around 2 weeks after a distinct viral prodrome. •Children will often present with grunting respirations and intercostal retractions. •Other: Megacolon. PHYSICAL EXAMINATION: •Vital signs are often abnormal (eg, fever, tachycardia, tachypnea, and occasionally hypotension). •Heart failure: signs of fluid overload (eg, peripheral edema, elevated jugular venous pressure, and pulmonary crackles), S3 gallop (third heart sound reflecting filling of a dilated ventricle). If left or RV dilation is severe, functional mitral or tricuspid regurgitation may occur, •Pericarditis: pericardial friction rub and/or effusion (muffled heart sounds). DIAGNOSIS: •In a patient with new-onset of Heart failure or chest pain or arrhythmia, positive cardiac biomarkers, absence of traditional coronary risk factors, and a history of preceding viral illness or infection, Acute myocarditis should be in the differential diagnosis. Chest radiograph: cardiomegaly classic. ECG: •Nonspecific - sinus tachycardia most common, normal or may show Pericarditis (eg, diffuse ST elevations and PR depressions in precordial leads), widened QRS patterns, low voltage, prolonged QT, variable atrioventricular (AV) blocks. An acute myocardial infarction pattern may be seen. Labs: •Cardiac biomarkers may be positive (eg, Troponin) due to myocardial damage & myonecrosis. •Erythrocyte sedimentation rate (ESR), C-reactive protein (CRP), and WBC count may be elevated. •Viral antibody titers include coxsackievirus B, HIV, CMV, Ebstein-Barr virus, hepatitis and influenza. Echocardiogram: •Should be ordered in all patients and is also helpful to rule out other causes. •Nonspecific findings include ventricular dysfunction (eg, decreased left ventricular function, decreased ejection fraction, global hypokinesis, regional wall motion abnormalities). Endomyocardial biopsy: •Definitive diagnosis gold standard - histopathologic evidence of myocardial tissue necrosis and increased inflammatory cellular infiltrates (g, histiocytic, lymphocytic, mononuclear, eosinophilic, granulomatous, or giant cells). •Indications: usually reserved for severe or refractory cases after other causes have been excluded. MANAGEMENT: Supportive management •Mainstay of treatment - including supplemental oxygen and fluid status optimization. •Management goals are to preserve left ventricular function and may include limitation of activity up to standard Heart failure treatment. •Patients with mild symptoms do improve spontaneously but recovery may take months. •All patients with Myocarditis should receive routine follow-up, including serial echocardiography (or other cardiac imaging). Standard Heart failure treatment: •HF with reduced EF treatment - eg, triple therapy with ACE inhibitor or Angiotensin receptor blocker, evidence-based Beta blocker (eg, Carvedilol, Bisoprolol, extended release Metoprolol), and diuretics for symptom control as well as to decrease preload and volume overload. •Mineralocorticoid receptor antagonist (eg, Spironolactone) in patients with persistent symptomatic HF with left ventricular ejection fraction (EF) 35% or less. Things to avoid: •Patients with Myocarditis should avoid NSAIDs, cardiotoxic medications, heavy alcohol consumption, and exercise (physical activity should be restricted during the acute phase). •NSAIDs should be withheld as they impede healing of the myocardium and exacerbate the inflammatory process. •Digoxin should be avoided acutely (may increase proinflammatory cytokines, worsening the injury).

Mobitz 1 Second Degree AV Block (Wenckebach)

•Interruption of electrical impulse at the AV node, resulting in occasional non-conducted impulses. •PATHOPHYSIOLOGY: AV node dysfunction (commonly above the bundle of HIS). ETIOLOGIES •Often a normal variant in individuals with high vagal tone without structural heart disease. •Inferior wall MI (AV node ischemia), AV nodal blocking agents (eg, Beta blockers, Digoxin, Calcium channel blockers), myocarditis due to Lyme, Hyperkalemia, cardiac surgery. CLINICAL MANIFESTATIONS •Asymptomatic in most cases. •Bradycardia-related decreased perfusion - eg, fatigue, dizziness, dyspnea, chest pain, syncope, or in severe cases (hypotension or altered mental status). DIAGNOSIS •ECG: progressive lengthening of the PR interval until an occasional non-conducted atrial impulse (dropped QRS complex) MANAGEMENT •Asymptomatic: no treatment, observation. Cardiac consult in some cases. •Symptomatic: Atropine first-line, Epinephrine. •Pacemaker definitive management in persistent cases.

Reperfusion in ST Elevation MI (STEMI)

•Mainstay of treatment - done within 12 hours of symptom onset or if ongoing chest pain. •Either PCI (percutaneous transluminal coronary angioplasty) or thrombolytics PCI (Percutaneous Coronary Intervention): •Best within 3 hours of symptom onset (especially within 90 minutes). PCI superior to thrombolytics. •Good especially for cardiogenic shock, large anterior MI, prior CABG, and if thrombolytics are contraindicated. •Coronary Artery Bypass Graft: 3-vessel disease, L main coronary artery, left ventricle EF <40%. THROMBOLYTIC (FIBRINOLYTIC) THERAPY: Used if PCI is not an option/unable to get PCI early.

For each of the following cardiovascular conditions, students should describe the underlying pathophysiology, presenting signs and symptoms, basic epidemiology, modifiable (when applicable) and non-modifiable risk factors, differential diagnosis, appropriate diagnostic studies, clinical intervention, treatment guidelines, pharmaceutical therapies, and health maintenance concerns: Venous thrombosis

•Most important consequence is pulmonary embolism (50%): both are manifestations of a single entity. RISK FACTORS (Virchow's triad) •Intimal damage: trauma, infection, inflammation Endothelial damage triggers the clotting cascade. •Stasis eg, Immobilization or prolonged sitting >4hours, surgery (typically within 12 weeks of surgery or trauma), Stroke with hemiplegia or immobility. •Hypercoagulability: eg, Protein C or S Deficiency, Factor V Leiden mutation, antithrombin III deficiency, oral contraceptive use, malignancy, pregnancy, smoking. CLINICAL MANIFESTATIONS •DVT should be suspected in patients who present with leg swelling, pain, warmth, and erythema. Physical examination: •Unilateral calf swelling (edema) 3 cm or greater than the other leg, measured 10 cm below the tibial tuberosity most specific sign. Calf pain & tenderness. May be warm to palpation. •Homan sign: deep calf pain with foot dorsiflexion while squeezing the calf (not reliable). DIAGNOSIS •Venous Duplex Ultrasound: usually first-line imaging. Most DVTs originate in the calf. •Compression ultrasonography (CUS) with Doppler is the diagnostic test of choice in patients with suspected DVT. Non-compressibility of the affected vein = DVT. •D-dimer: highly sensitive but not specific. There are 2 main uses of D-dimer: negative D-dimer with a low-risk for DVT can exclude DVT as the diagnosis. In a patient with moderate risk, a positive D-dimer and a negative initial ultrasound, serial ultrasounds are recommended. In general, any positive VT should be followed by ultrasonography. •Contrast venography: definitive diagnosis (gold standard). It is invasive, difficult to perform, and rarely used. CT venography and MR venography rarely used. MANAGEMENT •Anticoagulation: first-line treatment for most patients with DVT (eg, popliteal, femoral, iliac veins). Options include (1) Low molecular weight heparin + Warfarin, (2) LMWH + either Dabigatran or Edoxaban or (3) monotherapy with either Rivaroxaban or Apixaban. A minimum of 3 months of oral therapy has been suggested after a first episode of DVT or PE. •IVC filter: 3 main reasons for IVC filter placement: recurrent DVT/PE despite adequate anticoagulation OR stable patients in whom anticoagulation is contraindicated OR right ventricular dysfunction with an enlarged RV on echocardiogram. •In general, subcutaneous LMWH (eg, Enoxaparin) is preferred over IV Unfractionated Heparin (UFH) or subcutaneous UFH in most pregnant patients with DVT/PE because it is easier to use, appears to be more efficacious, and has a better safety profile. •Thrombolysis or Thrombectomy: generally not performed (reserved for massive DVT or if severe) RISK FACTORS VENOUS THROMBOEMBOLISM (VTE) RECOMMENDED DURATION OF THERAPY 1st event with reversible or time-limiting RF for VTE at least 3 months (Risk factors: trauma, surgery, OCPs etc). 1st episode of IDIOPATHIC VT (no malignancy) Proximal DVT or PE Distal DVT Long-term anticoagulation 3 months if severely symptomatic distal DVT No tx & surveillance (ultrasound) if asymptomatic distal DVT LMWH preferred as initial & long-term therapy. Pregnancy LMWH as initial & long-term therapy. Warfarin or direct oral Malignancy anticoagulants are alternatives to LMWH in these patients. 2016 ACCP guidelines: novel oral anticoagulants (Apixaban, Dabigatran, Edoxaban, Rivaroxaban) are preferred over Warfarin therapy in the management of DVT/PE (if no cancer is present).

Atrial fibrillation

•Multiple irritable atrial foci fire at fast rates. •Atrial fibrillation is the most common chronic arrhythmia. Most patients are asymptomatic. •Similar to Atrial flutter, there is an increased risk of atrial thrombus formation that can lead to cerebral &/or systemic embolization (eg, acute ischemic stroke). TYPES •Paroxysmal: self-terminating within 7 days (usually <24 hours). +/- Recurrent. •Persistent; fails to self-terminate, lasts >7 days. Requires termination (medical or electrical). •Permanent: persistent AF >1 year (refractory to cardioversion or cardioversion never tried). •Lone: paroxysmal, persistent or permanent without evidence of heart disease. RISK FACTORS: •Cardiac disease: eg, hypertension, valvular heart disease, Heart failure ischemia. Advanced age, Pulmonary disease eg, Obstructive sleep apnea, PE. Metabolic: obesity (BMI >30 kg/m2), Chronic kidney disease, electrolyte imbalance (eg, Hypomagnesemia, Hypokalemia), Endocrine: Diabetes mellitus, Hyperthyroidism, Pheochromocytoma. Alcohol consumption. Omega-3 fatty acid use. Inflammation. CLINICAL MANIFESTATIONS •As many as 90% of Atrial fibrillation episodes are asymptomatic. •Symptomatic: palpitations, dizziness, fatigue, generalized weakness, poor exercise tolerance, mild dyspnea, and presyncope. These signs and symptoms in patients reflect decreased cardiac output as a result of rapid ventricular rate. •Unstable: symptoms are due to hypoperfusion and can include significant hypotension (eg, systolic BP in double digits), altered mental status, refractory chest pain (uncontrolled angina or ischemia), and decompensated Congestive heart failure (CHF). DIAGNOSIS •ECG: Irregularly irregular rhythm with fibrillatory waves (no discrete P waves). •Often atrial rate >250 beats per minute. •The AV nodal refractory period determines the ventricular rate. •Cardiac monitoring: a Holter monitor or telemetry can be used if Atrial fibrillation is not seen on an ECG but is suspected. MANAGEMENT Stable: •Rate control: Beta blockers (eg, Metoprolol, Atenolol, or Esmolol) OR non-dihydropyridine Calcium channel blockers (eg, Diltiazem, Verapamil) to slow AV node conduction. •Digoxin is another option for rate control but is usually reserved for patients in whom Beta blockers or CCs are contraindicated [eg, severe heart failure (NYHA class III or IV), hypotension]. Unstable: •Direct current (synchronized) cardioversion. Long-term: •Rate control usually preferred over rhythm control for long-term management. •Direct current (synchronized cardioversion) or pharmacologic cardioversion. •Radiofrequency catheter ablation or surgical "MAZE" procedure. •CHA2DS2-VASc criteria for nonvalvular Atrial fibrillation to determine the patient's yearly thromboembolic risk, in order to select the appropriate anticoagulation regimen. In patients with AF and elevated CHA2DS2-VASc score of 2 or more, oral anticoagulation is recommended. CARDIOVERSION •Direct current (synchronized cardioversion) or pharmacologic cardioversion. Cardioversion is most successful when performed within 7 days after the onset of Atrial fibrillation. •Transesophageal echocardiogram done prior to cardioversion to ensure there are no atrial clots. •AF > 48 hours: initiate anticoagulation therapy for at least 3 weeks before & at least 4 weeks after cardioversion. If cardioversion is required sooner, anticoagulate with IV Heparin & perform TEE as close to the time of cardioversion as possible. If thrombus is observed or suspected based on TEE, delay cardioversion. Rate control & therapeutic anticoagulation required for 4 week minimum. •AF < 48 hours cardioversion may be attempted as soon as possible often with anticoagulation. •Anticoagulation must be continued for 4 weeks after cardioversion. With effective anticoagulation, the stroke risk is decreased 3-fold after 4 weeks of anticoagulation. CANDIDATES FOR ANTICOAGULATION WITH NONVALVULAR ATRIAL FIBRILLATION OR FLUTTER •CHA2DS2-VASc score for nonvalvular Atrial fibrillation and A flutter assesses patients' risk for embolization. Chronic oral anticoagulation (eg, Novel oral anticoagulants or Warfarin) is recommended for moderate to high risk (score of 2 or greater). •The use of anticoagulant therapy has been shown to reduce embolic risk by 70%. ANTICOAGULANT AGENTS: 1. Non-vitamin K antagonist oral anticoagulants (NOACs) are recommended over Warfarin in most cases due to similar or lower rates of major bleeding as well as lower risk of ischemic stroke, convenience of not having to check the INR, & less drug interactions. Dabigatran: direct thrombin inhibitor (binds & inhibits thrombin). Factor Xa inhibitors: Rivaroxaban, Apixaban, Edoxaban. 2. Warfarin: Indications: may be preferred in some of the following patients - some with severe chronic kidney disease, contraindications to the NOAC (eg, HIV patients on protease inhibitor-based therapy, on CP450-inducing antiepileptic medications such as Carbamazepine, Phenytoin etc.), patients already on Warfarin who prefer not to change, cost issues (Warfarin is less expensive). Warfarin usually bridged with heparin until Warfarin is therapeutic. Monitoring: International Normalized Ratio (INR) goal of 2-3. Prothrombin Time (PT). 3. Dual antiplatelet therapy: (eg, Aspirin + Clopidogrel). Anticoagulant monotherapy is superior to dual antiplatelet therapy. Dual antiplatelet therapy may be reserved for patients who cannot be treated with anticoagulation (for reasons OTHER than bleeding risk).

Mitral stenosis

•Narrowing of the mitral valve orifice, leading to obstruction of flow from the left atrium (LA) to the left ventricle (LV) & backflow of blood into the left atrium. PATHOPHYSIOLOGY •Obstruction of flow from LA to LV 2ry to narrowed mitral orifice - blood backs up into the left atrium. •increased L-atrial pressure/volume overload - pulmonary congestion -> pulmonary HTN -> CHF. ETIOLOGIES •Rheumatic heart disease is almost always the cause. Most common in the 3rd/4th decade. •Congenital, left atrial myxoma, thrombus, valvulitis (SLE, amyloid, carcinoid). CLINICAL MANIFESTATIONS •Pulmonary: exertional dyspnea (most common symptom), exercise intolerance, fatigue, hemoptysis, cough, frequent bronchitis, pulmonary HTN (if rheumatic, may occur in 20s - 30s). •Atrial fibrillation: secondary to atrial enlargement -> thromboembolic events (ex. CVA). •Right-sided heart failure; due to prolonged pulmonary hypertension. •Mitral faces = ruddy (flushed) cheeks with facial pallor (chronic hypoxia). •Signs of left atrial enlargement: dysphagia (esophageal compression), Ortner's syndrome: recurrent laryngeal nerve palsy due to compression by the dilated left atrium -> hoarseness. PHYSICAL EXAMINATION •Prominent (loud) S1 due to forceful closure of mitral valve. Opening snap: (OS) may be heard after the A2 component of the second heart sound (S2) due to forceful opening of the mitral valve when the pressure in the left atrium > than the left ventricle. •Loud P2: The P2 (pulmonic) component of the second heart sound (S2) will be loud if severe Pulmonary hypertension due to Mitral stenosis occurs. •Mitral faces; chronic severe MS, Pulmonary hypertension, cardiac output, & vasoconstriction may lead to cutaneous vasodilation results in pinkish-purple patches (ruddy appearance) on the cheeks. •Low-pitched, mid-diastolic, rumbling murmur best heard at the mitral area/apex (left fifth intercostal space at the mid-clavicular line). Often heard after the opening snap. Best heard with the bell of the stethoscope at the apex with the patient lying on the left side in held expiration. •Increased murmur intensity: left lateral decubitus position, expiration, isometric exercise, increased venous return (eg, squatting, leg raise, lying supine). •Decreased intensity: decreased venous return (eg, Valsalva, standing), inspiration. Amyl nitrate. •Increased severity of MS: (1) shorter A2-OS interval (as the MS progresses and left atrial pressure is higher, the OS occurs earlier after S2 or A2), (2) prolonged murmur duration. DIAGNOSIS •ECG: left atrial enlargement (P wave > 3 mm, biphasic P wave in V1 and V2), atrial fibrillation, pulmonary hypertension (RVH, right axis deviation). •Echocardiography: most useful noninvasive tool. •Chest radiograph: left atrial enlargement (eg, straightening of the left border, prominent pulmonary arteries, posterior displacement of the esophagus, elevation of left mainstem bronchus). MANAGEMENT •Percutaneous balloon valvuloplasty: best treatment for symptomatic MS in younger patients with noncalcified valves or if refractory to medical therapy. •Valve replacement; reserved if mitral valvuloplasty is contraindicated or unfavorable valve morphology. •Medical: diuretics and sodium restriction for edema & volume overload. Rate control of Atrial fibrillation with Beta blockers, Calcium channel blockers, or Digoxin. Anticoagulation if A fib.

Aortic Stenosis (AS)

•Pathophysiology: LV outflow obstruction leads to a fixed cardiac output, increased afterload, Left ventricular hypertrophy (LVH), and eventually LV failure. •Most common valvular disease. Symptoms usually occur when Aov orifice <1 cm2 (Normal 3-4 cm2). ETIOLOGIES •Degenerative; calcifications, wear and tear, especially > 70 years of age. •Congenital & Bicuspid valve common in patients < 70 years of age. •Rheumatic heart disease: may be isolated or accompanied with Aortic regurgitation. CLINICAL MANIFESTATIONS •Once symptomatic, lifespan is dramatically reduced. Dyspnea is the most common symptom. •Exertional dyspnea is often the most common presenting symptom in patients with AS. Decreased exercise tolerance. Increased LV filling pressures with exercise, diastolic dysfunction, & inability of the LV to increase cardiac output during exercise - fixed cardiac output. •Angina: (5 year mean survival if valve not replaced), syncope (3 years), and CHF (2 years). PHYSICAL EXAMINATION •Harsh, low-pitched, mid-late peaking, systolic, crescendo-decrescendo murmur best heard at the right upper sternal border (R 2nd intercostal space at base) radiates to the carotid arteries. •Increased murmur intensity: sitting while leaning forward (all aortic murmurs), increased venous return (eg, squatting, supine, and leg raise), expiration (increased flow to the left side during expiration), and decreased afterload (inhalation of Amyl nitrate). •Decreased-murmur intensity: decreased venous return (eg, Valsalva, standing), inspiration (decreased flow to the left side of the heart), or increased afterload (handgrip). •Weak, delayed carotid pulse (pulsus parvus et tardus), narrow pulse pressure (low stroke volume). •Fourth heart sound (S4), also known as an atrial gallop, occurs during left ventricular (LV) filling when vigorous left atrial contraction forces blood into a noncompliant (stiff and hypertrophic) LV. DIAGNOSTIC STUDIES •Echocardiogram: test of choice small aortic orifice, LVH, thickened/calcified aortic valve. •ECG: left ventricular hypertrophy classic. Left atrial enlargement, atrial fibrillation. •Chest radiograph: nonspecific - postaortic dilatation, aortic valve calcification, pulmonary congestion. •Cardiac catheterization: definitive diagnosis (may be used prior to surgery). MANAGEMENT •Surgical therapy: aortic valve replacement only effective treatment (treatment of choice) Indications: symptomatic AS, severe AS [eg, aortic area < 0.8 cm? (normal 3-4cm2), or decreased ejection fraction (<50%). -Mechanical: prolonged durability but thrombogenic (eg, stroke), bleeding. Must be placed on long-term anticoagulant therapy. -Bioprosthetic: less durable but minimally thrombogenic (usually used in patients that that are not candidates for anticoagulant). Heterograft (porcine valve); pericardial. •Percutaneous aortic valvuloplasty (PAV): results in 50% fAV area, but 50% restenosis at 6-12 months so used as a bridge to AVR, if not a surgical candidate, or in pediatric patients. •Intraaortic balloon pump; used for temporary stabilization as a bridge to valve replacement. •Medical therapy: no medical treatment truly effective. No exercise restrictions in mild AS. Severe AS prior to surgery: •Because patients are dependent on preload to maintain cardiac output -> avoid physical exertion/venodilators (eg, nitrates)/negative inotropes (Ca*2 channel blockers, ß-blockers).

For each of the following cardiovascular conditions, students should describe the underlying pathophysiology, presenting signs and symptoms, basic epidemiology, modifiable (when applicable) and non-modifiable risk factors, differential diagnosis, appropriate diagnostic studies, clinical intervention, treatment guidelines, pharmaceutical therapies, and health maintenance concerns: Cardiac tamponade

•Pericardial effusion causing significant pressure on the heart, impeding cardiac filling, leading to decreased cardiac output and shock (medical emergency). •The rate of accumulation of fluid is more critical than the volume. Rapid accumulation of as little as 150ml of fluid can cause tamponade, while as much as 1 liter can slowly accumulate if compliant. •Etiologies: complication of Pericarditis or trauma. Malignancy most common nontraumatic cause. CLINICAL MANIFESTATIONS •Beck's triad: 3 "Ds" - Distant (muffled) heart sounds, Distended neck veins (increased JVP), and Decreased blood pressure (systemic hypotension). Classic but uncommon. •Pulsus paradoxus: exaggerated (>10 mmHg) decrease in systolic blood pressure with inspiration. •Dyspnea, fatigue, peripheral edema, shock, reflex tachycardia is common, cool extremities. DIAGNOSTIC STUDIES •Patients suspected of having cardiac tamponade should be evaluated with an ECG, chest radiograph, and echocardiography (echocardiogram may be performed first if unstable). •Echocardiogram: pericardial effusion + diastolic collapse of cardiac chambers. •ECG: signs of pericardial effusion (low voltage QRS complexes, electrical alternans). •Chest radiograph: may show an enlarged cardiac silhouette. MANAGEMENT •Pericardiocentesis (immediate) removal of the fluid to relieve the elevated intracardiac pressure and improve hemodynamic status (can be done percutaneous vs. surgical). Volume resuscitation (eg, saline or blood) or vasopressors if needed. •Treat the underlying cause eg, High-dose NSAIDs, with or without Colchicine for Acute pericarditis. High-dose Aspirin for Dresser syndrome. •Pericardial window definitive management for drainage if recurrent.

A hypertensive emergency is defined as having elevated blood pressures (systolic > 180 mm Hg or diastolic > 120 mm Hg) with concomitant end-organ damage. Most commonly, the targeted organs are the brain, heart, aorta, kidneys, or eyes. Examples of hypertensive emergencies include acute aortic dissection, acute pulmonary edema,

A hypertensive emergency is defined as having elevated blood pressures (systolic > 180 mm Hg or diastolic > 120 mm Hg) with concomitant end-organ damage. Most commonly, the targeted organs are the brain, heart, aorta, kidneys, or eyes. Examples of hypertensive emergencies include acute aortic dissection, acute pulmonary edema, acute myocardial infarction, acute coronary syndrome, acute kidney injury, severe preeclampsia or eclampsia, hypertensive retinopathy, hypertensive encephalopathy, subarachnoid hemorrhage, intracranial hemorrhage, acute ischemic stroke, or a sympathetic crisis. Acute pulmonary edema secondary to a hypertensive emergency commonly presents with respiratory distress, severe hypertension, and rales auscultated in all lung fields. Tachycardia, accessory muscle use, and other signs of severe distress will also likely be present. In patients presenting with an acute hypertensive emergency with acute pulmonary edema, the mainstay of therapy is vasodilators, predominantly nitrates. Nitrates reduce blood pressure, decrease myocardial oxygen consumption, and improve coronary blood flow. A reduction in blood pressure by 25% is recommended in the first hour of treatment.

Complications of MI

Arrhythmias (eg, ventricular fibrillation), ventricular aneurysm/rupture, cardiogenic shock, papillary muscle dysfunction, heart failure, left ventricular wall rupture. Dresser syndrome: post-MI pericarditis + fever + pulmonary infiltrates.

For each of the following cardiovascular conditions, students should describe the underlying pathophysiology, presenting signs and symptoms, basic epidemiology, modifiable (when applicable) and non-modifiable risk factors, differential diagnosis, appropriate diagnostic studies, clinical intervention, treatment guidelines, pharmaceutical therapies, and health maintenance concerns: Valvular disease

MURMUR ACCENTUATION MANEUVERS POSITION •AORTIC: SITTING UP & LEANING FORWARD ACCENTUATES AORTIC MURMURS (AS, AR). •MITRAL: LYING ON LEFT SIDE ACCENTUATES MITRAL MURMURS (MS, MR). INCREASED VENOUS RETURN •increased venous return INCREASES ALL MURMURS/opening Snap (left & right) - squatting, leg raise, lying down •Exceptions: decreased murmur of hypertrophic cardiomyopathy & delayed ejection click (decreased prolapse/shorter murmur duration) of mitral valve prolapse (MVP) DECREASED VENOUS RETURN •decreased venous return (Valsalva/standing) DECREASES ALL MURMURS/Opening Snap (left & right side). •Exceptions: murmur of hypertrophic cardiomyopathy & earlier ejection click (increased prolapse & longer murmur duration) of MVP. INSPIRATION: •Inspiration increases venous return on right side: increases ALL murmurs/opening snap on the R side (decreased ejection click R side) •Right-sided murmurs best heard with inspiration. •Inspiration decreases venous return on left side: •decreases ALL murmurs/opening snap on the L side (earlier ejection click on the L side) EXPIRATION Expiration increases venous return on left side: •increases ALL murmurs/opening snap on the L side (delayed ejection click on the L side) O Left-sided murmurs best heard after maximal expiration. Expiration les venous return on right side: fALL murmurs/opening snap on the R side (earlier ejection click R side). HANDGRIP 1'es afterload (by compressing the arteries of the upper extremity) leading to ILV emptying (decreased forward flow & increased backward flow). o Outflow murmurs (eg, AS, hypertrophic cardiomyopathy) and MVP | with handgrip (note that handgrip & amyl nitrate are the only maneuvers that affect hypertrophic cardiomyopathy & AS in the same direction - because both maneuvers affect AFTERLOAD & FORWARD FLOW). Because handgrip increases afterload, the increased afterload prevents blood from being ejected from the ventricles, lessening the blood flowing through the stenotic aortic valve and less blood ejected in hypertrophic cardiomyopathy. -egurgitant murmurs (AR, MR) increased with handgrip backward flow; MS 1'es due to increased afterload. AMYL NITRATE J'es afterload (direct arteriolar vasodilator) leading to 1LV emptying (increases forward flow & decreases backward flow of blood). O AS, MVP, hypertrophic cardiomyopathy murmur 1 with amyl nitrate o Regurgitant murmurs (AR, MR) decrease with amyl nitrate This is why afterload reducers like ACEl are used in the management of AR, MR. Aortic stenosis (AS) VS. Hypertrophic obstructive cardiomyopathy (HOCM) • Both: angina, syncope, systolic murmur. Both murmurs go in the same direction with afterload maneuvers (eg, both increase with Amyl nitrate & both decrease with handgrip). • HOCM: preload maneuvers that decrease LV volume (eg, Valsalva, standing) will worsen the murmur of HOCM whereas these maneuvers will decrease the intensity of most other murmurs (including AS). Increased LV volume (eg, squatting, leg raise) will decrease the murmur of HO whereas these maneuvers will increase the intensity AS. No carotid radiation.

Abdominal aortic aneurysm (AAA) is a localized dilation of the aorta > 3 cm, typically related to weakness in the arterial wall. Risk factors for AAA are vascular risk factors such as age > 65, hypertension, smoking, and peripheral vascular disease. AAA may also occur as a result of infection, connective tissue disease, and arteritis. The classic symptoms of rupture are abdominal, flank, or back pain, hypotension, and a pulsatile abdominal mass. Hemorrhage may also lead to syncope. A ruptured AAA is a true emergency, and patients require aggressive management with large bore IV access, administration of blood products, and emergent open surgical or endovascular repair.

Asymptomatic AAAs are managed based on their size and risk of rupture.

For each of the following cardiovascular conditions, students should describe the underlying pathophysiology, presenting signs and symptoms, basic epidemiology, modifiable (when applicable) and non-modifiable risk factors, differential diagnosis, appropriate diagnostic studies, clinical intervention, treatment guidelines, pharmaceutical therapies, and health maintenance concerns: Cardiac Arrest

DETERMINANTS OF CARDIAC ARREST SURVIVAL The two most important factors for survival in the undifferentiated adult cardiac arrest victim are minimizing the elapsed time from patient collapse and the onset of effective cardiopulmonary resuscitation (CPR) and rapid defibrillation. Unfortunately, as many as 85% of arrests occur at home and not in a public place where there may be access to a defibrillator. Studies with patients in VF have demonstrated that for every minute that passes without intervention, the odds of survival decreases by 7-10% (decreased to 3-4% with effective CPR). Other factors related to a positive outcome include witnessed cardiac arrest and early use of advanced life support (ALS). Factors associated with poor prognosis include dyspnea as the presenting complaint, malignancy or sepsis as the underlying cause of cardiac arrest, coexistence of pneumonia, prolonged anoxia, presence of hypotension prior to cardiac arrest, and increasing age. A patient who is not successfully resuscitated in the field is unlikely to be resuscitated in the emergency department. The risks of transporting a patient who remains in cardiac arrest after ALS procedures have been performed in the field may outweigh the likelihood of a successful resuscitation with good neurologic outcome. Medical directors should consider protocols to determine death and termination of resuscitative efforts in the field. THE TEAM APPROACH TO CARDIAC ARREST A resuscitation team has many pivotal participants in a cardiac arrest: the first responder, emergency medical service (EMS) personnel, the emergency department resuscitation team leader, emergency department resuscitation team members, and ancillary personnel. A successful resuscitation depends on a functional team. The responsibilities and expectations for each member must be identified prior to a resuscitation. Attempting to assign these roles after a resuscitation has already begun and will simply add confusion to an already chaotic situation. The composition of the resuscitation team is based largely on resource availability and institutional preferences. There must be a team leader who is responsible for the overall direction of the resuscitation. One member of the team is responsible for airway management. Another member will be necessary to provide support for this function, such as suctioning, or assisting in bag ventilation. Multiple team members may be required to provide chest compressions. To ensure adequate quality of chest compressions, the team leader must ensure that these providers do not become fatigued by rotating the team members every 2 minutes. Vascular access must be obtained, and the team member performing this role can also administer medications. If a medication dispenser (eg, Pyxis) is being used, it is helpful to designate one person to manage it. Finally, it is helpful to have a recorder to record events, provide cues, track laboratory results, and communicate with areas outside of the resuscitation area. It is also important that the team leader control the number of people in the resuscitation area. Those individuals who are not part of the resuscitation may distract other members of the team. Care providers are often trained as individuals, even though they most often respond as an integrated team. Utilization of team behaviors can minimize the number of errors, duplication of efforts, and loss of information that can occur during a resuscitation. Maintaining situational awareness is the key to being an essential team member. Reinforcing situational awareness in others emphasizes the team construct. Some mechanisms to enhance cross-monitoring include call outs and check backs. Call outs involve information that must be shared with the entire team (eg, calling out vital signs). Check backs are means to avoid order errors—the recipient of the order repeats the order verbatim, and then the orderer confirms that order verbatim. These techniques sustain dynamic situational awareness and allow for an organized, integrated team approach to comprehensive resuscitative care. Many universities and other institutions now have simulation centers that can be utilized to more effectively train health care providers for resuscitations in realistic settings, both as individuals and in team settings. FAMILY PRESENCE DURING RESUSCITATION To allow family members to be present during a resuscitation or invasive procedure has been a subject of great debate. The majority of family members who have been surveyed indicate they would like to be present at the resuscitation. This is true for both adult and pediatric patient populations. The families often state that their presence provides emotional and spiritual support. The family's presence may also provide closure and facilitate grieving. Opponents of family presence argue that there may be disruption of code team functions and the event may cause lasting psychological damage to the witnessing family members. Neither of these concerns has been supported by the literature.

Cocaine induced myocardial ischemia or infarction

PATHOPHYSIOLOGY: •Coronary artery vasospasm due to cocaine's activation of the sympathetic nervous system & alpha-1 receptors -> vasoconstriction of the coronary arteries. Ischemia is also caused by a prothrombotic state in the coronary circulation, and increases in heart rate and blood pressure, which increase myocardial demand for oxygen consumption. •MI may occur if vasoconstriction is prolonged (due to decreased blood flow). CLINICAL MANIFESTATIONS •Chest pain or discomfort: Cocaine-induced myocardial ischemia symptoms are indistinguishable from other causes of myocardial ischemia. •Associated symptoms: The chest pain is often accompanied by anxiety, dyspnea, palpitations, and nausea. •Focal neurological & extremity symptoms may suggest, vascular complications (eg, Aortic dissection). PHYSICAL EXAMINATION •Vital signs: Hypertension and tachycardia almost universal; hyperthermia may occur •CNS: Agitation common; focal signs suggest cerebrovascular accident •Pupils: Mydriasis common. •Lungs; Decreased breath sounds after smoking crack suggest Pneumothorax. •Extremities; Decreased pulses suggest vascular complications (eg, Aortic dissection). DIAGNOSIS •ECG: transient ST elevations classic. May induce Myocardial infarction if prolonged constriction. •Laboratory evaluation; fingerstick glucose, Acetaminophen and salicylate levels, and urine pregnancy test in women of childbearing age. Urine toxicology screening & cardiac enzymes. MANAGEMENT Cocaine-associated Myocardial ischemia: •Calcium channel blockers & nitrates drugs of choice to reverse the vasospasm. •Often treated with Aspirin, Heparin, & Benzodiazepines until atherosclerotic disease is ruled out. •Aspirin 325 mg oral (nonenteric coated) should be given to the patient to chew and swallow (assuming aortic dissection not suspected). •Nitroglycerin 0.4 mg sublingual up to 3 doses every 5 minutes, with or without continuous infusion. •Benzodiazepine: Administer Diazepam (5 mg IV) or lorazepam (1 mg IV) for agitation or hypertension; may repeat as necessary. •Phentolamine 1 to 5 mg IV, repeat as necessary; hold for SBP <100. •Avoid nonselective Beta-blockers in cocaine-induced MI - increased risk of vasospasm (unopposed alpha-1 mediated constriction). If Beta blockers are to be used, mixed alpha/beta blockers (eg, Labetalol and Carvedilol) should be used rather than nonselective beta blockers. Psychomotor agitation: •Administer Benzodiazepines (eg, Diazepam 5 to 10 mg IV every 3 to 5 minutes until agitation controlled). STEMI & recent cocaine use •In addition to above, patients with STEMI and recent cocaine use should proceed to reperfusion via either (1) coronary angiography and primary percutaneous coronary intervention (if indicated. usually preferred over fibrinolysis) or (2) fibrinolytic therapy is an alternative when timely coronary angiography is not feasible,

Early repolarization

Usually a normal variant. May be seen in thin, healthy males; African-American males. ST elevation >2mm CONCAVE diffuse leads c large T waves (esp precordial) Tall QRS voltage P& Fishhook (slurring/notching at 1 point) EARLY REPOLARIZATION ABNORMALITIES • Diffuse CONCAVE ST elevations >2 mm with large T waves (especially precordial). Tall QRS voltage. Fishhook (slurring/notching) at the J point.

Ventricular fibrillation

•A type of sudden cardiac arrest (SCA) or sudden cardiac death (SD) with ineffective ventricular contraction. Without treatment, the chances of survival decline by ~2-10% per minute. For those who survive, anoxic brain damage and neurological deficits are common. ETIOLOGIES: •Ischemic heart disease most common - 65 - 70% of all SCDs are attributable to Coronary heart disease (CHD). Among patients hospitalized with acute MI, 5% - 10% have VF or VT, and another 5% will have VF or VT within 48 hours of admission. Heart failure. •Structural heart defects, Cardiomyopathies, Brugada syndrome. Congenital QT abnormalities •Sustained Ventricular tachycardia (VT) may transition to VF. Hypothermia. •Electrolyte abnormalities: Hypokalemia, Hyperkalemia, Hypomagnesemia; Acidosis, hypoxia. CLINICAL MANIFESTATIONS: •Sudden collapse with the patient becoming unconscious, unresponsive, and without palpable pulse (pulseless) due to low cardiac output as a result of ineffective ventricular contraction. •Patients may demonstrate signs of acute MI before the event (eg, chest pain or discomfort, shortness of breath, diaphoresis, nausea, and vomiting). DIAGNOSIS: •Electrocardiogram: Disorganized high frequency undulations with erratic pattern of electrical impulses, fibrillation waves of varying amplitude, shape, and periodicity, occurring at a rate above 320/minute with no identifiable P waves, ORS complexes, or T waves. MANAGEMENT: •Early Unsynchronized cardioversion (Defibrillation) + prompt, high-quality Cardiopulmonary resuscitation (CPR) instituted while awaiting arrival of a defibrillator. •Prompt defibrillation is essential once a defibrillator arrives. The chest compressions should cease, and the rhythm analyzed as soon as the defibrillator or AED becomes available. The CPR should be interrupted for the application of paddles and delivery of the shock. •If the rhythm is shockable, defibrillation at 120 to 200 joules on a biphasic defibrillator or 360 joules using a monophasic is recommended; otherwise, CPR should be continued. •CPR should be resumed immediately after defibrillation, and rhythm analysis should be conducted after 5 cycles of chest compressions or 2 minutes. •Pharmacological intervention should not delay or replace prompt and early defibrillation in shockable cardiac rhythms. •Administer Epinephrine and Amiodarone as per ACLS protocol in patients sustaining VF rhythm regardless of receiving 3 shocks. Amiodarone significantly improves survival to hospital admission without affecting survival to hospital discharge. PREVENTATIVE MANAGEMENT: •Secondary prevention: Implantable cardioverter-defibrillator (ICD) placement is indicated for secondary prevention of SCD in patients with prior episodes of VF and sustained VT. ICDS improve long-term survival status-post VF when compared to patients receiving only medications. •Primary prevention: ICD placement is recommended for primary prevention of SCD in patients at increased risk of life-threatening VF/VT (eg, left ventricular ejection fraction <35%).

Pulseless electrical activity and asystole

•Pulseless electrical activity (PEA): the presence of coordinated (organized) rhythms on ECG without sufficient mechanical cardiac contraction to produce a palpable pulse or measurable blood pressure (electrical activity is not coupled with sufficient mechanical cardiac contraction). •Asystole complete absence of demonstrable electrical and mechanical cardiac activity. ETIOLOGIES: •6 "H"S: Hypovolemia, Hypoxia, Hydrogen ions (Acidosis), Hyper/hypokalemia, Hypothermia •6 T"s: "Toxins, Cardiac Tamponade, Tension Pneumothorax, Thrombosis (MI, PE), Tachycardia, Trauma (hypovolemia). •Narrow-complex PEA may be the result of mechanical problems - eg, cardiac tamponade, pneumothorax, mechanical hyperinflation, pulmonary embolism, or myocardial rupture. •Wide-complex PEA may be the result from a metabolic problem - eg, hyperkalemia, drug toxicity (eg, sodium channel-blocker toxicity), cardiac ischemia (acute MI with pump failure), or left ventricular failure and should be treated as appropriate. MANAGEMENT: •CPR + Epinephrine administered as soon as possible after CPR has been initiated, rapid identification and reversal of underlying causes, and checks for a shockable rhythm every 2 minutes. Epinephrine (1 mg IV every 3 to 5 minutes). Institute the secondary ABCDs. •Vasopressin alone or in combination with Epinephrine is no more effective than Epinephrine alone but Vasopressin may be used to replace the first or second dose of Epinephrine. •Neither systole nor PEA respond to Unsynchronized cardioversion (defibrillation). Narrow-complex PEA: In addition to above, consider mechanical cause: •Cardiac tamponade - treat with bedside Pericardiocentesis. •Tension Pneumothorax - treat with needle decompression. •Pulmonary embolism - treat with thrombolysis (if not contraindicated). •Hypovolemia/bleeding - treat with resuscitation (eg, fluids and blood products). Wide-complex PEA In addition to above, consider metabolic cause: • Hyperkalemia - IV calcium gluconate, Insulin/glucose, sodium bicarbonate bolus. • Sodium channel blocker overdose - treat with IV sodium bicarbonate boluses. • Cardiac ischemia, or left ventricular failure and should be treated as appropriate. • Hvpoxia- - assess for correct placement of the airway adjunct and check breath sounds to rule out slippage of the endotracheal tube or that a tension Pneumothorax is not present. • Hydrogen ion (acidosis) - Respiratory acidosis is addressed by early endotracheal intubation and alveolar ventilation. Metabolic acidosis can be somewhat addressed by good-quality CPR. Continuous endovenous hemodialysis may be required for severe acidosis Sodium bicarbonate may be needed for severe metabolic acidosis (not used routinely). • Hypothermia - treat with gradual rewarming with blankets and warm IV fluids. If there is no recovery of consciousness in the hypothermic patient following ROSC, maintain a body core temperature of 33°C until further assessment and decisions can be made. • Beta-blocker or calcium channel-blocker overdose may require high-dose Insulin therapy and lipid emulsion infusion. PROGNOSIS: •Survival is low if patients in systole do not achieve return of spontaneous circulation (ROSC) in the field or convert to a shockable rhythm.

Acute coronary syndrome

•Symptoms of acute myocardial ischemia secondary to acute plaque rupture & varying degrees of coronary artery thrombosis (occlusion). Angina that is new in onset, crescendo, or at rest (usually >30 minutes). >90% occlusion can cause symptoms @ rest ETIOLOGIES •Atherosclerosis: most common cause of MI. Plaque rupture - acute coronary artery thrombosis with platelet adhesion/activation/aggregation along with fibrin formation. Vasculitis, embolism. •Coronary artery vasospasm (2%): cocaine-induced, variant (Prinzmetal) angina. CLINICAL MANIFESTATIONS •Chest pain: retrosternal pressure not relieved with rest or nitroglycerin, pain at rest, often lasting > 30 minutes, may radiate to the lower jaw & teeth, left arm, epigastrium, back or shoulders or change from typical pattern. Pain at rest usually indicates >90% occlusion. •Sympathetic stimulation - anxiety, diaphoresis, tachycardia, palpitations, nausea, vomiting, dizziness. •Silent MI: ~25% are atypical/silent: eg, women, elderly, diabetics & obese patients. Atypical symptoms include abdominal pain, jaw pain, or dyspnea without chest pain. PHYSICAL EXAMINATION: •Usually normal. Patients may be tachycardic. •Inferior wall MI: may be associated with bradycardia or heart blocks (the RCA supplies the AV node in 90%). May have S4 (especially with inferior MI). Triad of right ventricular infarction: increased JVP + clear lungs + positive Kussmaul sign. DIAGNOSTIC STUDIES: •12 lead ECG •Cardiac enzymes •STEMI: ST elevations >1mm in >2 anatomically contiguous leads with reciprocal changes in the opposite leads. ECG progression: Hyperacute T waves first change -> ST elevations -> Q waves A new Left bundle branch considered an STEMI equivalent ^Troponin may be falsely elevated in patients with renal failure, advanced heart failure, acute PE, CVA.

Heart failure

• Inability of the heart to pump sufficient blood to meet the metabolic demands of the body at normal filling pressures. TYPES • Left-sided: most common cause is Coronary artery disease. Hypertension (increased afterload), valvular disease [Aortic stenosis causes pressure overload (increased, afterload) and regurgitant lesions cause volume overload], Cardiomyopathies. • Right-sided; most common cause is left-sided Heart failure. Pulmonary disease (eg, COPD, Pulmonary hypertension), Mitral stenosis. • HE with reduced ejection fraction (HFrEF) most common type of HF (systolic dysfunction). Post myocardial infarction most common cause, myocarditis, dilated cardiomyopathy. Associated with decreased ejection fraction and S3 gallop. • Heart failure with preserved ejection fraction (HFpEF) characterized by diastolic dysfunction & a normal or increased ejection fraction. Causes include longstanding Hypertension (eg, in the elderly), left ventricular hypertrophy, & valvular heart disease. PATHOPHYSIOLOGY • Initial injury: An initial injury to the heart leads to (1) pathologic increase in preload, (2) increase in afterload, and/or (3) decrease in contractility, subsequently decreasing cardiac output. • Compensatory responses: After a Myocardial infarction and other pathological insults, compensatory responses, such as activation of the Renin angiotensin aldosterone system, increased sympathetic activity, and ventricular remodeling confer short-term benefit. • Decompensation: However, left unchecked, these compensatory responses are maladaptive, leading to sodium, water, and volume overload (continued RAAS activation), and pathological ventricular remodeling (dilatation of the ventricular walls and systolic dysfunction). Lower cardiac output results in reduction of renal blood flow and glomerular filtration rate, which leads to further sodium and fluid retention. CLINICAL MANIFESTATIONS Left-sided Heart failure: • Pulmonary symptoms: increased pulmonary venous pressure from fluid backing up into the lungs (Think L for Lungs and L-sided). • Dyspnea: progressive dyspnea most common symptom - includes exertional dyspnea, which may progress to orthopnea (on lying flat), paroxysmal nocturnal dyspnea, or dyspnea at rest. May have exercise intolerance or fatigue. Fluid retention may contribute to the symptoms. • Cough: chronic, nonproductive cough or productive with pink, frothy sputum, worse in the supine position or at night. Rales may be heard on examination. Right-sided Heart failure; • Systemic symptoms of peripheral and abdominal congestion due to increased systemic venous pressure from fluid backing up into the "roads" [R for R-sided and 3 Roads to the heart = inferior vena cava (IVC), Superior vena cava (SVC), & hepatic circulation]. • Peripheral edema: pitting edema of the legs, cyanosis. May have increased body weight. • Jugular venous distention due to increased jugular venous pressure. • GI & hepatic congestion: anorexia, loss of appetite, nausea, vomiting, hepatojugular reflux (increased VP with liver palpation), hepatosplenomegaly, ascites. Noncardiac symptoms: • Anorexia, nausea, fatigue, weakness, nocturia, and memory impairment PHYSICAL EXAMINATION: Signs are due to congestion (eg, dyspnea, fatigue, and fluid retention): Lungs: • Crackles (rales) due to fluid in the alveoli; rhonchi, or expiratory wheezing. • Diminished air entry at the lung bases if a Pleural effusion is present (more common on the right side). • Tachypnea (rapid, shallow breathing). • Advanced disease: Cheyne-Stokes breathing - deeper, faster breathing with gradual decrease & periods of apnea. Cyanosis. Peripheral: • Lower extremity edema (especially in the pretibial region and ankles in ambulatory patients with sacral edema in bedridden patients), elevated jugular venous pressure. • Pulsus alternans. • Dusky pale skin, diaphoresis, and cool lower extremities may be suggestive of worsened cardiac output "cold and wet". • Abdomen: Epigastric tenderness, positive hepatojugular reflex (sustained moderate pressure on the liver may increase jugular venous pressure); tender and enlarged liver. Ascites or hepatomegaly may be seen. Cardiac examination: • S3 gallop: advanced systolic dysfunction associated with a third heart sound (S3 gallop) and a laterally displaced apex beat. • A murmur of Mitral regurgitation is often audible when the left ventricle (LV) is markedly enlarged, or a Tricuspid regurgitation murmur is present when the right ventricle is volume or pressure overloaded. • S4 gallop: may be seen with diastolic dysfunction. DIAGNOSIS: Echocardiogram: • Indications: diagnostic test of choice in an outpatient setting to make the diagnosis of Heart failure. Doppler echocardiography is helpful in the diagnosis and classification of HF, enables evaluation of ventricular size, global and regional systolic function, diastolic function, valvular disease, & pericardial disease. It measures ejection fraction, assesses ventricular function, and may reveal the cause (ejection fraction most important determinant of prognosis). • Heart failure with reduced ejection fraction: (HFrEF) - decreased ejection fraction, thin ventricular walls, dilated LV chamber cavity due to systolic dysfunction. • Heart failure with preserved ejection fraction: (HFpEF) _ normal or increased ejection fraction, thick ventricular walls, small LV chamber cavity due to diastolic dysfunction. Electrocardiogram(ECG): • Assesses for evidence of ACS (myocardial ischemia or MI) and dysrhythmias (eg, atrial fibrillation). • The ECG may also identify other predisposing or precipitating conditions for HF such (eg, ventricular hypertrophy or left atrial abnormalities). ECG findings often nonspecific. Labs: • Serum electrolytes, renal tests (eg, BUN, creatinine, urinalysis), CBC, lipid levels, LFTs, TSH, cardiac markers if there is a concern for cardiac injury, NEW YORK HEART ASSOCIATION FUNCTIONAL CLASS Class | - No symptoms, no limitation during ordinary physical activity. Class Il - Mild symptoms (dyspnea &/or angina), slight limitation during ordinary activity. Class III -Symptoms cause marked limitation in activity (even with minimal exertion) comfortable only at rest. Class IV - Symptoms even while at rest, severe limitations, & inability to carry out physical activity. LIFESTYLE MODIFICATION • Self-management: includes daily monitoring of signs and symptoms (including daily weight monitoring to detect fluid accumulation) and lifestyle modifications. • Sodium restriction: 3g/day with avoidance of excessive intake >6g/day. • Fluid restriction: 1.5-2 L/day only in patients with refractory (stage D, class IV) HF or symptomatic or severe hyponatremia (serum sodium <120 mEq/L). • Smoking cessation, alcohol abstinence or restriction. Weight management avoidance of obesity. MANAGEMENT OF HErEF Long-term initial management: • A combination of a diuretic and a renin-angiotensin system inhibitor should be the initial treatment in most symptomatic patients with Heart failure and reduced LVEF, with the early addition of a beta-blocker or triple combination therapy initially. • Initial triple combination therapy: Initial pharmacologic therapy of HFrEF includes a combination of (1) a renin-angiotensin system inhibitor (ARNI, ACE inhibitor, or single agent ARB), (2) diuretic therapy (as need to treat volume overload), and (3) a Beta blocker. • Angiotensin system blocker - Angiotensin receptor-neprilysin inhibitor [ARNI] or Angiotensin converting enzyme [ACE] inhibitor, or single agent angiotensin receptor blocker [ARB]. • Diuretic therapy for volume overload & symptomatic relief (Thiazide diuretics, Loop diuretics). • Beta blocker often added soon after Angiotensin system blockers to reduce mortality (eg, Metoprolol, Carvedilol, Bisoprolol). • If a patient is intolerant of all 3 of the angiotensin system blockers, then Hydralazine plus Nitrate therapy is used as an alternative for initial therapy (used along with Beta blocker therapy). SECONDARY THERAPY • Secondary pharmacologic agents indicated in addition to continued initial pharmacologic therapy. For most patients with HFrEF, secondary pharmacologic therapy consists of a mineralocorticoid receptor antagonist and a sodium-glucose co-transporter 2 inhibitor. • Mineralocorticoid receptor antagonist (MRA), Spironolactone & Eplerenone, reduce mortality (due to aldosterone antagonism) when added to optimal initial pharmacologic therapy. • SGLT2 inhibitors substantially reduce the risk of cardiovascular death & hospitalization for HFrEF, with or without diabetes (Dapagliflozin, Empagliflozin). May be part of initial therapy. • Hydralazine plus Nitrate is an alternative or additional add-on therapy for patients with persistent symptoms on initial therapy plus MRA (if indicated). Improved outcomes in African-Americans. OTHER THERAPIES: • An implantable cardioverter-defibrillator (ICD) for primary or secondary prevention of SCD in patients with an ejection fraction (EF) ≤35% and clinical heart failure is well established (an arrhythmia can be lethal in these patients). • Biventricular pacing (resynchronization) if EF 35% or less, & QRS duration of 120 msec or more, in addition to optimal medical therapy, reduces mortality & hospitalization from any cause by ~20%. THERAPIES THAT SHOULD BE AVOIDED: • Thiazolidinediones (glitazones) can worsen or precipitate Heart failure. • Most Calcium channel blockers (with the exception of Amlodipine and Felodipine) because there is no mortality benefit of non-Dihydropyridine Calcium channel blockers with negative inotropy (Diltiazem, Verapamil) in the management of HFrEF and there may be a possible deleterious effect of the use of non-Dihydropyridines. Amlodipine appears to be safe in HF and may be used if a CCB is necessary for a concomitant disease, such as Hypertension or Angina. • NSAIDs: Nonsteroidal anti-inflammatory medications, and cyclooxygenase-2 inhibitors can lead to sodium and water retention as well as renal impairment. • Combination of an ACE inhibitors, ARB, and aldosterone blocker - increases the risk of hyperkalemia. SACUBITRIL-VALSARTAN Mechanism of action: • Sacubitril is an angiotensin receptor neprilysin inhibitor (ARNI), increasing levels of natriuretic peptides. Valsartan is an angiotensin Il receptor blocker. Indications: • Reduces mortality & decreases hospitalization for chronic Heart failure (Class II-III) with reduced EF. Associated with lower rates of renal insufficiency, hyperkalemia, & cough compared to ACE inhibitors but higher rates of hypotension and non-serious angioedema. Usage criteria: • For patients with HFrEF with all 3 of the following criteria: (1) hemodynamic stability - systolicblood pressure 100 mmHg or greater for at least 6 hours, no increase in dose of IV diuretics in the preceding 6 hours, and no IV inotropes in the preceding 24 hours; (2) no history of angioedema & (3) the patient has access to medications (eg cost). • In terms of the Angiotensin system blockers, the improvement outcome is strongest for ARNI, intermediate for ACE inhibitors, and weakest for ARBs. Contraindications: • Include pregnancy (risk of fetal toxicity & death), history of angioedema (of any cause), and severe hepatic impairment (Child-Pugh C classification). • Not used concomitantly with ACEs (increased risk of angioedema), Aliskiren in patients with Diabetes mellitus, or another Angiotensin Il receptor blocker (ARB) to avoid dual ARB therapy. MINERALOCORTICOID RECEPTOR ANTAGONIST (MRA) Rationale: • Spironolactone & Eplerenone reduce mortality (due to aldosterone antagonism) when added to optimal initial pharmacologic therapy. Mechanism of action: • Because increased aldosterone in Heart failure mediates some of the major effects of renin- angiotensin-aldosterone system activation, such as myocardial remodeling & fibrosis, as well as sodium retention and potassium loss at the distal tubules, aldosterone (mineralocorticoid) receptor antagonists cause sodium loss (some diuretic effect) while sparing the loss potassium (potassium-sparing diuretics), improving clinical outcomes, including survival. Due to their potassium sparing properties, serum potassium must be <5 mEq/L prior to initiation of MRAs. Indications: • For patients receiving initial pharmacologic therapy for HFrEE, the next step is often to add a mineralocorticoid receptor antagonist (MRA) if the patient has an indication. • May be added for patients with persistent symptoms on optimal initial therapy - For patients with HFrEF who have symptomatic HF (New York Heart Association [NYHA] class II, Ill, or IV) and an LVEF 35% or less. • May be added in patients post-myocardial infarction with LVEF 40% or less - who are already receiving a renin angiotensin system inhibitor and have either symptomatic HF or diabetes mellitus (DM). • The oral potassium-sparing agents are often useful in combination with the loop diuretics and thiazides. SODIUM GLUCOSE TRANSPORT 2 (SGLT2) INHIBITORS • Dapagliflozin, Empagliflozin, Canagliflozin Rationale: • SGLT2 inhibitors substantially reduce the risk of cardiovascular death and hospitalization for Heart failure for patients with reduced EF, with or without diabetes. • SGLT2 inhibitors have an additional benefit of reducing hyperglycemia and reduce the progression of diabetic kidney disease. • SGLT2 inhibitors reduce blood glucose, lower blood pressure, & reduce weight. Indications: • An SGLT2 inhibitor is a treatment option for patients with HFrEF with type 2 DM (Dapagliflozin, Empagliflozin, Canagliflozin) or without type 2 DM (Dapagliflozin or Empagliflozin). May be used as part of initial therapy (quadruple therapy) instead of the classic triple therapy. Contraindications: • SGLT2 inhibitors are contraindicated in patients with type 1 DM, type 2 DM with risk factors for ketoacidosis, or severely impaired or rapidly declining kidney function DIURETICS • Thiazides: when fluid retention is mild, thiazide or related diuretics (eg, Hydrochlorothiazide, Metolazone, Chlorthalidone) may be sufficient and can offer improved control of Hypertension. • Loop diuretics: may be indicated for more severe Heart failure (eg, Furosemide, Bumetanide, Torsemide). • The oral potassium-sparing agents are often useful in combination with the loop diuretics and thiazides. IVABRADINE Mechanism of action: • Selective sinus node inhibitor (slows the sinus rate). Indications: • Associated with hospitalization & mortality. • Used in symptomatic chronic stable heart failure with LVEF $35%, in sinus rhythm with a resting pulse of ≥70 bpm & already maxed out on the beta-blocker dose or are unable to take beta-blockers. IMPORTANT PRINCIPLES IN THE MANAGEMENT OF HEART FAILURE WITH REDUCED EF • ACE inhibitors are the single most effective medications for mortality benefit in Heart failure with reduced ejection fraction (systolic heart failure). However, ARNI is the most effective medication. • Beta-blockers (Carvedilol, Metoprolol, Bisoprolol) are often added to ACE inhibitors for additional mortality benefit. Medications & interventions that decrease mortality: • ACE inhibitors, Beta blockers, Angiotensin receptor blockers, Angiotensin receptor-neprilysin inhibitors, Hydralazine plus Nitrate, and mineralocorticoid receptor antagonists. • Automated Implantable Cardioverter Defibrillator in patients with EF <35% (because these patients tolerate arrhythmias poorly). Medications with no mortality benefit • There is no mortality benefit of non-Dihydropyridine Calcium channel blockers (Verapamil, Diltiazem) in the management of Heart failure with reduced ejection fraction and there may be a possible deleterious effect (they have a greater depressive effect on cardiac conduction and contractility and are somewhat less potent dilators in comparison to the Dihydropyridines). • Digoxin is associated with decreased hospitalization but no mortality benefit

ACLS/BLS protocol

https://cpr.heart.org/en/resuscitation-science/cpr-and-ecc-guidelines/algorithms

Tricuspid stenosis

• Blood backs up into the right atrium -1 right atrial enlargement - right-sided heart failure. PHYSICAL EXAMINATION Mid-diastolic murmur at the left lower sternal border (Xyphoid, 4th intercostal space). Low frequency. • 1 intensity: venous return: (squatting, laying down, leg raising, inspiration). • Opening snap (OS): usually occurs later than the opening snap of mitral stenosis. MANAGEMENT • Medical: decrease right atrial volume overload with diuretics & Na* restriction. • Surgical: commissurotomy or replacement if right heart failure or I cardiac output.

mitral valve prolapse (MVP)

• Mitral valve leaflets extend abnormally above the mitral annulus into the left atrium during systole. • Most common in young women (15 - 35 years of age). Seen in 2-5% of the population. ETIOLOGIES: • Myxomatous degeneration of one or both leaflets of the mitral valve - weakened and elongated chordae tendineae, mitral annular dilatation, or thickened leaflet tissue. • Connective tissue diseases g, Marfan or Ehlers-Danlos syndromes, Osteogenesis imperfecta. CLINICAL MANIFESTATIONS: • Most patients are asymptomatic. MVP most common cause of Mitral regurgitation in the US. • MVP syndrome: combination of symptoms & signs with Mitral valve prolapse - eg, atypical chest pain, palpitations (arrhythmias), exertional dyspnea, exercise intolerance, dizziness, & fatigue. Anxiety, low blood pressure, supraventricular arrhythmias, and syncope suggest autonomic nervous system dysfunction. • Symptoms associated with Mitral regurgitation progression (not common) - dyspnea, fatigue, CHF. • In rare cases, it may present with arrhythmias, sudden cardiac death, Infective endocarditis or stroke. PHYSICAL EXAMINATION: • Mid-late systolic click best heard at the apex due to tensing of the chordae tendineae & mitral valve apparatus as the leaflets prolapse into the left atrium. The click may be followed by the high- pitched mid-late systolic murmur of Mitral regurgitation (the murmur of MR is late systolic early in the disease and becomes holosystolic with severe prolapse). • Earlier click: any maneuver that makes the LV smaller or decreases preload (eg, Valsalva, standing) results in an earlier click & longer murmur duration due to increased prolapse (although the murmur of MR will be fainter). • Delayed click; any maneuver that increases preload (eg, squatting, leg raise, supine) results in a delayed click & shorter murmur duration due to decreased prolapse. Handgrip. • The handgrip maneuver increases the intensity of the Mitral regurgitation associated with MVP and decreases the murmur of Hypertrophic cardiomyopathy. • Skeletal abnormalities: may have narrow anteroposterior diameter, hypotension, scoliosis, and pectus excavatum deformity. Patients tend to have a lower BMI compared to controls. DIAGNOSIS Echocardiography: • Diagnostic test of choice (key imaging study) in the diagnosis of MVP - helps to identify MVP and other associated valvular abnormalities. Two- or three-dimensional echocardiogram allows measurement of leaflet thickness and displacement relative to the annulus. • Findings include displacement of any portion of the mitral leaflets 2 mm or more above the annular plane into the left atrium (systolic billowing) in a long axis view, posterior bulging leaflets with tissue redundancy. MANAGEMENT • Asymptomatic: MVP patients with no symptoms often require no treatment other than reassurance, conservative management, observation, and monitoring. Patients with no concomitant mitral regurgitation can be followed every 3-5 years and patients with Mitral regurgitation can be followed annually. Reassurance is a major component of management of most patients with MVP - MVP is associated with a good prognosis, has a benign nature, and has a low incidence of serious complications (for most patients, it is of no clinical significance). • Autonomic dysfunction; In addition to reassurance and lifestyle changes, Beta-blockers may be used in patients with autonomic dysfunction.

Distinguish among non-ST segment elevation acute myocardial infarction, ST segment elevation acute myocardial infarction, angina pectoris, unstable angina, Prinzmetal/variant angina on an EKG and by clinical presentation.

ANGINA PECTORIS •Chest discomfort due to myocardial ischemia as a result of fixed epicardial coronary artery obstruction (insufficient blood flow to the cardiac muscle as a result of ischemic heart disease). •A complication of Coronary artery disease (CAD) usually due to Atherosclerosis (hardening and narrowing of the coronary arteries), leading to symptoms. RISK FACTORS: •Major: Diabetes mellitus: worst risk factor, considered a CAD equivalent; Smoking most important modifiable risk factor. Hyperlipidemia, Hypertension, male sex, age >45 years in men or >55 years in women, family history of Coronary artery disease. •Minor: obesity, elevated homocysteine, increased C-reactive protein, & lack of estrogen. •Coronary artery disease equivalents include Diabetes mellitus, Carotid artery disease, Abdominal aortic aneurysm, Peripheral arterial disease, or 10-year risk of MI at least 20%. PATHOPHYSIOLOGY: •Myocardial ischemia: inadequate myocardial tissue perfusion due to imbalance between increased myocardial oxygen demand & decreased coronary artery blood supply, leading to chest pain or pressure (or its equivalent). •Increased myocardial oxygen demand: with increased heart rate, systolic blood pressure (afterload), increased myocardial contractility, increased myocardial wall tension or stress. CLINICAL MANIFESTATIONS •Chest pain or discomfort: classic - although there is significant variation, the pain is classically substernal, poorly localized, exertional, short in duration (< 30 minutes but often resolves within 5 minutes of cessation of activity), exacerbated with activity or stress and relieved with rest &/or Nitroglycerin, may radiate to the arm, teeth, lower jaw, back, epigastrium, or shoulders. •Associated symptoms: dyspnea, nausea, diaphoresis, vomiting, numbness, fatigue. •Anginal equivalent: instead of chest pain, patients develop dyspnea, epigastric or shoulder pain. This is especially seen in women, elderly, diabetics, and obese patients. •Physical examination is usually normal. DIAGNOSIS •Stable angina is usually a clinical diagnosis along with testing. •ECG: initial test of choice - ST depression classic finding, T wave inversions, poor R wave progression, T wave pseudonormalization. The resting ECG is normal in 50% of cases. •Stress testing: most important noninvasive testing. Options include stress ECG, myocardial perfusion imaging, or stress echocardiography. •Coronary angiography: definitive diagnostic test. Defines location and extent of CAD. MANAGEMENT •Typical outpatient regimen includes 4 drugs: daily Aspirin + Beta blockers (both decrease mortality), sublingual short-acting Nitroglycerin as needed for immediate symptom relief, and a daily Statin. •Calcium channel blockers & long-acting Nitrates can be used in lieu of Beta blockers if Beta blockers are contraindicated or have adverse effects, or in Vasospastic disorders (eg, Prinzmetal). •Reduction of risk factors: hypertension and DM control, exercise, diet, smoking cessation. Revascularization: definitive management •Percutaneous transluminal coronary angioplasty - 1 or 2 vessel disease in nondiabetics not involving the left main coronary artery, with normal or near-normal ejection fraction. •Coronary artery bypass graft - left main coronary artery stenosis, 3 vessel disease (2 vessel disease in diabetics), or decreased left ventricular ejection fraction <40% MEDICATIONS USED IN THE MANAGEMENT OF ANGINA PECTORIS •Typical outpatient regimen includes 4 drugs: daily Aspirin + Beta blockers (both decrease and a daily Statin. mortality), sublingual short-acting Nitroglycerin as needed for immediate symptom relief, •Calcium channel blockers can be used in lieu of Beta blockers if Beta blockers are contraindicated or in Vasospastic disorders (eg, Prinzmetal). Long-acting Nitrates are also an alternative. BETA BLOCKERS Indication: First-line long-term therapy to reduce anginal episodes & improve exercise tolerance. Benefits: •Reduce mortality, prevent ischemic occurrences, and improve symptoms by reducing myocardial oxygen demand via decrease in heart rate and myocardial contractility. Mechanism of action: •Increase myocardial blood supply - increase coronary artery filling time (coronary arteries fill during diastole), improving coronary circulation. •Decrease myocardial oxygen demand - reduce myocardial 02 requirements during stress & exercise (negative chronotropes & inotropes that decrease heart rate and contractility), as well as decrease blood pressure. •Because Beta blockers reduce the heart rate-blood pressure product during exercise, the onset of angina or the ischemic threshold during exercise is delayed or prevented. Contraindications: •Vasospastic or variant (Prinzmetal) angina - Beta blockers may increase the tendency to induce coronary vasospasm from unopposed alpha-receptor activity (vasoconstriction). SHORT-ACTING NITROGLYCERIN Indications: •First-line therapy for immediate relief of acute anginal symptoms (no mortality benefit). Short-acting Nitrates in the sublingual form most commonly used. Mechanism of action: •Increases myocardial blood supply - increases coronary artery blood flow & collateral circulation as well as reduces coronary artery vasospasm. •Decreases myocardial oxygen demand venodilation (decreases preload) & vasodilation of arteries (decreases afterload). Administration: •Administered sublingual if chest pain occurs. Given up to 3 doses 5 minutes apart. •Can be used prophylactically 5 minutes before an activity likely to cause ischemia. Adverse effects: •Vasodilation: headache, flushing of the skin, hypotension, peripheral edema. •Tolerance: tachyphylaxis after 24 hours (allow for nitrate-free period for 8 hours). •Deteriorates with exposure to light, moisture, and air. Contraindications: •Systolic blood pressure < 90 mmHg •Right ventricular infarction •Use of phosphodiesterase-5 inhibitors (eg, Sildenafil). Phosphodiesterase-5 inhibitors when coadministered with Nitrates can cause significant hypotension. ASPIRIN Indications: •In the absence of a contraindication, all patients with established CVD should be treated with Aspirin (81-325 mg orally daily). Benefits: •Reduces in the risk of subsequent myocardial infarction (MI), stroke, and vascular death among a wide range of patients who have survived an occlusive cardiovascular disease (CVD) event. Considerations: •Risk stratification is important as the use of Aspirin comes with an increased risk of major bleeding, Adverse effects: •Increased risk of major bleeding - extracranial (upper Gl bleeding most common) and intracranial hemorrhage. •In patients who are unable to take Aspirin, or those with a history of gastrointestinal bleeding, Clopidogrel is a reasonable alternative. •The most common adverse effect from therapeutic Anti-inflammatory doses of Aspirin is gastric upset due to COX1 inhibition. Chronic use can result in gastric ulceration. •Renal injury including acute failure and interstitial nephritis. •Aspirin hypersensitivity: When thromboxane and prostaglandin synthesis is inhibited by even small doses of Aspirin, persons with Aspirin hypersensitivity (especially associated with nasal polyps) can experience Asthma from the increased synthesis of leukotrienes. •At higher doses of Aspirin, tinnitus, vertigo, hyperventilation, and respiratory alkalosis may occur. At very high doses, the drug causes metabolic acidosis, dehydration, hyperthermia, collapse, coma, and death. •Children with viral infections who are treated with Aspirin have an increased risk for developing Reye syndrome, a rare but serious syndrome of rapid liver degeneration and encephalopathy. CALCIUM CHANNEL BLOCKERS •LONG-acting Non-dihydropyridines: Diltiazem or Verapamil •LONG-acting Dihydropyridines: Amlodipine or Felodipine. Nifedipine, Nicardipine. Indications: •Calcium channel blockers and long-acting Nitrates are alternatives if Beta blockers are contraindicated or cause adverse effects. •They can also be added as combination therapy if monotherapy is unsuccessful. •Long-acting Diltiazem or Verapamil or a second-generation Dihydropyridine (Amlodipine or Felodipine) are preferred. Caution: •Short-acting dihydropyridines, especially Nifedipine, should be avoided unless used in conjunction with a Beta blocker in the management of CCS because of evidence of an increase in mortality after a myocardial infarction and an increase in acute myocardial infarction in hypertensive patients •Unlike the Beta-blockers, Calcium channel blockers have not been shown to reduce mortality postinfarction and in some cases have increased ischemia and mortality rates. This appears to be the case with some short-acting dihydropyridines (eg, Nifedipine) and with Diltiazem and Verapamil in patients with clinical heart failure or moderate-severe LV dysfunction. LONG-ACTING NITROGLYCERIN Administration: •Oral Isosorbide dinitrate, isosorbide mononitrate, Nitroglycerin ointment, 2% ointment, and transdermal nitroglycerin patches. •Patches should be taken off after 12-14 hours of use for a 10-12-hour patch-free interval daily, Indications: •Calcium channel blockers and long-acting Nitrates are alternatives if Beta blockers are contraindicated or cause adverse effects. •They can also be added as combination therapy if monotherapy is not successful. Adverse effects: •Tachyphylaxis The main limitation to long-term Nitrate therapy is tolerance, which can be limited by a regimen that includes a minimum 8- to 10- hour period per day without Nitrates (overnight). •Vasodilation: headache (Nitrate therapy is often limited by headache), flushing, hypotension, light-headedness, peripheral edema. Contraindications: •Phosphodiesterase inhibitors used commonly for erectile dysfunction should not be taken within 24 hours of Nitrate use. RANOLAZINE Mechanism of action: •Late sodium channel blocker; Reduces oxygen requirements of cardiac muscle (reduced tension in the heart wall) by reducing intracellular calcium overload and the subsequent increase in diastolic tension via its inhibition of late inward sodium channel in myocardial diseased states (eg, ischemia and hypertrophy). •It does not exert a significant effect on the normal myocardium at usual dosages. Indications: •Ranolazine is effective at reducing anginal symptoms and improving exercise capacity when added to conventional medical therapy, especially in patients refractory to conventional therapy. •Ranolazine has no effect on heart rate and BP, and it has been shown in clinical trials to prolong exercise duration and time to angina, both as monotherapy and when administered with conventional antianginal therapy. •It is safe to use with erectile dysfunction medications. •It also decreases occurrence of atrial fibrillation and results in a small decrease in HbA1c. In spite of the QT prolongation, there is a significantly lower rate of ventricular arrhythmias with its use following acute coronary syndromes. •Ranolazine is not to be used for treatment of acute anginal episodes. Contraindications: •Because Ranolazine can cause QT prolongation, it is contraindicated in patients with existing QT prolongation; in patients taking QT prolonging medications, such as class I or III antiarrhythmics (eg, Quinidine, Dofetilide, Sotalol); and in those taking potent and moderate CYP450 3A inhibitors (eg, Clarithromycin and Rifampin). •Significant liver and kidney disease.

Unstable angina

•A type of ACS associated with critical coronary artery stenosis without myocardial cell death. •UA is characterized by: (1) ischemic symptoms suggestive of Acute coronary syndrome, (2) negative cardiac biomarkers Troponin and CK-MB, and (3) with or without ECG changes indicative of ischemia (eg, ST segment depression or new T wave inversion). PATHOPHYSIOLOGY: •Plaque rupture: The most common cause is plaque rupture of a previous nonsevere lesion with subsequent thrombus formation, leading to critical coronary artery stenosis that is not fully occlusive (so no myocardial cell death). •Unstable plaques (eg, thin fibrous cap, large lipid core, increased macrophages) are more prone to rupture than more stable plaques (eg, thick fibrous cap, small lipid core, low amounts of macrophages). •Coronary artery vasospasm: Less common cause [eg, Variant (Prinzmetal) angina or Cocaine use. CLINICAL MANIFESTATIONS: •Angina is considered unstable if it presents in any of the following 3 ways: (1) Rest Angina, generally lasting longer >20-30 minutes; (2) New-onset Angina, especially if it significantly limits physical activity, and (3) Change in Angina pattern: - increasing Angina that is more frequent, lasts longer, or occurs with less exertion than previous Angina. •Chest discomfort: Chest discomfort (eg, pain, pressure, tightness, constriction, heaviness, or burning) in the center or the left of the chest is classically substernal, diffuse and poorly localized, exertional, not relieved with rest &/or nitroglycerin, pain at rest, > 30 minutes, or change from typical pattern (eg, longer to resolve, increase in severity). •May radiate to the lower jaw & teeth, left arm, epigastric area, shoulders. DIAGNOSIS: •Electrocardiogram: with or without ECG changes indicative of ischemia - eg, ST segment depression, new deep T wave inversions or flattening, poor R wave progression, pseudonormalization of the T wave, or hyperacute T-waves. •Cardiac enzymes: Negative CK and Troponin, reflecting ischemia without cell death. MANAGEMENT: The management of UA/NSTEMI include the simultaneous management consisting of: •relief of ischemic pain - eg, Aspirin, Oxygen (only if hypoxic), Beta blocker therapy to prevent recurrent ischemia and life-threatening ventricular arrhythmias. Morphine in select cases. •initiation of antithrombotic - antiplatelet (Aspirin + P2Y12 receptor blocker) and anticoagulant therapies (eg, Unfractionated Heparin, Enoxaparin, Fondaparinux, Bivalirudin) to prevent further thrombosis. High-intensity statin. •Risk factor assessment - long-term risk stratification with assessment of left ventricular function and either diagnostic coronary arteriography or pre-discharge stress testing. TIMI, GRACE, and PURSUIT scores are used to identify patients at highest risk for further cardiac events who may benefit from a more aggressive therapeutic approach (eg, immediate coronary arteriography and revascularization). A majority of other patients without extremely high risk undergo early coronary revascularization (eg, within 24 hours). Acute management of UA/NSTEMI: •Antiplatelet therapy P2Y12 receptor antagonist (eg, Ticagrelor, Prasugrel, Clopidogrel) in addition to Aspirin to all patients. Consider adding a GP IIb/IIIa inhibitor (either Eptifibatide or Tirofiban) in patients not treated with an invasive approach. •Anticoagulant therapy in all patients: eg, Unfractionated Heparin, Enoxaparin, Bivalirudin, Fondaparinux.

Aortic dissection (ROSH)

Acute aortic dissections are rare but are frequently fatal. Aortic dissection occurs after a tear of the intima allows blood to enter the aortic media, dissecting between the intimal and adventitial layers. The dissecting column of blood forms a false lumen and may extend proximally, distally, or in both directions. Aortic dissection can occur in younger patients with connective tissue disorders that specifically predispose them to aortic dissection. The second and most common group of patients who develop aortic dissection include those over age 50 with a history of chronic hypertension. Chronic hypertension is the most common and most important predisposing risk factor found in patients with aortic dissection. These patients usually complain of acute chest and back pain. Although painless aortic dissections do occur, usually in older patients with diabetes, aortic aneurysm, or previous cardiovascular surgery. Other signs of aortic dissection include a pulse deficit on one side or another, a new heart murmur, or a focal neurological deficit. Diagnosis is confirmed with computed tomographic imaging with angiography. Treatment depends on the type of aortic dissection found.

Aortic stenosis (ROSH)

Aortic stenosis is most commonly due to calcific degeneration and is seen in older patients with coronary artery disease. Other causes of aortic stenosis include a congenitally abnormal valve (usually bicuspid) with calcification or rheumatic heart disease. The normal aortic valve area is typically 3.0 to 4.0 cm2. With aortic stenosis, the valve area is reduced by greater than 50%. Left ventricular hypertrophy develops as a compensatory mechanism to maintain cardiac output. This can result in ventricular dysfunction and left atrial enlargement. Most patients with aortic stenosis are asymptomatic until the stenosis is severe. The most common presenting complaints include dyspnea with exertion or decreased exercise tolerance, presyncope or syncope, and angina. Physical examination reveals a crescendo-decrescendo systolic murmur that radiates to the carotids and is heard best at the second intercostal space. As the stenosis becomes more severe, the murmur peaks later in systole and becomes less intense. Carotid pulses may be delayed and diminished in intensity. The ECG will often show evidence of left ventricular hypertrophy. Surgical valve replacement is the only effective treatment of severe aortic stenosis.

Hypertrophic Cardiomyopathy RAPID REVIEW

Autosomal dominant; 1st degree relatives May present as sudden death in a young athlete Sx: dyspnea on exertion (most common), syncope, orthopnea, chest pain, palpitations PE: harsh crescendo-decrescendo systolic murmur that increases in intensity with Valsalva maneuver and decreases with squatting Dx: repolarization changes on ECG, echocardiography (LVH with septal hypertrophy) Management includes refraining from vigorous physical activity, ICD for high risk patients Rx options for symptomatic patients: beta-blockers or calcium channel blockers

HEART (history, ECG, age, risk factors, and troponin) score ROSH

Chest pain is one of the most common presentations in the emergency department. ED clinicians are tasked with identifying high-risk individuals in need of further investigation for acute coronary syndrome (ACS) as well those who are at low risk and may forego additional studies and, subsequently, be discharged from the ED. The HEART (history, ECG, age, risk factors, and troponin) score was designed as a decision aid for ED clinicians to identify patients presenting with acute, undifferentiated chest pain who are at low risk for major adverse cardiovascular events (MACEs), defined as acute myocardial infarction, need for percutaneous coronary intervention or coronary artery bypass graft, or death within 6 weeks. Patients identified as low risk (HEART score of 0 to 3) are deemed suitable for discharge from the ED without further testing or workup for ACS. The HEART score takes into account the historical features of the patient's chest pain (e.g., location, alleviating and exacerbating factors, radiation, associated symptoms), 12-lead ECG findings at presentation (e.g., ST segment changes, repolarization abnormalities), age, risk factors for ACS (e.g., hypertension, diabetes mellitus, hyperlipidemia, history of coronary revascularization procedure, obesity, current or recent smoker), and troponin lab value. Each of these categories is given a score from 0 to 2. Patients with a total score of 0 to 3 are determined to be at low risk for ACS with a 1.7% rate of MACE at 6 weeks. Therefore, these patients are deemed safe for ED discharge without additional workup. The HEART pathway combines the HEART score with serial troponin testing into a clinical algorithm. Following this pathway, patients with a low HEART score and negative serial troponins at 0 and 3 hours may be discharged home without further testing.

COMPLICATIONS OF MYOCARDIAL INFARCTION 1. Cardiogenic Shock

Clinical Findings Hypotension accompanied by confusion, obtundation or restlessness, cool skin, oliguria, and metabolic acidosis suggests shock. Mild-to-moderate hypotension alone is common in myocardial infarction and does not itself indicate shock. Shock in myocardial infarction may be due to many causes (see Table 11-1), which may be difficult to differentiate Treatment Use any or all of the measures discussed here as necessary (see also Chapter 11). A. Airway Management Give oxygen by mask or nasal cannula. Patients in shock with respiratory failure require endotracheal intubation. B. Pressure Monitoring Consider monitoring central pressure with a Swan-Ganz pulmonary artery catheter (or, far less desirably, a central venous pressure catheter, since in acute myocardial infarction, left ventricular filling pressure can be markedly elevated with normal right ventricular filling pressure, and vice versa). Use an arterial line to measure blood pressure. C. Other Measures Give a fluid challenge (200 mL of saline intravenously over 20 minutes) if the patient is not in congestive heart failure (ie, no rales and no pulmonary edema on chest X-ray). Repeat as needed if congestive heart failure does not develop. Correct arrhythmias (see below). Insert a Foley catheter, and measure urine output hourly. D. Drug Therapy The need for inotropic or pressor support with medications such as dobutamine, dopamine, and norepinephrine should be assessed on an individual patient basis. Use the smallest effective dose, guided by hemodynamic response. E. Percutaneous Coronary Intervention Evidence indicates that acute revascularization by PCI decreases mortality rates compared to immediate medical stabilization. PCI acutely should be seriously considered in such patients because thrombolytics are ineffective in cardiogenic shock associated with an acute myocardial infarction. An intra-aortic balloon pump (IABP) can also be placed, typically in the catheter laboratory, but again this therapy should be considered on a patient by patient basis. IABP counterpulsation can increase cardiac output and improve both coronary and systemic perfusion. IABP counterpulsation is contraindicated in patients with aortic valve disease or those with aortic dissection. Disposition All patients with cardiogenic shock must be hospitalized, preferably in an intensive care unit. Therapy is directed at the likely causes of shock.

COMPLICATIONS OF MYOCARDIAL INFARCTION 2. Congestive Heart Failure

Congestive heart failure is caused by extensive myocardial infarction, volume overload, arrhythmias, acute mitral regurgitation, or ventricular septal rupture. Clinical Findings Symptoms and signs of congestive heart failure include dyspnea, anxiety, tachypnea, tachycardia, pulmonary rales or frank pulmonary edema, jugular venous distention, hypoxemia, and typical findings on chest X-ray (cardiomegaly, pulmonary vascular plethora, Kerley B lines, pleural effusion, or pulmonary infiltrates consistent with pulmonary edema). Wheezing may also be a sign of congestive heart failure (cardiac asthma). Suspect right ventricular infarction in inferior myocardial infarction if signs of right heart failure (right ventricular gallops, elevated central venous pressure, hepatomegaly, peripheral edema) are prominent in the absence of signs of left heart failure (dyspnea, rales, pulmonary congestion on chest X-ray). Treatment A. Airway Management Give oxygen if indicated. Treat respiratory failure if present. B. Drug Therapy 1. NITROGLYCERIN Give NTG either sublingual or spray, followed by IV infusion. NTG will decrease preload as well as afterload. In patients with inferior or right ventricular AMI, nitrates are relatively contraindicated because they may precipitate profound hypotension. Hypotension related to nitrates is treated with reduction or discontinuation of the infusion depending on the degree of symptomatic hypotension. Intravascular volume expansion with intravenous fluid infusions will often quickly correct hypotension. 2. FUROSEMIDE Give furosemide as an intravenous bolus of at least the patient's normal total daily dose. If the patient is not already taking furosemide, administer a 40-mg intravenous bolus initially, and observe the diuretic response by monitoring the patient's symptoms and urine output. Diuretics are contraindicated if right ventricular infarction is suspected. 3. ANGIOTENSIN-CONVERTING ENZYME (ACE) OR ANGIO-TENSIN RECEPTOR BLOCKER (ARB) INHIBITORS can be used to decrease afterload. 4. NONINVASIVE POSITIVE-PRESSURE VENTILATION (CONTINUOUS POSITIVE AIRWAY PRESSURE CPAP/BILEVEL POSITIVE AIRWAY PRESSURE [BIPAP]) has been shown to decrease need for intubation. Alveoli are kept open with positive pressure and work of the heart is reduced. Disposition In the setting of an AMI, patients with congestive heart failure should be closely monitored in an intensive care unit.

pulmonic regurgitation

ETIOLOGIES • Primary PR is almost always congenital. • Secondary (functional) PR can occur in the setting of Pulmonary hypertension or pulmonary artery dilatation. Tetralogy of Fallot, endocarditis, rheumatic heart disease. PATHOPHYSIOLOGY • Retrograde blood flow from pulmonary artery into RV - R-sided volume overload. CLINICAL MANIFESTATIONS: • Most clinically insignificant. If symptomatic, may be associated with right-sided Heart failure symptoms and signs - hepatic congestion (abdominal fullness and bloating), ascites, jugular venous distention, and lower extremity (pedal) edema. PHYSICAL EXAMINATION • Graham Steel murmur; brief high-pitched decrescendo early diastolic blowing murmur heard maximally at the left upper sternal border (left second or third intercostal space). • Increased murmur intensity: inspiration, increased venous return (eg, squatting, leg raise, supine). • Decreased murmur intensity: decreased venous return (eg, Valsalva, standing), expiration. MANAGEMENT • No treatment needed in most (well tolerated). Pulmonic valve replacement (definitive treatment).

For each of the following cardiovascular conditions, students should describe the underlying pathophysiology, presenting signs and symptoms, basic epidemiology, modifiable (when applicable) and non-modifiable risk factors, differential diagnosis, appropriate diagnostic studies, clinical intervention, treatment guidelines, pharmaceutical therapies, and health maintenance concerns: Hypertensive emergencies

HYPERTENSIVE URGENCY •SBP > 180 mmHg and/or DBP >120 mmHg without evidence of end-organ damage. CLINICAL MANIFESTATIONS •General: headache (most common), dyspnea, chest pain, focal neurologic deficits, altered mental status, delirium, seizures, nausea, vomiting. MANAGEMENT •Patients without symptoms or signs of target organ damage have not been shown to benefit from aggressive antihypertensive therapy in the acute setting. •When treatment is indicated, treatment goal is blood pressure <160/100 mmHg (the mean arterial pressure should not be lowered by more than 25 - 30% over the first 2-4 hours). •Gradual reduction of mean arterial pressure by 25% over 24-48 hours with oral medications (eg, Clonidine or Captopril most commonly used agents). •Gradual reduction of mean arterial pressure by no more than 25% over 24-48 hours with oral medications (eg, Clonidine, Captopril, Labetalol, Nicardipine, Furosemide). HYPERTENSIVE EMERGENCY •SBP > 180 mmHg and/or DBP >120 mmHg with evidence of end-organ damage. CLINICAL MANIFESTATIONS •General: headache (most common), dyspnea, chest pain, focal neurologic deficits, altered mental status, delirium, seizures, nausea, vomiting. •Neurologic; encephalopathy, stroke (hemorrhagic or ischemic), seizure. •Cardiac: Acute coronary syndrome, Aortic dissection, Acute heart failure (pulmonary edema). Workup includes CXR, ECG, cardiac enzymes, BNP. •Renal: Acute kidney injury, proteinuria, hematuria (glomerulonephritis). •Retinal; malignant Hypertension, severe (Grade IV) retinopathy. MANAGEMENT •IV blood pressure reduction agents. For most hypertensive emergencies, mean arterial pressure should be reduced gradually by about 10-20% in the first hour and by an additional 5-15% over the next 23 hours. The 3 main exceptions are as follows: •Acute phase of an ischemic stroke: blood pressure is usually not lowered unless it is ≥ 185/110 mmHg in patients who are candidates for reperfusion treatment OR ≥ 220/120 mmg in patients who are not candidates for reperfusion. •Acute aortic dissection: systolic blood pressure is rapidly lowered to a goal of 100-120 mmHg within 20 minutes. •Intracerebral hypertension: treatment depends on different factors.

Contraindications to thrombolytics

History of any intracranial hemorrhage Any known structural cerebral vascular lesion Significant closed-head or facial trauma within 3 months Intracranial or intraspinal surgery within 2 months Any ischemic stroke within 3 months Any intracranial neoplasm Active internal bleeding (eg, serious gastrointestinal bleeding) excluding menses Suspected aortic dissection Uncontrolled hypertension that is severe and refractory to emergency treatment Streptokinase: Previous treatment within 6 months B. RELATIVE CONTRAINDICATIONS In the following conditions, the risks associated with thrombolytic therapy may be increased, and clinical judgment should be used in evaluating expected benefits: Recent (within 10 days) puncture of a noncompressible blood vessel Poorly controlled hypertension of several years' duration Significant hypertension on presentation (diastolic blood pressure >110 mm Hg or systolic blood pressure >180 mm Hg) Diabetic hemorrhagic retinopathy or hemorrhagic ophthalmic condition Oral anticoagulant therapy Major surgery within 3 weeks Pregnancy, active peptic ulcer, dementia Internal bleeding within 2-4 weeks Traumatic or prolonged (>10 minutes) cardiopulmonary resuscitation (CPR) History of nonhemorrhagic cerebrovascular accident beyond 3 months 5. Monitoring Admit patients given thrombolytic therapy to an intensive care unit as soon as possible after initiation of treatment. Monitor the following: Blood pressure every 15 minutes during infusion and every 30-60 minutes thereafter ECG rhythm strip for reperfusion arrhythmias and ST-segment changes Bleeding complications and changes in neurologic status; avoid venous or arterial punctures and unnecessary trauma 12-Lead ECG 4 hours after the start of therapy and as needed for changes cTni or cTnt 4 hours after initiation of treatment and at 4-hour intervals for 24 hours

Hypertrophic obstructive cardiomyopathy (ROSH)

Hypertrophic obstructive cardiomyopathy (HOCM) is a genetic primary cardiomyopathy that is the second most common cause of sudden cardiac death in adolescents. It is the leading cause of sudden cardiac death in athletes. It involves asymmetric interventricular septal hypertrophy that causes reduced filling of the left ventricle and diastolic dysfunction. The most frequent initial complaint is dyspnea on exertion, but other symptoms include chest pain, palpitations, and syncope. Syncope usually occurs during or immediately after exercise. Exam findings include an S4 heart sound and a systolic ejection murmur that is intensified with standing up and performing a Valsalva maneuver. Maneuvers that decreased the murmur include hand grip, passive leg raise, and squatting. Diagnosis is ultimately made with echocardiography or cardiac MRI, along with genetic testing. Hospitalization is indicated for patients presenting for syncope who are suspected to have HOCM, as these patients are at significant risk for sudden cardiac death. Beta-blockers are the treatment of choice in patients with HOCM who have chest pain.

adenosine

INDICATIONS • Paroxysmal supraventricular tachycardia (PSVT): Adenosine slows the sinus rate, slows AV node conduction time, and blocks AV nodal reentry pathways. Adenosine can be both therapeutic and diagnostic (Adenosine can slow down the heart rate long enough to determine if the cause of the tachycardia is due to a different narrow complex tachycardia - eg, Atrial fibrillation or Atrial flutter). Because it has a short half-life (< 10 seconds), each dose of Adenosine needs to be flushed rapidly with 10-20 mL of Normal saline. • Pharmacologic cardiac stress testing: activates adenosine receptors (A1 & A2). Adenosine AZA receptor activation causes vasodilation of normal coronary arteries (excluding diseased coronary arteries). This leads to a shunting effect to mimic increased demand. Adenosine is usually used with thallium-201 stress testing. ADVERSE EFFECTS • Common & short-lived: chest discomfort, dyspnea, flushing, lightheadedness, headache. • Serious: bronchospasm, hypertension, arrhythmias, and myocardial infarction. CAUTIONS/CONTRAINDICATIONS • Use in patients with second and third degree heart block (without a pacemaker), WPW, wide complex tachycardias. • Clinically active bronchospastic disorders (severe asthma/COPD), acute Myocardial ischemia. • hypotension due to A2A activation-related vasodilatation. • AV blocks - A1 receptor activation causes atrioventricular conduction delay. Not used in patients with second or third-degree Heart block without a pacemaker.

Infective endocarditis (ROSH)

Infective endocarditis occurs when microorganisms invade the endocardial surface of the heart, including the heart valves. This causes a range of clinical symptoms, including fever, chills, heart murmur, embolic phenomena, and distinctive skin lesions such as Osler nodes, splinter hemorrhages, and Janeway lesions. If the valves are severely affected, heart failure and cardiogenic shock may result. Most cases of endocarditis occur in individuals either with a predisposing cardiac lesion resulting in turbulent flow around a valve, such as a structural heart defect (congenital or acquired valvular disease), a prosthetic valve, or in those with a risk factor for bacteremia, such as intravenous drug use, intravascular devices, or poor dental hygiene. In endocarditis related to intravenous drug use, the tricuspid valve is the valve most commonly affected. This is because bacteria introduced with dirty needles or nonsterile technique meet the right-sided circulation and the tricuspid valve first. This is in contrast to native valve endocarditis not related to intravenous drug use, in which the left-sided valves are more commonly affected.

Tricuspid regurgitation

PHYSICAL EXAMINATION • High-pitched holosystolic soft (blowing) murmur at the subxiphoid area, left mid sternal border, or right mid sternal border with little to no murmur radiation. • Increased murmur intensity: inspiration, increased venous return (eg, squatting, leg raise, supine). Hepatic compression and exercise. • Decreased murmur intensity: decreased venous return (eg, standing, Valsalva), expiration. • Carvallo's sign: increased murmur intensity with inspiration (due to increased right sided blood flow during inspiration). Helps to distinguish TR from MR. #Pulsatile liver. MANAGEMENT • Medical: diuretics (for volume overload & congestion). If LV dysfunction - standard HF therapy. • Surgical: suggested for patients with severe TR despite medical therapy. Repair >replacement

Amiodarone

MECHANISM OF ACTION • Class III antiarrhythmic (K+ channel blocker) with class I through IV properties. • Prolongs the action potential. INDICATIONS • Most commonly used for stable wide-complex tachycardias but useful for both atrial and ventricular arrhythmias, including refractory SVT. • May be used for rate control in the setting of concomitant Atrial fibrillation + Decompensated heart failure (in this setting both Calcium channel blockers and Beta blockers are contraindicated). ADVERSE EFFECTS • IV use; hypotension most common (due to vasodilation), bradycardia, heart block, vasodilatation, polymorphic ventricular tachycardia, phlebitis. • Long-term use: corneal deposition with > 6 month use (most common side effect), thyroid disorders: hypo- or hyperthyroidism (contains iodine), pulmonary fibrosis, increased LFTs, blue-green discoloration of the skin. Monitor PFTs, TTs, and LFTs for adverse effects. CAUTION • Procainamide and Amiodarone are not generally used together. • Amiodarone is a cytochrome p450 inhibitor. CONTRAINDICATIONS • Second or third-degree heart block who do not have pacemakers • Wolff-Parkinson-White with concurrent Atrial fibrillation.

COMPLICATIONS OF MYOCARDIAL INFARCTION 3. Acute Mitral Regurgitation and Ventricular Septal Rupture

Mechanical failure of infarcted tissue (eg, rupture of the ventricular septum or of papillary muscle supporting the chordae tendineae) is a common cause of acute mitral regurgitation and ventricular septal rupture. Minimal-to-moderate mitral regurgitation is common after myocardial infarction as a result of papillary muscle and left ventricular wall dysfunction. Severe degrees of mitral regurgitation can result from marked ischemia with little or no infarction and can be completely reversed with revascularization. Clinical Findings Abrupt, severe congestive heart failure with pansystolic regurgitation murmur suggests acute mitral regurgitation or ventricular septal rupture. Echocardiography to detect mitral regurgitation or the abnormal velocity jet of a ventricular septal defect can establish the diagnosis. Treatment A. Immediate Measures Treat heart failure with diuretics, morphine, and nitroglycerin. Obtain urgent cardiologic and cardiac surgical consultation. IABP is a useful temporizing measure while the patient is being prepared for surgery. B. Follow-Up Measures The only lifesaving treatment for most patients is emergency cardiac catheterization followed by surgery. Disposition Hospitalize all patients in a critical care unit for treatment and surgery.

For each of the following cardiovascular conditions, students should describe the underlying pathophysiology, presenting signs and symptoms, basic epidemiology, modifiable (when applicable) and non-modifiable risk factors, differential diagnosis, appropriate diagnostic studies, clinical intervention, treatment guidelines, pharmaceutical therapies, and health maintenance concerns: Chest pain

Myocardial infarction: -Hx: hypertension, hyperlipidemia, smoking -Sx: sudden onset substernal heavy chest pain, radiation to left arm, dyspnea, diaphoresis, nausea -Labs: ECG, CPK-MB, troponin x3, CXR, CBC, electrolytes, helical CT, echo, cardiac catheterization Angina: ECG changes -Sx: retrosternal squeezing pain that lasts for 2 minutes and occurs with exercise; relieved with rest; not related to food intake -Labs: ECG, CPK-MB, CXR, CBC, electrolytes, exercise stress test (easiest / most affordable), upper endoscopy / pH monitoring, cardiac catheterization (coronary angiography = gold standard) Aortic dissection -Hx: uncontrolled hypertension -Sx: sudden onset severe chest pain that radiates to back -Labs: TTE, ECG, CPK-MB, troponin, CXR, CBC, amylase, lipase, CTA (chest with contrast), MRI/MRA (aorta), aortic angiography, upper endoscopy Pericarditis -Hx: viral infection -Sx: retrosternal stabbing, chest pain that improves when leaning forward, worsens with deep inspiration -Labs: ECG, CPK-MB, troponin, CXR, echo, CBC, upper endoscopy, ESR CHF -Sx: cough exacerbated by lying down at night and improved by propping with pillows, exertional dyspnea -Labs: CBC, CXR, ECG, echo, PFTs, BNP, CT-chest Lung cancer -Hx: heavy smoker -sx: 6 mo worsening cough, hemoptysis, dyspnea, weakness, weight loss -labs: CBC, sputum gram stain, culture, cytology, CXR, CT-chest asthma: -sx: SOB, cough, wheezing worse in cold air -labs: CBC, CXR, peak flow measurement, PFTs Costochondritis -Hx: viral infection -Sx: stabbing chest pain that worsens with deep inspiration, relieved by aspirin -Exam: chest wall tenderness -Labs: ECG, CXR, CPK-MB, troponin, CBC Pneumonia -Hx: heavy smoker + COPD -sx: 1 week pleuritic chest pain, fever, chills, cough with purulent yellow sputum -labs: CBC, sputum gram stain and culture, CXR, CT - chest, ECG, quantiferon TB gold Pulmonary embolism -Hx: recent immobilization (e.g. surgery) -Sx: acute onset SOB at rest and pleuritic chest pain, tachycardia, hypotension, tachypnea, mild fever -Labs: d-dimer, CTA - chest with IV contrast, CXR, ECG, ABG, CPK-MB, troponin, CBC, electrolytes, BUN/Cr, glucose, doppler U/S (legs) COPD exacerbation (bronchitis) -Hx: COPD, smoker -Sx: increased dyspnea and sputum production -Labs: CBC, CXR, ABG, PFTs, sputum gram stain and culture, CT-chest, echo TB -Hx: contact with TB pts, healthcare workers, traveling -Sx: worsening cough of 6 weeks, weight loss, fatigue, night sweats, fever -Labs: CBC, PPD/quantiferon-TB gold, sputum gram stain, acid fast stain and culture, CXR, CT-chest, bronchoscopy, HIV antibody, lymph node biopsy Pulmonary edema: -Sx: worsening dyspnea of 6 hours + cough with pink, frothy sputum -Labs: CXR, ECG, CBC, ABG, PFTs, BNP GERD -Sx: retrosternal burning sensation that occurs after heavy meals and when lying down; relieved by antacids -Labs: ECG, barium swallow, upper endoscopy, esophageal pH monitoring, H. pylori stool antigen Sickle cell disease - acute chest syndrome -Hx: African American pt. with hx of sickle cell disease -Sx: acute onset severe chest pain with hx of sickle cell disease -Labs: CBC with retic count and peripheral smear, LDH, ABG, d-dimer, CXR, CPK-MB, troponin, ECG, CTA - chest with IV contrast

Orthostatic hypotension (ROSH)

Orthostatic hypotension secondary to dehydration is often a diagnosis of exclusion in syncope patients. However, there are many physical, historical, and laboratory findings that can make the diagnosis of dehydration more likely. Volume depletion generally results from loss of water and sodium from the gastrointestinal tract, renal system, skin loss, or third spacing. This patient reports a recent gastrointestinal problem, which likely indicates fluid loss via vomiting or diarrhea. In this case, when other causes of syncope such as intracranial hemorrhage and dysrhythmia have been ruled out, dehydration is most likely. Often, laboratory testing is unnecessary in the emergency department to diagnose dehydration, and orthostatic vital signs are a common physical exam finding. A patient is considered orthostatic when they have a decrease in systolic or diastolic blood pressure by greater than 20 mm Hg or 10 mm Hg, respectively, within 2 to 5 minutes of standing.

Mobitz type I heart block (ROSH)

This patient is presenting with an ECG demonstrating Mobitz type I heart block. Mobitz type I heart block is typically asymptomatic but can present with weakness, fatigue, and syncope. An ECG will demonstrate a PR interval that gradually prolongs until a QRS complex is dropped. A patient with a new-onset Mobitz type I rhythm should be worked up for an acute ischemic event, including troponins to rule out a new myocardial infarction, and electrolytes to rule out an electrolyte abnormality. A stable patient with Mobitz type I heart block requires no acute interventions and should be managed outpatient by their primary care physician and cardiologist once acute causes are ruled out. Mobitz type I heart block carries a low risk of progressing to complete heart block. This is in contrast to Mobitz type II, an unstable rhythm that carries a high risk of progressing to complete heart block and requires pacemaker placement. Unstable patients with Mobitz type I heart block can initially be treated with atropine and transcutaneous pacing.

Aortic Dissection RAPID REVIEW

Risk factors: advancing age, male sex, HTN, Marfan syndrome Sx: acute onset of "ripping" or "tearing" chest pain or back pain PE: asymmetric pulses or SBP difference of > 20 mmHg CXR: widened mediastinum Dx: CT angiography or transesophageal echocardiogram (TEE) Treatment: reduce BP and HR (beta-blockers), pain control, emergency surgery (Type A dissection) -Type A: involves ascending aorta -Type B: involves only descending aorta

Bacterial Endocarditis RAPID REVIEW

Risk factors: injection drug use, valvular heart disease Sx: fever, rash, cough, and myalgias PE: fever, Roth spots, Osler nodes, murmur, Janeway lesions, anemia, nailbed hemorrhages, emboli (FROM JANE) Diagnosis is made by echocardiography and Duke criteria Most commonly caused by: IVDA: Staphylococcus aureus, tricuspid Native valve: Staphylococcus aureus, viridans streptococci (most common in previously diseased), mitral Tx: antibiotics GI malignancy: Streptococcus bovis Dental prophylaxis in some cases

For each of the following cardiovascular conditions, students should describe the underlying pathophysiology, presenting signs and symptoms, basic epidemiology, modifiable (when applicable) and non-modifiable risk factors, differential diagnosis, appropriate diagnostic studies, clinical intervention, treatment guidelines, pharmaceutical therapies, and health maintenance concerns: Cardiac Arrhythmias (ACLS/BLS protocol)

SINUS ARRHYTHMIA •Regularly irregular rhythm originating from the sinus node with the variations in the rhythm associated with changes of the respiratory cycle. •Normal variation of normal sinus rhythm (meets the same criteria for Normal sinus rhythm except that the rhythm is irregular). EPIDEMIOLOGY: •More commonly seen in children, young adults, healthy individuals, & with Sinus bradycardia. PHYSIOLOGY: Respiratory-phasic: •Beat to beat variations with respiration - rhythm increases with inspiration and decreases with expiration, reflecting changes in stroke volume during respiration. Most common. •During inspiration, blood shunts to the right side to get oxygenated, leaving less blood volume on the left side during inspiration. Because cardiac output = heart rate x stroke volume (CO = HR x SV), during inspiration, the left side of the heart compensates for the physiologic decrease in stroke volume by increasing heart rate to maintain the same cardiac output. Non-respiratory Sinus arrhythmia: •ECG will appear similar to the respiratory type; the difference is that it is not associated with the respiratory cycle. Although it may occur in healthy individuals, it may be indicative of an underlying pathology. CLINICAL MANIFESTATIONS: •The majority of patients with Sinus arrhythmia are asymptomatic (considered a normal variant). It is rare for patients to develop symptoms. •In patients with significant bradycardia, symptoms such as fatigue, exercise intolerance, lightheadedness, dizziness, syncope or presyncope, altered mental status changes, or anginal symptoms may occur. DIAGNOSIS: ECG: •Regularly irregular rhythm: regularly occurring beat to beat variation of the P-P interval (>0.12 sec), shorter intervals during inspiration (increased rate), and longer P-P intervals during expiration (decreased heart rate). •Normal-appearing P waves that are monoform in appearance and consistent with origination from the sinus node - the P wave is upright in leads I and Il, P wave is biphasic in V1. The maximum height of a P wave is less than or equal to 2.5 mm in leads Il and III. MANAGEMENT: •No treatment needed in most cases (it is considered a normal variant in young healthy adults). Symptomatic: •If symptomatic bradycardia occurs, Atropine is the first-line management. •Transcutaneous pacing, Epinephrine and Dopamine are second-line agents.

Pericarditis RAPID REVIEW

Sx: pleuritic chest pain radiating to the back that is worse when lying back and improved when leaning forward PE: tachycardia and pericardial friction rub, distant heart sounds ECG: PR depression, PR elevation (aVR), diffuse ST segment elevation (concave) Most common causes: idiopathic then viral (coxsackie) Tx: NSAIDs, colchicine

For each of the following cardiovascular conditions, students should describe the underlying pathophysiology, presenting signs and symptoms, basic epidemiology, modifiable (when applicable) and non-modifiable risk factors, differential diagnosis, appropriate diagnostic studies, clinical intervention, treatment guidelines, pharmaceutical therapies, and health maintenance concerns: Syncope

Syncope refers to a transient loss of consciousness (TLOC) due to global cerebral hypoperfusion with a rapid onset, brief duration, and spontaneous complete recovery. The evaluation of syncope involves (1) an initial evaluation of stability and any needed resuscitation, (2) an evaluation for any trauma from the syncope, (3) a focused evaluation for cause of the syncope, and lastly (4) risk stratification and appropriate disposition. Unfortunately, there are conditions that resemble syncope causing TLOC which must be included in the initial differential work-up of a patient. An initial evaluation of airway patency, adequate ventilation, and appropriate perfusion (blood pressure/pulse) is the first step in syncope evaluation. Patients with cardiac arrest can sometimes exhibit seizure-like activity with loss of consciousness, and their care should begin with advanced cardiopulmonary life support. Patients presenting with coma or altered mental status need a glucose check, evaluation for hypoxemia, and assessment for need of naloxone or thiamine. Patients presenting with seizures may initially mimic syncope. Most patients with true syncope require no resuscitation having normal vital signs and are back to normal neurologically. Place the patient on cardiac monitoring and establish intravenous access. After initial resuscitation, evaluate for any trauma that occurred due to the syncope. The syncope work-up can be done during the resuscitation as well as any treatment of traumatic injuries which takes precedence if significant trauma has occurred A focused evaluation is done to assess for causes of syncope. The mnemonic "head heart and vessels" can guide one in the work-up as it covers true causes of syncope, as well as syncope mimics. A detailed history from the patient as well as any witnesses, a focused thorough physical examination, electrocardiography (ECG) and orthostatic vital signs comprise the initial diagnostic approach. Further testing is guided by these initial investigations. The primary focus of the initial investigations are to assess for syncope mimics, look for overt causes, and search for lethal cardiac causes. Cerebral malfunction causes of TLOC include (H) hypoglycemia, hypoxemia (carbon monoxide, ventilation/perfusion shunts, and hypercapnic respiratory failure), (E) epilepsy, (A) anxiety, and (D) dysfunction of the brain stem due to basal vertebral transient ischemic attack (TIA) or stroke. Cardiac causes of transient loss of consciousness include (H) heart attack (acute coronary syndrome [ACS]), (E) embolism (PE), (A) aortic obstruction due to hypertrophic obstructive cardiomyopathy, aortic stenosis or myxoma, (R) rhythm disturbances that are slow bradycardia and (T) tachydysrhythmias. Vascular causes include (V) vasovagal (the common faint), (E) ectopic pregnancy (a reminder of blood loss and/or volume loss), (S) subclavian steal, (S) situational (micturition, defecation, cough), (E) ENT (glossopharyngeal or trigeminal neuralgia), (L) low systemic vascular resistance (medications, diabetic neuropathy, Addison disease), and (S) sensitive carotid sinus.

COMPLICATIONS OF MYOCARDIAL INFARCTION 5. Systemic or Pulmonary Embolization

Systemic or pulmonary embolization is commonly caused by intracardiac mural thrombosis or phlebothrombosis. Clinical Findings The most common findings in pulmonary embolism are sudden unexplained dyspnea and tachycardia. Occasionally, pleuritic pain, signs of right heart strain, or abnormal chest X-ray may occur. Patients at greatest risk are those with thrombus visualized in the left or right ventricle by two-dimensional echocardiography. The diagnosis may be confirmed by computed tomography (CT) scan of the chest, lung scan, or arteriography (see Chapter 33). Systemic embolization is suspected when symptoms and signs of arterial occlusion occur. The clinical picture depends on the artery occluded, for example, flank pain and hematuria with renal artery embolism; pallor, pain, and loss of pulse with brachial or femoral artery embolism; stroke with cerebral artery embolism. Treatment and Disposition Give oxygen, draw blood for determination of prothrombin and partial thromboplastin times, and then begin systemic anticoagulation with heparin, 80 units/kg bolus followed by 18 units/kg/h infusion. Pericarditis is a relative contraindication to anticoagulation because of the risk of bleeding into the pericardial sac, with resulting cardiac tamponade. Heparin is also contraindicated in patients with recent stroke, active duodenal ulcer, or active bleeding that cannot be controlled by direct pressure. With a massive pulmonary embolism with right heart failure or shock, intravenous fibrinolysis with tPA has been recommended (see Chapter 33). Seek appropriate surgical consultation (with a thoracic, general, or vascular surgeon) for patients with persistent hypotension, contraindications to thrombolytic therapy, or systemic embolization who may benefit from surgical intervention (eg, angioplasty and embolectomy for a pulseless and ischemic extremity). Patients who have received tPA or have hemodynamic instability should be hospitalized in an intensive care unit.

BLS

THE CHAIN OF SURVIVAL The AHA first proposed the phrase "the chain of survival" to describe the interrelated series of interventions that must be in place to maximize functional survival from sudden cardiac death. Any failure to implement a single link in the chain will lead to a decrease in survival. As depicted by the AHA, there are five links in the chain of survival for out-of-hospital cardiac arrest: recognition and activation of the emergency response system, immediate high-quality CPR, rapid defibrillation for shockable rhythms, basic and advanced EMS services with rapid access to medical care, and ALS and postarrest care. Efforts have been made to expand this chain to include prevention prior to the event and secondary prevention of cardiac arrest. This includes lifestyle modifications (see below) and medical interventions such as statins for hypercholesterolemia, better control of diabetes, and the placement of implantable cardioverter-defibrillators (ICDs). TECHNIQUES OF BASIC LIFE SUPPORT Early Prevention As most cardiac arrests occur in the absence of trained medical personnel, it is imperative that we identify those patients at risk and decrease the likelihood of a cardiac event. Early prevention involves both primary prevention, including lifestyle modification (exercise and smoking cessation), and secondary prevention, including medical therapy. In addition, the patient and family should be educated on the signs and symptoms of chest pain related to cardiac ischemia. Patients with known cardiac disease should attempt to treat the pain with sublingual nitroglycerin. If, after taking the nitroglycerin, there is no relief, the patient should call the emergency number for his or her area. Patients without known disease should contact the emergency number if unremitting chest pain is present for more than 5 minutes. In the hospital setting, an early, organized response to chest pain, dyspnea, and abnormal vital signs can decrease the number of cardiac arrests. Early Access As previously discussed, the need for immediate CPR and defibrillation following cardiac arrest cannot be overstated. On finding the unresponsive patient, a lone rescuer should immediately activate the EMS system and if available obtain an automated external defibrillator (AED). They should then provide adequate CPR. If two or more rescuers are present, CPR should be initiated immediately by one while the other activates EMS. As opposed to a lay provider, a lone health care worker may alter this sequence. If the event is likely asphyxial in origin, the provider may give five cycles of CPR before activating EMS. In the hospital setting, initiate the hospital's emergency response or "code" system. Early access into the system is imperative to minimize the time delay until defibrillation. Early Basic Life Support On finding an unresponsive patient and activating the EMS system, return to the patient to provide basic life support (BLS) until further help arrives after assessing the scene for safety. Remember the importance of provider safety. If the provider is injured, it will obviously impair his or her ability to provide BLS and will add further strain to limited EMS resources. Assume a position alongside the patient's thorax so that rescue breathing and chest compressions can be done without repositioning one's body. Position the patient on a hard surface to allow for efficient chest compressions. Anticipate the arrival of a defibrillator, which is usually positioned on the left side, next to the patient's ear. The Circulation, Airway, and Breathing A. Circulation For a single rescuer, the recommended sequence has changed from checking first the airway, then breathing, and circulation sequence to first checking circulation. Circulation is assessed by palpation of the carotid pulse. The pulse is located in the groove lateral to the trachea. Check the pulse for no more than 10 seconds. If a pulse is present, rescue breathing can continue with one breath every 5-6 seconds. If a pulse is absent, start chest compressions. This is essential for providing blood flow during CPR. To perform chest compressions, identify the lower sternum in the center of the chest, between the nipples. The heel of the hand should be in this position, and place the second hand over the first so that they are overlapped and parallel. Compressions should depress the sternum at least 2 in (5 cm) for the average adult, and then allow the chest to return to normal position. Chest recoil allows venous return to the heart. Compressions are given at a rate of 100-120/min. The compression-ventilation ratio in adults should be 30:2, for both single rescuer and dual rescuer CPR. This results in minimal interruption of compressions and blood flow. Once a definitive airway is established, ventilations and compressions can be performed asynchronously. Continuous end-tidal capnography (ETCO2) is useful to assess the adequacy of chest compressions and to determine when a person performing chest compressions is becoming fatigued. In addition, spikes in continuous ETCO2 can signal the return of spontaneous circulation and conversely, persistently low levels (≤10 mm Hg) after 20 minutes of CPR accurately predicted death. "Hands-Only" CPR Recent recommendations published by the AHA recommend lay rescuers (non-health care providers) provide chest compression only CPR. This stems from research showing no survival advantage from ventilations during bystander-provided CPR compared to chest compressions only. Additionally, without interruptions to chest compressions, more blood flow is delivered to vital organs, and mouth-to-mouth resuscitation is thought to be one of the major psychological obstacles to would-be rescuers during a bystander-witnessed arrest. Extracorporeal CPR Extracorporeal CPR or extracorporeal membrane oxygenation (ECPR, or ECMO) is a technique that requires cannulation of major arteries and veins as well as specialized equipment to provide cardiopulmonary bypass and ECMO during cardiac arrest. The goal of ECPR is to support patients during cardiac arrest while potentially reversible causes of arrest can be treated (such as pulmonary embolism, hypothermia, myocarditis, congestive heart failure [CHF], cardiomyopathy, coronary artery occlusion, drug overdose, etc). A small number of studies suggest that ECPR should be considered as an alternative to conventional CPR for these patients who are identified as having a reversible cause of arrest, especially those who have more than 10-15 minutes of conventional CPR with no return of spontaneous circulation. A. Airway Opening the patient's airway may be achieved using either the head tilt-chin lift or the jaw-thrust method. It is recommended that lay rescuers use the head tilt-chin lift method due to concern that the jaw thrust is more difficult to perform and may not be as effective in opening the airway. To perform the head tilt-chin lift, place one hand firmly on the patient's forehead and apply firm downward, backward pressure to tilt the head. Hook the fingers of the other hand under the bony part of the chin, lifting up the chin (Figure 9-1). If a health care provider is the rescuer and suspects C-spine injury, the jaw thrust is the method of choice to reduce movement of the neck. This maneuver is performed by using the index and middle fingers to grasp the angles of the mandible and lifting with both hands. The head is maintained in neutral position B. Breathing After opening the patient's airway, breathing is assessed. This is done by leaning over the patient's open mouth and looking down at the chest for any rise or fall of breathing. Place an ear near the patient's mouth, listening for breathing or feeling the flow of air over the cheek. This should take no more than 10 seconds. In the absence of adequate breathing (agonal respirations are inadequate), perform mouth-to-mouth, mouth-to-mask, or bag-mask ventilation. Health care professionals should be familiar with all these techniques. A mask with a one-way valve should be readily available in all health care settings to minimize the risk of contamination from the patient's oral secretions. A bag-mask should be available in all critical care areas and on crash carts. 1. MOUTH TO MOUTH Maintain the airway by using the head tilt-chin lift method. Pinch the patient's nose shut. Seal your lips around the patient's mouth. Give two slow breaths over 2 seconds initially. There should be good chest rise and fall. If not, reposition the patient's airway and attempt to ventilate again. If there is still no movement of air, presume that the patient has a foreign body airway obstruction (FBAO) and attempt to clear it (see below). 2. MOUTH TO MASK A clear mask with or without a one-way valve (often called a pocket mask) can be used as an adjunct to mouth-to-mouth ventilation. Ventilations are provided in the same way as for mouth to mouth. It is often easier and more effective for the inexperienced provider to adequately ventilate the patient with the pocket mask than with the bag-mask. When possible, the mask should be attached to supplemental oxygen. There are two techniques to achieve an effective seal with the mask. A. Cephalic technique Used when there is more than one provider. The provider ventilating the patient is positioned at the head of the bed. The mask is applied in one of the following two ways: with the thumbs and thenar eminences providing a seal, while the fingers are used to perform jaw thrust, or with the thumbs and index fingers providing a seal, while the remaining fingers provide a jaw thrust. B. Lateral technique Used when there is only one provider, the lateral technique allows the provider to perform one-person CPR. With the provider at the patient's side, a head tilt-chin lift is applied. The hand closer to the top of the patient's head performs a head tilt while creating a seal on the upper part of the mask with index finger and thumb. The lower hand performs a chin lift with the fingers while creating a seal on the lower part of the mask with the thumb and thenar eminence. 3. BAG-MASK VENTILATION When two providers are available to manage the patient's airway, the mask is applied as with the cephalic technique of the pocket mask. Ventilations are then administered by the second provider squeezing the bag over 2 seconds and then releasing for 3 seconds. If only one provider is available for airway management, he or she must provide an adequate seal with one hand, using the index finger and thumb while providing a chin lift with the other three fingers. The other hand is used to squeeze the bag-mask. Optimizing the head tilt by positioning towels underneath the patient's shoulders may help, especially if the patient is obese. Ventilation can be improved by squeezing the bag against the provider's body. Whenever possible, use supplemental oxygen to provide up to a 100% concentration. 4. CRICOID PRESSURE APPLICATION The application of cricoid pressure is controversial and no longer recommended to be routinely used. Cricoid pressure (Sellick maneuver) is the posterior displacement of the cricoid cartilage to close off the esophagus. This was thought to minimize gastric insufflation associated with ventilation and prevent reflux of gastric contents into the upper airway and lungs in the unconscious patient. FOREIGN BODY AIRWAY OBSTRUCTION While uncommon, FBAO is a preventable cause of death. Identifying a patient with an FBAO may be as simple as recognizing the conscious patient with difficulty breathing, coughing, or the universal "I'm choking" sign, with fingers clutching around the throat, but it can be much more difficult to detect in the unconscious/altered level of consciousness patient. In the conscious patient, ask if he or she is choking. If the patient has a strong cough, is not cyanotic, does not have labored breathing or retractions, and can speak, then a partial obstruction with good air exchange exists. Observe the patient, but do not intervene initially. Allow the patient to attempt to clear the obstruction by himself or herself. If the patient has a weak cough, is cyanotic, has labored breathing, stridor, or has difficulty speaking, then a partial obstruction with poor air exchange exists. Prompt intervention is indicated. If the airway is obstructed completely, the patient is unable to speak or cough and may become cyanotic. Rapid intervention is essential. The abdominal thrust (Heimlich maneuver) is used in the conscious choking patient. While standing behind the patient, position the fist with the thumb facing up over the umbilicus. Roll the fist so that the thumb is against the abdominal wall. Grasp the fist with the other hand. Apply sharp upward thrusts until the foreign body is cleared or the patient collapses and becomes unconscious. In the unconscious patient, position him or her on the floor. Perform a tongue-jaw lift by grasping the jaw and tongue with one hand and observe the airway. Only if a solid obstruction is visually identified, use the other hand to perform a finger sweep. Do not perform a blind finger sweep. Attempt to ventilate. If the first attempt does not succeed, reposition the airway and attempt to ventilate again. If the second attempt fails, perform the Heimlich maneuver by straddling the patient. Place the heel of one hand in the same location as that used for the standing abdominal thrust. Place the other hand over the top of the first. Provide five sharp upward thrusts. Repeat the sequence until the obstruction is relieved or alternative means of establishing the airway are available (eg, Magill forceps, surgical or needle cricothyroidotomy). DEFIBRILLATION Defibrillation is the intervention for the heart in pulseless VT and VF. When a critical level of energy reaches the myocardium, the ventricles become depolarized. This provides an opportunity for the sinoatrial node or another pacemaker to restore an organized perfusing rhythm. For this to occur, the provider must select the proper size paddles or self-adhesive pads, apply them in the correct position (Figure 9-3), use conductive gel, pregelled conductive pads, or saline-soaked gauze to minimize skin resistance, and apply 25 lb of pressure to the paddles. Increasing the energy selected will increase the energy delivered. For a witnessed cardiac arrest into pulseless VT or VF, the defibrillator should be used as soon as possible. For an adult with unmonitored cardiac arrest, CPR can be initiated while the defibrillator is being retrieved and applied. If the monitor displays a rhythm of pulseless VT or VF, defibrillation can then be attempted. The same techniques for defibrillation are used for synchronized cardioversion of tachydysrhythmias. The defibrillator must be placed in the synchronized mode. This is done by activating the SYNCH button on most defibrillators. This coordinates the shock so that it is not delivered during the relative refractory period (a shock during this period could result in VF). Most defibrillators will revert to an unsynchronized mode after delivering a synchronized shock. This allows for rapid defibrillation if VF occurs. For repeated synchronized cardioversion, the defibrillator must be resynchronized. A patient should be adequately sedated and receive analgesia prior to cardioversion, if circumstances permit. Defibrillation Waveforms Waveforms indicate the flow of energy between the two paddles or pads of the defibrillator. A monophasic waveform travels in one direction, whereas a biphasic waveform travels initially in one direction and then reverses flow. Compared to monophasic waveforms, biphasic waveforms have the same rate of defibrillation, with less damage to the myocardium due to lower amounts of energy applied. Research indicates that the biphasic waveform is at least as effective as the monophasic waveform in terminating VF. Most defibrillators manufactured today use biphasic waveforms. Because lower-energy settings are used in biphasic defibrillators, the units are smaller and maintain a battery charge longer than monophasic models. Automated External Defibrillation VF and pulseless VT are the most common dysrhythmias seen in cardiac arrest and may be successfully converted with defibrillation. However, in a matter of minutes this rhythm may degenerate into asystole, which is usually refractory to further interventions. It is thus imperative to deliver defibrillation as rapidly as possible. In an effort to improve the time of arrest till shock, the AHA has recommended the development of lay rescuer AED programs, with ideal sites in areas with large groups of people (ie, sporting events, airports, schools, and shopping malls). The AED is a computerized device that provides visual and often audible prompts to guide lay rescuers in safely defibrillating patients with sudden cardiac arrest. The device itself analyzes the patient's rhythm and determines if a shock is required. Several trials have shown a significant increase in survival to hospital discharge with a community AED program. This is true regardless of what type of first responder (emergency medical technician, firefighter, or police) used the AED. Before applying the AED, the rescuer should initiate airway management and ventilations. If the patient has no pulse, an AED should be brought to the patient's side. If two rescuers are available, one can perform CPR while the AED is sought, but obtaining the AED is the critical action. The AED requires the rescuer to power on the device, apply the self-adhesive pads to the patient's chest, press the Analyze button, and, if an electrical shock is indicated, press the Shock button. The current algorithm for AED use is shown in

Acute coronary syndrome refers to a spectrum of disease when cardiac ischemia results from an imbalance between myocardial oxygen demand and delivery, most often as a result of atherosclerotic plaque disruption and thrombosis in a coronary artery, reducing or blocking coronary blood flow. A STEMI is diagnosed based on characteristic pattern of elevation of ST segments on ECG, which generally corresponds to transmural infarction, and is the most serious type of acute coronary syndrome. Cardiac biomarkers are elevated, reflecting myocardial cell death. In non-ST segment elevation myocardial infarction, cardiac biomarkers are elevated, indicating infarction; however, ECG changes do not include ST elevation. Instead ST segment depression or T wave inversion may be present, indicating ischemia. The ECG can also be normal. In unstable angina, there is diminished coronary blood flow, which causes ischemia, threatening myocardial tissue, but has not yet caused infarction. It is diagnosed clinically with either prolonged anginal chest pain at rest (> 20 minutes), new onset angina that markedly limits ordinary physical activity, or worsening anginal symptoms in a patient with prior angina, including more frequent episodes, and episodes of longer duration and lower in threshold. In unstable angina, both cardiac biomarkers and ECG are non-diagnostic.

This patient has elevated cardiac biomarkers and an ECG with ischemic changes but without ST-elevation, so he has a non-ST elevation myocardial infarction.

pericarditis (ROSH)

This patient's clinical presentation is consistent with pericarditis. Pericarditis is inflammation of the fibrous lining of the heart and can cause chest pain that is positional in nature, a friction rub on auscultation, and diffuse ST elevation as seen above. In the United States and other developed nations, the most likely etiology is viral infection. Other causes can include vasculitis, systemic rheumatic diseases, tuberculosis, and metastatic disease. The initial treatment is ibuprofen or other NSAIDs, with admission indicated for those with fever greater than 101°F, signs of pericardial effusion or tamponade, or those taking anticoagulants. Colchicine can be used as an adjuvant therapy with NSAIDs.

Reflex-Mediated Syncope

VASOVAGAL SYNCOPE •Due to vasovagal hypotension (self-limited systemic hypotension associated with bradycardia and/ or peripheral venodilation/vasodilation). •Most common cause of syncope, especially without apparent neurologic or cardiovascular disease. •Triggers: blood phobia, emotional stress/fear, pain, trauma. •Manifestations: prodromal phase (eg, dizziness, lightheadedness, epigastric pain, palpitations, blurred vision, darkening of visual fields) followed by syncope followed by a postdromal phase. CAROTID SINUS SYNCOPE •Syncope with minor stimulation of the carotid sinus (eg, shaving, putting on neckties, wearing a tight collar, head turning, or applying minor pressure to the carotids). SITUATIONAL SYNCOPE •Triggers include defecation, micturition, coughing/sneezing, post-prandial, or trigger points.

Wondering atrial pacemaker (WAP) and multi focal atrial tachycardia (MAT)

WANDERING ATRIAL PACEMAKER: •Multiple ectopic atrial foci generate impulses that are conducted to the ventricles. •ECG: heart rate <100 bpm & ≥ 3 P wave morphologies. MULTIFOCAL ATRIAL TACHYCARDIA: Same as Wandering atrial pacemaker except the heart rate is >100 bpm. •ECG: heart rate > 100 bpm & >3 P wave morphologies. •MAT classically associated with severe COPD Difficult to treat: Calcium channel blocker (eg, Verapamil) or ß-blocker used if LV function is preserved. PATHOPHYSIOLOGY: •Organized atrial activity with multiple atrial foci competing, including the sinus node, resulting in multiple P waves on the ECG and irregular atrial rhythms. RISK FACTORS: •Pulmonary disease: seen in ~60% of patients - COPD most common pulmonary disorder associated with MAT. Pneumonia, Pulmonary embolism, Pulmonary HTN, Pulmonary failure, hypoxemia, hypercarbia. •Cardiac disease: structural heart damage, Heart failure, acute MI. •Chronic renal disease; Medications: eg, Aminophylline, Theophylline, Isoproterenol. •Electrolyte abnormalities - eg, Hypomagnesemia, Hypokalemia. CLINICAL MANIFESTATIONS: •Usually asymptomatic until patients develop Multifocal atrial tachycardia. •Symptomatic: palpitations, dizziness, fatigue, dyspnea, & chest pain. •Symptoms predominantly relate to the underlying associated illness rather than tachycardia. PHYSICAL EXAMINATION: •Irregular pulse, may be rapid if MAT DIAGNOSIS: ECG: •Irregularly, irregular rhythm + 3 or more identifiable P wave morphologies (including the normal sinus P wave). •Heart rate < 100 bpm = WAP; Heart rate > 100 bpm = MAT. MANAGEMENT: MAT: •Non-dihydropyridine Calcium channel blocker first-line (eg, Diltiazem, Verapamil). They decrease atrial activity and slow atrioventricular node conduction. •Avoid Beta blockers if underlying pulmonary disease (can cause bronchoconstriction). Beta blockers are an option if no underlying lung disease. •In patients with Hypomagnesemia or Hypokalemia, magnesium and potassium repletion is paramount.

Wolff-Parkinson-White (ROSH)

Wolff-Parkinson-White syndrome is characterized by an accessory pathway through which a reentrant tachycardia can occur, resulting in a cardiac dysrhythmia. The location of the accessory pathway is most commonly the left lateral aspect of the atrioventricular (AV) ring, but it can be found anywhere along the AV ring or septum. Clinical manifestations include palpitations, lightheadedness, dizziness, chest pain, syncope, and sudden cardiac death. The majority of patients are asymptomatic and the pattern is noticed on ECG incidentally. The characteristic findings on ECG include a shortened PR interval with a slurring of the QR or R segment of the QRS complex better known as the "delta wave." The two major forms of atrioventricular reentrant tachycardia associated with Wolff-Parkinson-White are orthodromic and antidromic, and the latter is typically distinguished by a widened QRS complex. This is an important rhythm to recognize as procainamide and cardioversion are the only two appropriate treatments. Any other medications or AV-node blockers can result in deterioration and possibly death.

Non-ST Elevation Myocardial Infarction (NSTEMI)

•A type of ACS with critical coronary artery stenosis with myocardial cell injury or death. CHARACTERISTICS •NSTEMI is characterized by: (1) ischemic symptoms suggestive of Acute coronary syndrome (ACS), (2) positive cardiac biomarkers elevated troponin levels and/or CK-MB, AND (3) with or without ECG changes indicative of ischemia (eg, new horizontal or downsloping ST-depression 0.5 mm or greater in two contiguous leads and/or T wave inversion >1 mm in two contiguous leads with prominent R wave or R/S ratio >1). PATHOPHYSIOLOGY: •Plaque rupture: The most common cause is plaque rupture of a previous nonsevere lesion with subsequent thrombus formation, leading to critical coronary artery stenosis with some myocardial injury or cell death. •Unstable plaques (eg, thin fibrous cap, large lipid core, increased macrophages) are more prone to rupture than more stable plaques (eg, thick fibrous cap, small lipid core, low amounts of macrophages). •Coronary artery vasospasm: Less common cause [eg, Variant (Prinzmetal) angina or Cocaine use. CLINICAL MANIFESTATIONS: •Chest discomfort; Chest discomfort (eg, pain, pressure, tightness, constriction, heaviness, or burning) in the center or the left of the chest is classically substernal, diffuse and poorly localized, exertional, not relieved with rest &/or nitroglycerin, pain at rest, > 30 minutes, or change from typical pattern (eg, longer to resolve, increase in severity). •May radiate to the lower jaw & teeth, left arm, epigastric area, shoulders. Levine sign; clenched fist over the chest. DIAGNOSIS: •Electrocardiogram: with or without ECG changes indicative of ischemia - eg, ST segment depression, new deep T wave inversions or flattening, poor R wave progression, pseudonormalization of the T wave, or hyperacute T-waves. •Cardiac biomarkers: Positive. May be normal early on. Cardiac-specific troponins (eg, Troponin I and T) are the preferred cardiac biomarker because they are highly specific for detecting cardiac myocyte injury & are also more sensitive than CK-MB for myocardial necrosis, improving early detection of small myocardial infarctions. •Although blood troponin levels increase simultaneously with CK-MB levels (within 6 hours after the onset of infarction), they remain elevated for as long as 2 weeks. As a result, troponin values cannot be used to diagnose reinfarction. MANAGEMENT: •The management of UA/NSTEMI are the same (see previous page).

Torsades de pointes

•A variant of polymorphic Ventricular tachycardia. PATHOPHYSIOLOGY: •QTc prolongation: TP is a result of QT prolongation >450 ms in males and 460 ms in females (heart rate adjusted lengthening of the QT interval). A QTc > 500 ms has been associated with a 2-fold to 3-fold increase in risk for TdP. May be either be congenital or acquired. Acquired QT prolongation is most often drug-related. Congenital prolonged QT. •Prolonged repolarization phase due to inhibition of the delayed rectifier potassium current, leading to an excess of positive ions within the cellular membrane. •R-on-T phenomenon is the most common ECG phenomenon during ventricular arrhythmia initiation in long QT syndrome. "R on T" phenomenon - a premature ventricular contraction (PVC) or other ectopic beat causes a ventricular depolarization so early in the cardiac cycle it falls on the relative refractory period (apex of the T wave) or during the prolonged repolarization phase, possibly triggering Ventricular tachycardia or Ventricular fibrillation. •Prolonged repolarization and early afterdepolarization + triggered activity [long-QT-related ventricular ectopic beats (eg, PVC occurring on the preceding T wave] can trigger TdP. Bradycardia is a risk factor. CLINICAL MANIFESTATIONS: •Up to 50% of patients are asymptomatic. •Symptomatic: syncope, palpitations, and dizziness. sudden cardiac death in up to 10%. DIAGNOSIS •ECG: polymorphic Ventricular tachycardia (cyclic alterations of the QR amplitude on ECG around the isoelectric line) aka sinusoidal waveform. •Labs: rule out hypomagnesemia and hypokalemia. MANAGEMENT •Recurrent TdP: IV Magnesium sulfate first-line and highly effective treatment for Torsades de pointes. IV Magnesium suppresses early afterdepolarizations, terminating the arrhythmia. Correct any electrolyte abnormalities and discontinue all QT prolonging drugs. •Second-line: Isoproterenol or Transvenous overdrive pacing if refractory to IV Magnesium Isoproterenol helps prevent TdP in patients with prolonged QT that is refractory to magnesium & for patients with polymorphic VT triggered by pauses or bradycardia. Congenital long QT syndrome: •Beta blockers may be used to reduce the frequency of premature ventricular contractions and shorten the QT interval. Avoid Isoproterenol. Hemodynamically unstable: •Synchronized cardioversion should be performed on a hemodynamically unstable patient in Torsades who has a pulse, (100] monophasic, 50] Biphasic). Pulseless TdP: •Prompt defibrillation (unsynchronized) cardioversion is indicated in patients in cardiac arrest (remember that TdP is a variant of VT and pulseless VT is treated with defibrillation).

Wolf Parkinson White Syndrome (WPW)

•A ventricular preexcitation syndrome resulting in an abnormal conduction through an accessory pathway (bundle of Kent) that bypasses the atrioventricular (AV) node. •WPW is a type of AV reciprocating tachycardia (AVRT). PATHOPHYSIOLOGY: •Antegrade conduction through the fast accessory pathway outside of the AV node (bundle of Kent) bypasses the slower-conducting AV node, "preexciting" the ventricles (depolarization & activation of the ventricles earlier than normal via direct connection between the atria & ventricles), leading to aberrant conduction and the characteristic delta wave (slurred and widened QRS). CLINICAL MANIFESTATIONS •Most patients are asymptomatic but they are prone to the development of tachyarrhythmias. •Symptomatic: palpitations (most common), anxiety, exercise intolerance, dizziness, lightheadedness, diaphoresis, fatigue, dyspnea, presyncope, syncope (rare), chest pain or discomfort. DIAGNOSIS: ECG: 3 components "WPW" •Wave - delta wave (initial slurred upstroke of the QRS) •PR interval that is short (<0.12 seconds) •Wide QRS complexes (> 0.12 seconds) MANAGEMENT Stable (wide complex): •Procainamide preferred in hemodynamically stable WPW with wide complex tachycardia (including known or suspected antidromic AVRT) or Atrial fibrillation due to WPW syndrome. Synchronized cardioversion if Procainamide is ineffective or not available. •A fib due to WPW: IV Procainamide preferred. Ibutilide is an alternative. •Avoid AV nodal blocking agents ABCD (Adenosine, Beta blockers, Calcium channel blockers, Digoxin) in wide complex WPW and Atrial fibrillation due to WPW because AV nodal blockade can lead to preferential conduction down the aberrant tract, worsening the tachycardia. Stable (narrow complex): •Treated similar to SVT: Initial treatment of acute symptomatic orthodromic AVRT with one or more vagal maneuvers (eg, Valsalva maneuver and carotid sinus massage). •If vagal maneuvers are ineffective, AV nodal blocking agents - Adenosine, usually preferred. •IV Non-dihydropyridine Calcium channel blockers (eg, Verapamil, Diltiazem) often used second-line to Adenosine. IV Beta blockers (eg, Metoprolol, Esmolol, Atenolol). Procainamide. Unstable: •Direct current (synchronized) cardioversion should be performed urgently in hemodynamically unstable patients with tachyarrhythmias when the rhythm is not sinus tachycardia (preferred management in most). •Signs of instability include hypotension, hypoxia, altered mental status, signs of shock or poor end-organ perfusion, ongoing ischemic chest pain, and evidence of acute pulmonary edema. CHRONIC PREVENTION: •Radiofrequency catheter ablation of the accessory pathway definitive management due to its high success rate and low complication rates. Indicated for the long-term management of symptomatic tachyarrhythmias associated with WPW, occupations in which the development of symptoms would put themselves or others at risk (eg, truck drivers, bus and train operators, airline pilots), and some selected asymptomatic patients.

Third degree AV block

•AV dissociation = complete absence of AV conduction where no atrial impulses conduct to the ventricle, so the atrial activity and ventricular activity are independent of each other. ETIOLOGIES •Secondary or Primary (idiopathic) progressive cardiac conduction disease with myocardial fibrosis and/or sclerosis that affects the conduction system - eg, myocardial ischemia involving the conduction system (acute or chronic), myocardial fibrosis, myocarditis (eg, Lyme disease), Restrictive cardiomyopathy leading to infiltrative disease involving the conduction system (eg, Amyloidosis, Sarcoidosis), or Infective endocarditis associated with abscess formation. Increased vagal tone, hypothyroidism, hyperkalemia. •Inferior wall MI: the right coronary artery supplies the AV node in ~90% of the population. Anterior myocardial infarctions may involve infranodal tissues. •Hyperkalemia, Hypothyroidism, increased vagal tone. •Autoimmunity: eg, Systemic lupus erythematosus, Rheumatic fever, Systemic sclerosis. •iatrogenic - AV nodal blockers (eg Beta blockers, Non-dihydropyridine Calcium channel blockers, Digoxin), Amiodarone, post-catheter ablation, post-cardiac surgery, post-transcatheter aortic valve implantation, surgery near the septum (eg, mitral valve repair). CLINICAL MANIFESTATIONS •Patients are infrequently asymptomatic; most patients present with some degree of symptoms (eg, fatigue and/or dyspnea). •If symptomatic, it is due to bradycardia-related decreased perfusion, especially during exertion fatigue, dyspnea, dizziness, chest pain, near syncope, syncope. •Signs of hemodynamic instability include hypotension, altered mental status, signs of shock, ongoing ischemic chest pain, and evidence of acute pulmonary edema. DIAGNOSIS •ECG: AV dissociation: evidence of atrial (P waves) and ventricular (QRS complexes) activity which are independent of each other and an atrial rate faster than the ventricular rate. Regular P-P intervals & regular R-R intervals independent of each other. Patients are often bradycardic. MANAGEMENT Asymptomatic: •Urgent use of Atropine not needed if asymptomatic but continuous monitoring, transcutaneous pacer pads are placed in case of clinical deterioration, and assessment for reversible causes should be sought and corrected, with observation for improvement after correction. Symptomatic & stable: •Atropine first-line if clinically symptomatic bradycardia occurs as per ACLS. However, Atropine is rarely effective in raising the heart rate in complete heart block, so pacing often needed. •If unresponsive to Atropine, temporary pacing (transcutaneous or transvenous) should be initiated for stabilization. Temporary pacing & revascularization if in the setting of Acute MI. Unstable: •Urgently treated with Atropine, and in most cases, temporary cardiac pacing to increase heart rate and cardiac output (either with transcutaneous or, if immediately available, transvenous pacing). Atropine is 0.5 mg IV may be repeated every 3-5 minutes to a total dose of 3 mg. Definitive: •Permanent pacemaker indicated in most patients when there is no reversible etiology present.

AV junctional dysrhythmias

•AV node/junction becomes the dominant pacemaker of the heart in AV junctional rhythms. ECG: •Regular rhythm. P waves inverted (negative) if present in leads where they are normally positive (I, II, aVF) or are not seen. Classically associated with a narrow QRS (+ wide). Junctional Rhythm; heart rate is usually 40-60 bpm (reflecting the intrinsic rate of the AV junction). Accelerated Junctional: heart rate 60-100 bpm. Junctional Tachycardia: heart rate >100 bpm.

For each of the following cardiovascular conditions, students should describe the underlying pathophysiology, presenting signs and symptoms, basic epidemiology, modifiable (when applicable) and non-modifiable risk factors, differential diagnosis, appropriate diagnostic studies, clinical intervention, treatment guidelines, pharmaceutical therapies, and health maintenance concerns: Pericardial effusion

•Accumulation of fluid in the pericardial space. •Normally, about 5-15 ml of fluid is in the pericardial space. ETIOLOGIES •Same causes of acute Pericarditis (eg, viral, idiopathic). Immune, malignancy - lung cancer most common malignant cause, breast second most common; aortic dissection, uremia). CLINICAL MANIFESTATIONS •Chest pain (if associated with acute Pericarditis), dyspnea, fatigue. •Physical examination; decreased (muffled) heart sounds (due to fluid). DIAGNOSIS •Echocardiogram: test of choice increased fluid in the pericardial space. •ECG: The most common ECG findings in patients with a Pericardial effusion are sinus tachycardia, low QRS voltage, & electrical alternans (alternating amplitudes of the QRS complexes in large effusions). ELECTRIC ALTERNANS TALL QRS SHORT QRS •Chest radiograph: not used in the diagnosis. Findings may include appearance of the heart as a "water bottle" (not sensitive or specific). MANAGEMENT •Treat the underlying cause (eg, acute Pericarditis). Serial echocardiography if necessary. •Large effusions may need pericardiocentesis for symptomatic relief.

Right ventricular MI (inferior or posterior wall MI)

•Acute coronary syndrome with complete occlusion of the right coronary artery (RCA) through the RV marginal branches in 80%. 20% posterior descending artery a branch of the LCA in 20%. PHYSICAL EXAMINATION: •Bradycardia or heart blocks the RCA supplies the AV node in 90% so bradycardia may be due to AV nodal dysfunction (less commonly sinoatrial dysfunction) due to ischemia, activation of cardioinhibitory reflexes, or both). Tachycardia may be seen due to increased sympathetic activity from decreased cardiac output or anxiety. •Fourth heart sound: May have S4 (especially with inferior MI). •Hypotension is commonly seen with large RVMI (may be accompanied by shock). •Triad of right ventricular infarction: increased JVP (jugular venous distention) + clear lungs + positive Kussmaul sign. ELECTROCARDIOGRAM: •Inferior MI: ST elevations > 1 mm in at least 2 contiguous inferior leads (I, III, aVF) with or without reciprocal changes (ST depressions) in the lateral leads (I and aVL). •Posterior MI: ST depressions: Often horizontal ST depressions in at least 2 contiguous anterior leads (V1 through V4), tall broad R waves (>30 ms), or dominant R wave (R/S ratio >1) in V2. •Because the standard ECG does not look directly at the posterior heart, these changes occur because the standard ECG sees the "ST elevation MI" backwards (ST depressions) and the "O waves" backwards (as tall broad R waves). •Right-sided ECG may increase the diagnosis if ST elevations seen in V4R-V6R (ST elevation > 1 mm in lead V4R has a sensitivity & specificity for >90%). MANAGEMENT •Treated in a manner similar to those with acute left ventricular ST-elevation MI, including Aspirin, oxygen (if hypoxic), reperfusion therapy (PCI preferred or thrombolytics), dual oral antiplatelet therapy (Aspirin plus a platelet P2Y12 receptor blocker), statin therapy, an anticoagulant. •Although Beta blockers can be used in some patients, medications to improve chest pain (eg, Nitrates, Opioids, and Beta blockers) should be used with caution due to their potential to negatively impact preload, leading to hypotension (Opioids and Nitrates) or heart rate and contractility (Beta blockers and Calcium channel blockers). This is because Inferior and Posterior wall MIs are more preload dependent to maintain cardiac output. •Volume loading IV fluids (usually isotonic saline) should be given to patients with evidence of low cardiac output (hypotension, hypoperfusion, and a low or normal JVP) who do not have pulmonary congestion or evidence of right heart failure. Because RVMI are more dependent on preload, IV fluids improve preload and forward flow out of the right ventricle. In most cases, a carefully monitored volume challenge is initiated by infusing small amounts (200 to 300 mL) of normal saline with continuing assessment of JVP and blood pressure. Other considerations: •Correct any electrolyte abnormalities, especially hypokalemia and hypomagnesemia, which often occur simultaneously. Maintain the serum potassium concentration >4.0 mEg/L and a serum magnesium concentration >2.0 mEq/L (2.4 mg/dL or 1 mmol/L) during the acute phase. MEDICATIONS TO AVOID: •NSAIDs (except Aspirin) should be discontinued immediately if possible due to an increased risk of cardiovascular events associated with their use. •Although Beta blockers can be used in some patients, medications to improve chest pain (eg, Nitrates, Opioids, and Beta blockers) should be used with caution due to their potential to negatively impact preload, leading to hypotension (Opioids and Nitrates) or heart rate and contractility (Beta blockers and Calcium channel blockers). This is because Inferior and Posterior wall MIs are more preload dependent to maintain cardiac output.

For each of the following cardiovascular conditions, students should describe the underlying pathophysiology, presenting signs and symptoms, basic epidemiology, modifiable (when applicable) and non-modifiable risk factors, differential diagnosis, appropriate diagnostic studies, clinical intervention, treatment guidelines, pharmaceutical therapies, and health maintenance concerns: Congestive heart failure

•Acute decompensated heart failure (ADHF) is a clinical syndrome of new or worsening signs and symptoms of Heart failure, often leading to hospitalization or an ED visit. ETIOLOGIES: •Coronary artery disease: eg, Acute coronary syndrome, Myocardial ischemia & infarction, Ventricular septal rupture. Myocarditis; Valve disease: Acute valve syndromes or progressive valvular disease; Cardiomyopathy; Poorly controlled Hypertension. •Precipitating factors: eg, rhythm disturbances, infection, uncontrolled hypertension, and nonadherence to diet and/or medications. Beta-blockers, non-dihydropyridine Calcium channel blockers, NSAIDs, and alcohol. CLINICAL MANIFESTATIONS: •Left-sided: pulmonary symptoms progressive dyspnea most common complaint. Cough, rales. Think L for L-sided and Lungs. •Right-sided: systemic symptoms of peripheral and abdominal congestion (eg, increased JVP, lower extremity edema, hepatojugular reflux, Gl symptoms). Think R for R-sided and roads to the heart (IVC, SVC). •Other symptoms include nocturia and neurologic symptoms (eg, confusion, headaches, insomnia, anxiety, disorientation, and impaired memory). •Cardiogenic shock: systolic blood pressure < 90 mmHg with symptoms and signs of decreased end-organ perfusion - fatigue, altered mental status, tachycardia, cold and cyanotic periphery with poor capillary refill and cold mottled skin; diminished pulse pressure, or signs of organ hypoperfusion (eg, prerenal azotemia or abnormal hepatic enzymes). PHYSICAL EXAMINATION: Signs are due to congestion: •Lungs; crackles (rales), rhonchi, or wheezing. Diminished air entry at the lung bases if a Pleural effusion is present (more common on the right side). •Peripheral: lower extremity edema (especially in the pretibial region and ankles in ambulatory patients with sacral edema in bedridden patients), elevated jugular venous pressure. Pulsus alternans. Cool lower extremities may be suggestive of worse cardiac output. •Abdomen: epigastric tenderness, positive hepatojugular reflex; tender and enlarged liver. Ascites or hepatomegaly may be seen. •Cardiac examination: - advanced systolic dysfunction associated with a third heart sound (S3) and a laterally displaced apex beat. A murmur of Mitral regurgitation is often audible when the left ventricle (LV) is markedly enlarged, or a tricuspid regurgitation murmur is present when the right ventricle is volume or pressure overloaded. DIAGNOSIS: •The diagnosis of ADHF is based primarily on signs and symptoms (eg, exertional dyspnea and fluid retention) and supported by biomarkers (B-type natriuretic peptide [BNP] or N-terminal proBNP [NT-proBNP]), chest radiograph, ECG, and Doppler echocardiography. Chest radiograph: •Indications: important to assess for signs of pulmonary congestion or edema in acute decompensated heart failure. •Findings: cephalization of flow (redistribution of blood flow from bases to the upper lobes) followed by Kerley B lines (thin septal lucent horizontal lines in the peripheral lung fields), butterfly (bat wing) appearance (bilateral perihilar alveolar edema) followed by cardiomegaly, pleural effusions, and pulmonary edema (bilateral interstitial markings) •Up to 20% of ADHF may have normal chest radiographs. Brain natriuretic peptide: •Brain natriuretic peptide levels (BNP) > 100 or N-terminal proBNP >125 pg/mL makes CHF likely (increases in BNP levels may be caused by intrinsic cardiac dysfunction). •Indications: A BNP or NT-proBNP level is obtained when the diagnosis of HF is uncertain. The high negative predictive value of BNP tests is particularly helpful for ruling out Heart failure as BNP is an objective measure of cardiac function. •BNP levels increase with age & with renal insufficiency; May be decreased on chronic treatment. Electrocardiogram (ECG): •Indications: assesses for evidence of ACS (myocardial ischemia or MI) and arrhythmias (eg, Atrial fibrillation). The ECG may also identify other predisposing or precipitating conditions for HF such (eg, ventricular hypertrophy, left atrial abnormalities). •Findings: often nonspecific ST & T changes. Other labs: •Serum electrolytes, bicarbonate, CBC, blood urea nitrogen (BUN), and serum creatinine. •Lactic acid level in patients in shock as well as in those with marked weakness and/or uncertain peripheral perfusion. Serum troponin (Tor I) levels are obtained in any patient with possible ACS. Echocardiogram •Indications: particularly helpful if this is a first presentation of HF or if there has been abrupt deterioration in the patient's condition. •Benefits: Doppler echocardiography is helpful in the diagnosis and classification of HF. Doppler echocardiography enables evaluation of ventricular size, global and regional systolic function, diastolic function, valvular disease, & pericardial disease. Also aids in estimation of right atrial pressure, pulmonary artery pressures, and pulmonary capillary wedge pressure. Swan-Ganz catheterization •Generally, not required but helpful when the diagnosis is uncertain and for management of patients with persistent symptoms despite treatment of HF. MANAGEMENT •Initial therapy: includes early IV loop diuretic therapy for volume overload, seated posture, supplemental oxygen to treat hypoxemia (SpO2 <90%), and assisted ventilation if necessary. •Initial stabilization: eg, airway assessment, continuous pulse oximetry, supplemental oxygen and ventilatory support, and vital sign assessment, cardiac and urine output monitoring. Loop diuretic (eg, Furosemide): •Mechanism of action: IV diuretics decrease dyspnea, symptoms of fluid overload, and peripheral edema (removes fluid off of the lungs). Have anti-vasoconstrictor effects. •Indications: patients with ADHF and evidence of volume overload, regardless of etiology, should be promptly treated with IV diuretics as part of their initial therapy without delay. •Contraindications: Rare exceptions in which some delay in diuresis may be required include patients with severe hypotension (eg, SBP < 85 mmHg) or cardiogenic shock. •Monitoring: volume status, evidence of congestion, oxygenation, daily weight, fluid intake, and output should be continually reassessed. Position: •Posture: Seated posture and have legs dangle over bed to decrease preload/venous return. •Positive pressure ventilation - Continuous Positive Airway Pressure (CPAP) & Bilevel Positive Airway Pressure (BiPAP) are noninvasive methods of respiratory support for respiratory insufficiency secondary to pulmonary vascular congestion & pulmonary edema. CAP & BIBAP reduce the need for intubation & mechanical ventilation in HF with respiratory decompensation. Oxygen (02): •Mechanism of action; relieves the sensation of dyspnea. •Indications: hypoxemia (eg, Sp02 <90%). 02 supplementation can be titrated to keep the patient comfortable and arterial oxygen saturation consistently >90% (88-92% in patients with COPD). •Oxygen not recommended as routine therapy without hypoxemia as it may cause vasoconstriction and reduction in cardiac output. •Non-rebreather facemask (high-flow 100% oxygen) preferred because the concentration of oxygen delivered is greater than with nasal cannula. •Noninvasive ventilation (NIV) indicated if respiratory distress, respiratory acidosis, and/or hypoxia persist on oxygen therapy, as long as emergent intubation is not indicated. •Intubation if severe or no response to NIV (within 30 mins - 2 hours) Morphine (opiates): •Mechanism of action: decreases dyspnea, venodilator (decreased preload), and decreases anxiety. •Given the limited evidence of benefits and potential risks of opiates, opiate therapy is generally not used in the treatment of ADHF. Nitrates: •Mechanism of action: Nitrates reduce LV filling pressure primarily via venodilation (greater venous than arterial vasodilation). At higher doses, Nitrates may lower systemic vascular resistance & LV afterload, increasing stroke volume & cardiac output. Anxiolytic & analgesic properties. •Indications: early use of a vasodilator that decreases venous tone (eg, Nitroglycerin) when the initial response to diuretics is not sufficient to alleviate respiratory distress. •Administration: IV route is used for faster onset, reliability of delivery, and ease of titration. An initial dose of 5-10 mcg/min with the dose increased in increments of 5-10 mcg/min every 3-5 minutes as required and tolerated (dose range 10-200 mcg/ min). •Adverse effects: Headache, Hypotension. Should be avoided or used with caution in settings in which Hypotension is likely or could result in serious decompensation (eg, RV infarction or aortic stenosis). •Contraindications: after the use of PDE-5 inhibitors (eg, Sildenafil) - concomitant use of both can cause profound vasodilatation. Nitroprusside: •Mechanism: vasodilator that lowers arterial tone. •Indications: IV Sodium nitroprusside may be used in selected patients with ADHF who require a rapid decrease in systemic vascular resistance and LV afterload (g, severe Hypertension, acute Mitral regurgitation, or acute Aortic regurgitation). MANAGEMENT OF HYPOTENSION •The management of ADHF with hypotension may include positive Inotropic agents (eg, Dopamine, Dobutamine, Milrinone) &/or Vasopressor therapy (eg, Norepinephrine). Dopamine, Dobutamine, & Milrinone: •Mechanism of action: positive inotropes that increase contractility to increase cardiac output. •Indications: temporary (short-term) positive inotrope for severe decompensated Heart failure with hypotension to maintain systemic perfusion to vital organs. Indicated if systolic blood pressure is <85 mmHg or there is evidence of shock (eg, cool extremities, narrow pulse pressure, low urine output, confusion) and severe LV dysfunction. •May be needed until definitive therapy (eg, coronary revascularization, mechanical circulatory support, or heart transplantation) is instituted or resolution of the acute precipitating problem. •Dobutamine is preferable for beta-blocker naive patients, while Milrinone is preferred for patients previously taking oral beta blockers who experience an acute decompensation. •Contraindications: Not generally used as long-term therapy (long-term use can increase mortality). Inotropes are not indicated for ADHF treatment with preserved systolic function. Vasopressor therapy: Norepinephrine or high-dose Dopamine. •Indications: vasopressor use is limited to patients with persistent hypotension with symptoms or evidence of consequent end-organ hypoperfusion despite optimization of filling pressures and use of inotropic agents as appropriate. Used as a temporizing measure to preserve BP. CAUTION •Beta blockers (BB) reduce mortality when used in the long-term management of patients with HFrEF, but should be used cautiously in patients with decompensated HFrEF because of the potential to worsen acute HF. For patients already on BBs with severe decompensation (eg, severe volume overload and/or requiring inotropic support), BBs are usually temporarily withheld. For patients who are already taking a BB with moderate-to-severe decompensation or hypotension, decreasing or withholding beta blocker therapy is recommended. •Non-Dihydropyridine CCBs; There is no mortality benefit of non-Dihydropyridine Calcium channel blockers in the management of Heart failure with reduced ejection fraction and there may be a possible deleterious effect of the use of non-Dihydropyridines, •Amlodipine, a Dihydropyridine CCB, appears to be safe in HF and may be used if CCB is necessary for a concomitant disease, such as Hypertension or Angina.

For each of the following cardiovascular conditions, students should describe the underlying pathophysiology, presenting signs and symptoms, basic epidemiology, modifiable (when applicable) and non-modifiable risk factors, differential diagnosis, appropriate diagnostic studies, clinical intervention, treatment guidelines, pharmaceutical therapies, and health maintenance concerns: Acute peripheral ischemia

•Acute limb ischemia - rapidly developing or abrupt decrease in limb perfusion, usually associated with new or worsening symptoms or signs of limb ischemia. •Considered a vascular emergency - Acute arterial occlusion is time-sensitive and if left untreated, can lead to rapid development of infarction and loss of limb. ETIOLOGIES: •Thrombotic occlusion - in situ acute thrombosis of a diseased but previously patent vessel most common etiology (eg, preexisting Coronary artery disease or Peripheral arterial disease associated with intermittent claudication, history of limb revascularization, or atherosclerotic risk factors). •Embolic occlusion from a proximal source lodging into a more distal vessel - the left heart (eg, atrial thrombus due to Atrial fibrillation), Infective endocarditis. CLINICAL MANIFESTATIONS: •6 Ps - paresthesias (often early), pain, pallor, pulselessness, poikilothermia, paralysis (late finding associated with a worse prognosis). Symptoms usually distal to the occlusion. •Physical examination: Classic findings of Acute limb ischemia include decreased capillary refill, decreased or absent pulses, & cool temperature. WORKUP •Bedside arterial Doppler A handheld Doppler should be used to confirm the presence of distal pulses (eg, dorsalis pedis, posterior tibial Doppler signals). •Vascular imaging: Duplex ultrasound: often the first imaging choice to assess Acute limb ischemia because it is low cost, widely available, non-invasive, non-irradiant, and can be performed rapidly. In emergent situations, angiography is often performed. •Angiography: CT angiography (quicker and more readily available) or catheter-based angiography used in patients with viable or marginally threatened limbs to assess arterial anatomy, distinguish between thrombus and embolus, and allows for possible catheter-based therapeutic intervention (eg, thrombolysis, angioplasty). •Catheter-based angiography (with digital subtraction) provides the most useful information and plays a role in therapeutic strategy (eg, also allows for treatment). •An immediately threatened limb may undergo further evaluation and treatment in a surgical suite. MANAGEMENT: •Therapeutic management depends on the type of occlusion (thrombus or embolus), location, type of conduit (artery of graft), Rutherford class, duration of ischemia, co-morbidities. •Supportive management; While the patient waits for surgical or interventional radiology assessment, initial therapy should include fluid resuscitation, pain control, and urgent anticoagulation with Unfractionated heparin to minimize thrombus propagation and preserve microcirculation (target aPTT 2.0 - 2.5 above baseline). •Revascularization for reperfusion is the mainstay of treatment - surgical procedures include surgical or catheter-based thrombectomy with a balloon catheter (Fogarty), bypass surgery and adjuncts such as endarterectomy, patch angioplasty, and intra-operative thrombolysis depending on the duration of the ischemia and the extent of occlusion. Frequently, a combination of these techniques is required. •Catheter-directed thrombolysis is often reserved for patients with a salvageable limb. •Open revascularization: may be indicated as the best option in an immediately threatened or nonviable limb, bypass graft with suspected infection, or if contraindication to thrombolysis. •Amputation: indicated in irreversible ischemia (eg, profound paralysis and absent pain with inaudible arterial and venous pulses).

Hypertrophic Cardiomyopathy (HCM)

•Autosomal dominant genetic disorder of inappropriate LV and/or RV hypertrophy resulting in left ventricular outflow obstruction, diastolic dysfunction, and myocardial ischemia. ETIOLOGY: •Familial Hypertrophic cardiomyopathy most common ~50% of cases. Associated with autosomal dominant mutations on chromosome 14 (the genes encoding for sarcomere proteins). PATHOPHYSIOLOGY: •Concentric hypertrophy (especially septal) from sarcomeres added in parallel without chamber dilation. •Diastolic dysfunction: ventricular hypertrophy results in impaired left ventricular relaxation (decreased compliance) of the LV. The ventricles are hypercontractile. •Subaortic outflow obstruction left ventricular outflow tract (LVOT) obstruction due to 1) asymmetrical septal hypertrophy and 2) systolic anterior motion (SAM) of the mitral valve. •The obstruction worsens with increased contractility (eg, exercise, Digoxin, beta agonists) and decreased LV volume (eg, dehydration, decreased venous return, Valsalva maneuver). Histology: •Myocyte hypertrophy with a gross disarray of the myofibrils & disorganization of the muscle architecture with varying amount of interstitial fibrosis. Abnormal internal coronary vessels. CLINICAL MANIFESTATIONS: •Many patients with HCM are asymptomatic and are diagnosed during family screening, by auscultation of a murmur during routine examination, or incidentally after an abnormal ECG. •Symptomatic patients usually present with left ventricular outflow tract (LVOT) symptoms of dyspnea on exertion (most common 90%), fatigue, chest pain, and syncope. •Chest pain anginal or atypical. Typical exertional chest pain (angina) seen in 25-30% of patients with HCM, usually in the setting of a normal coronary arteriogram. •Pre syncope, syncope, or dizziness especially during or immediately after exertion. Palpitations. •Sudden cardiac death especially in adolescent or preadolescent children especially during times of extreme exertion often due to Ventricular arrhythmias (eg, Ventricular fibrillation). PHYSICAL EXAMINATION: •Fourth heart sound (S4) occurs during left ventricular (LV) filling when atrial contraction forces blood into a noncompliant (stiff or hypertrophic) LV. Double apical impulse (noncompliant LV). •Pulsus bisferiens: biphasic pulse - aortic waveform with two peaks per cardiac cycle, a small one followed by a strong and broad one). Due to sudden deceleration of blood due to the development of midsystolic obstruction to blood flow and partial closure of the aortic valve. Systolic murmur •Harsh crescendo-decrescendo systolic murmur that begins slightly after S1 and is heard best at the apex and lower left sternal border. Usually no carotid radiation (unlike Aortic stenosis & AS is best heard at the right upper sternal border). •Increased murmur intensity with decreased venous return (eg, Valsalva, standing, or assuming an upright posture from a squatting, sitting, or supine position) due to decreased preload; decreased afterload (eg, vasodilators like Amyl nitrate) due to increased forward flow through the LV outflow tract. •Decreased murmur intensity with increased venous return (eg, after going from a standing to a sitting, squatting or supine position, leg raise) due to increased preload; increased afterload (eg, handgrip) due to decreased forward flow through the LV outflow tract. DIAGNOSIS: •A cardiac history, physical examination, ECG, and cardiac imaging (eg, echocardiogram or cardiac MR) to identify left ventricular hypertrophy should be performed in all patients with suspected HCM. ECG: •ECG testing is the most sensitive routinely performed diagnostic test for HCM. Because the ECG abnormalities are not specific to HCM, echocardiography often also performed. •LVH; left ventricular hypertrophy associated with left axis deviation. •Prominent abnormal Q waves - pseudo q waves in the anterolateral (1, aVL, V4-V6) and inferior leads (Il, III, aVF). These changes are due to septal depolarization of the hypertrophied myocytes. Echocardiography: •hypertrophy: asymmetric ventricular wall thickness (especially interventricular septum) 15mm or greater, systolic anterior motion of the mitral valve, small LV chamber size Genetic testing: •Can be done for patients or their first-degree relatives to look for disease-causing mutation. MANAGEMENT: •Focus on early detection, medical management surgical and/or ICD placement. •Pharmacological therapy is the first-line approach to symptomatic HCM. The best initial medications include negative inotropic agents, including (1) Beta blockers (preferred in most), (2) nondihydropyridine Calcium channel blockers (Verapamil), or (3) Disopyramide. Medical management: •Beta blockers first-line medical management for symptomatic patients. Beta blockers are negative inotropes that increase LV outflow and improve anginal symptoms by improving diastolic ventricular filling by decreasing heart rate, increase LV compliance, and reduce diastolic dysfunction since HCM is associated with poor relaxation. •Calcium channel blockers: Non-dihydropyridine Calcium channel blockers (eg, Verapamil) are alternatives to Beta blockers when there are contraindications to Beta blockers (eg, reactive airway disease) or if unable to tolerate Beta blockers due to adverse effects. They should not be used in HCM and LVOT obstruction with volume overload, severe dyspnea, or hypotension at rest. •Disopyramide: may be added onto BBs or CCBs (negative inotrope that increase LV outflow). Surgical: •Myomectomy septal reduction therapy usually performed in young patients with persistent symptoms despite maximal dual medical therapy, patients with symptoms + severe LVOT obstruction (50 mmHg or greater at rest or with provocation), persistent NYHA III/IV class Heart failure despite optimal medical therapy, or syncope related to LVOT obstruction. •Alcohol septal ablation: a minimally invasive procedure that is an alternative to surgical myomectomy or if myomectomy is contraindicated. Alcohol septal ablation reduces LVOT obstruction, improves symptoms, increases exercise capacity, and may improve long-term survival. Alcohol is injected via the septal artery to obliterate obstructing muscle tissue. •Implantable cardioverter defibrillator recommended for secondary prevention of sudden cardiac death when there is any personal history of ventricular fibrillation or sustained ventricular tachycardia, in patients with a history of syncope or events seen on Holter monitor. CAUTIONS: •Patients should avoid dehydration, extreme exertion, and strenuous exercise, especially athletic activities. Low-level physical activity is not a risk factor for sudden death. •Cautious use of Digoxin because Digoxin increases contractility. •Medications that decrease left ventricular preload or systemic vascular resistance should be avoided, as they can decrease the diastolic volume in the LV and increase left ventricular tract outhow obstruction vasodilators (eg, Dihydropyridine Calcium channel blockers), diuretics, ACE inhibitors, Nitrates, or Angiotensin receptor blockers.

Atrial flutter

•Characterized by rapid, regular atrial depolarizations at a characteristic rate around 300 beats/min (240 - 400) due to 1 single irritable atrial focus firing at a fast rate with some degree of AV node conduction block. •Similar to Atrial fibrillation, there is an increased risk of atrial thrombus formation that can lead to cerebral &/or systemic embolization (eg, stroke). EPIDEMIOLOGY: •Atrial flutter & Atrial fibrillation are more common in men than in women, increasing with age. PATHOPHYSIOLOGY: •Typical (classic) Atrial flutter involves a single reentrant circuit in the right atrium around the tricuspid valve annulus. The electrical wavefront often propagates in a counterclockwise direction. ETIOLOGIES: •~60% of patients with Atrial flutter have Coronary artery disease or hypertensive heart disease; 30% have no underlying cardiac disease. •Rheumatic heart disease, Congenital heart disease, Pericarditis, Cardiomyopathy, open heart surgery. •Alcohol consumption, Pulmonary embolism, hypoxia, Hyperthyroidism, Pheochromocytoma, electrolyte imbalance, obesity, Digitalis toxicity. CLINICAL MANIFESTATIONS •Symptomatic: palpitations, dizziness, fatigue, poor exercise tolerance, mild dyspnea, and presyncope due to decreased cardiac output as a result of rapid ventricular rate. •Unstable; symptoms are due to hypoperfusion and can include hypotension (eg, systolic BP in double digits), altered mental status, or refractory chest pain. DIAGNOSIS •ECG: flutter ("sawtooth") atrial waves usually ~300 beats per minute (atrial rate usually 240-400 beats/min) but no discernable P waves. Identical flutter waves (1 ectopic atrial focus). •Transthoracic echocardiography: preferred initial imaging modality for evaluating Atrial flutter. Echocardiogram can evaluate right and left atrial size, the size and function of the right and left ventricles, and assess for valvular heart disease, LVH, and pericardial disease. MANAGEMENT •Stable: Vagal maneuvers. Rate control with Beta blockers (eg, Metoprolol, Atenolol, or Esmolol) OR non-dihydropyridine Calcium channel blockers (eg, Diltiazem, Verapamil). Digoxin is another option for rate control but is usually reserved for patients in whom Beta blockers or CCBs are contraindicated [g, severe heart failure (New York Heart Association class III or IV), hypotension)] due to its adverse effects and toxicity. •Unstable; direct current (synchronized) cardioversion. •Anticoagulation: similar criteria (eg, CHADS2-VASc) for nonvalvular Atrial fibrillation in patients at risk for embolization to decrease thromboembolic complications. •Reversion to normal sinus rhythm: -Radiofrequency catheter ablation (definitive management) & superior to antiarrhythmics. -Direct current cardioversion (electrical cardioversion). -Class IA, IC, or III antiarrhythmics (eg, Ibutilide).

Ventricular tachycardia

•Defined as 3 or more consecutive PVCs (wide complex QRS duration >120 ms) at a rate >100 beats per minute (usually between 120-300 bpm). •If sustained ventricular tachycardia is not managed, it can result in Ventricular fibrillation. CLASSIFICATION: •Sustained VT = duration at least 30 seconds or causes hemodynamic collapse in < 30 seconds. Regular wide QRS complex (120 ms or greater) at a rate > 100 bpm with uniform consecutive beats. Monomorphic (same QRS morphology); Polymorphic (different morphologies). ETIOLOGIES: •Underlying heart disease: ischemic heart disease most common cause (eg, post MI) 70%, structural heart defects, cardiomyopathies. •Prolonged QT interval. Electrolyte abnormalities (eg, Hypomagnesemia, hypokalemia, hypocalcemia), Digoxin toxicity. Medications: Digoxin toxicity. CLINICAL MANIFESTATIONS •Symptomatic: palpitations, dizziness, fatigue, dyspnea, & chest pain. •Unstable: hypoperfusion can cause hypotension (eg, systolic BP in double digits), altered mental status, refractory chest pain, or acute pulmonary edema. ECG: Regular, wide complex tachycardia with no discernable P waves. MANAGEMENT OF ACUTE TACHYARRHYTHMIAS Stable VT: •Antiarrhythmics: Class 1 or Ill - eg, Amiodarone, Procainamide. Lidocaine may be helpful in the setting of acute myocardial ischemia or infarction. Sotalol can be used in patients without structural heart disease. IV Amiodarone. Unstable VT: •Direct current (synchronized) cardioversion initial management of choice. Initial recommended dose for wide regular: 100 J. Consider sedation. Pulseless VT: •Unsynchronized cardioversion (defibrillation) + CPR. Non-sustained VT: •Asymptomatic patients with non-sustained Ventricular tachycardia (VT) and no underlying cardiac comorbidities require no additional therapy. CHRONIC THERAPY •Beta blockers: unless contraindicated, nearly all patients who experience sustained monomorphic VT (including patients with a prior MI, patients with HF and reduced LV systolic function) should be placed on Beta-blocker therapy. Most likely due to their blockade of sympathetic input to the heart, Beta blockers reduce the risk of recurrent ventricular tachyarrhythmias and sudden cardiac death as well as improve survival. •ICD therapy: patients with ischemic heart disease that survive sudden cardiac arrest due to ventricular tachycardia, or experience hemodynamically unstable or stable VT if estimated survival > 1 year.

Abdominal Aortic Aneurysm (AAA)

•Focal aortic dilation > 1.5 normal (> 3.0 cm considered aneurysmal). Infrarenal most common site. •Pathophysiology: proteolytic degeneration of aortic wall & connective tissue inflammation. RISK FACTORS •Smoking (main modifiable risk factor), atherosclerosis, age >60 years, Caucasians, males, hyperlipidemia, connective tissue disorder (eg, Marfan, Ehlers-Danlos), Syphilis, Hypertension. •Protective factors: female sex, Diabetes mellitus, non-Caucasian race, moderate alcohol consumption. CLINICAL MANIFESTATIONS •Most patients are asymptomatic - may be found to incidentally on imaging or in patients with an abdominal bruit or a palpable abdominal mass. •Symptomatic (unruptured): abdominal, flank, or back pain. On examination, an abdominal bruit may be auscultated and a pulsatile abdominal mass may be palpated. •Symptomatic (ruptured); Classic triad of acute abdominal, flank, or back pain (may be diffuse), abdominal distention, & hemodynamic instability (eg, shock, hypotension, syncope). Flank ecchymosis or pulsatile mass. •Aortoenteric fistula: presents as acute GI bleed in patients who underwent prior aortic grafting. DIAGNOSIS •CT scan with IV contrast; best initial test in symptomatic, hemodynamically stable patients to confirm rupture, exact location, size, extent, and whether endovascular repair is feasible. •Focused bedside ultrasound; initial study of choice in hemodynamically unstable patients with suspected AAA prior to abdominal exploration if possible. • Patients with known AAA who present with classic symptoms or signs of rupture can be taken to the operating room for surgical repair without preoperative imaging. Asymptomatic with suspected AAA: •Abdominal ultrasound: initial test in asymptomatic patients & to monitor progression. MANAGEMENT •Symptomatic or ruptured; immediate surgical repair (endovascular stent graft or open repair). Endovascular repair from a femoral arterial approach is now applied for most repairs when possible, especially in older and higher-risk patients. •B-blockers reduces shearing forces, l'es expansion & rupture risk. AAA SCREENING •One-time screening via abdominal ultrasound in men 65-75 years of age who ever smoked. >5.5 cm OR > 0.5 cm expansion in 6 months = IMMEDIATE SURGICAL REPAIR (even if asymptomatic), symptomatic patients or patients with acute rupture >4.5 cm = Vascular surgeon referral. 4 -4.5 cm = Monitor by ultrasound every 6 months. 3 - 4 cm = Monitor by ultrasound every year.

postural (orthostatic) hypotension

•Hypotension within 2-5 minutes of quiet standing (or after a 5 minute period of supine rest) defined by at least 20 mmHg fall in systolic pressure and/or at least 10 mmHg fall in diastolic pressure. ETIOLOGIES •Impaired autonomic function and/or decreased intravascular volume. Common in elderly > 65 years. •Medications: includes antihypertensives (eg, Alpha blockers, Nitroglycerin, ACE inhibitors), diuretics, narcotics, antipsychotics, antidepressants, and alcohol consumption •Neurologic: include diabetic neuropathy, Parkinson disease, polyneuropathies etc. •Hypovolemia (eg, Loop diuretics, hemorrhage or vomiting). CLINICAL MANIFESTATIONS •Due to cerebral hypoperfusion - dizziness, lightheadedness, palpitations, blurred vision, darkening of visual fields, and/or syncope. WORKUP •Blood pressure measurement. Tilt table test: blood pressure reduction at a 60-degree angle. •Labs (eg, hematocrit, electrolytes, BUN, creatine, glucose) to evaluate for anemia or dehydration. MANAGEMENT •Conservative initial management of choice - increasing salt and fluid intake (volume resuscitation), gradual positional changes (eg, rising slowly from a supine or sitting position), compression stockings or abdominal binder, exercise, and discontinuation of offending medications if polypharmacy is the cause. Caffeine may be helpful. Management of any contributing disease (eg, Diabetes mellitus). In the acute phase, place patient in prior position. •Medical: Both Fludrocortisone and Midodrine are first-line agents to increase vascular tone. •Fludrocortisone acetate is the first-line medical management if persistent symptoms despite nonpharmacologic measures. Fludrocortisone is a synthetic mineralocorticoid (similar to aldosterone) that causes blood volume expansion and increased vascular tone. Adverse effects include Hypokalemia (may require potassium supplementation), edema, CHF, or worsening seated or supine hypertension, which may require dose reduction or discontinuation. •Pressor agents: Midodrine is an alpha-1 adrenergic agonist and Droxidopa is a norepinephrine precursor that increase vascular tone. May be used in combination with Fludrocortisone or first-line monotherapy in those unable or unwilling to tolerate Fludrocortisone adverse effects. •Avoiding the flat position, sleeping with the head of the bed raised 30-45 degrees.

Acute pericarditis

•Infection of the endothelium/valves 2ry to colonization (eg, during transient/persistent bacteremia). ETIOLOGIES: •Idiopathic most common. Most cases of "idiopathic" Pericarditis are presumed viral in etiology. •Viral: 2 most common causes of Pericarditis are idiopathic & viral (especially Coxsackieviruses A and B & Echovirus). Adenoviruses, parvovirus B19, HIV, Influenza, EBV, and CMV. •Dresser syndrome post MI pericarditis + fever + pleural effusion that may occur after several weeks after the Myocardial infarction. Delayed autoimmune process post MI. •Autoimmune, uremia, bacterial, radiation, medications: Procainamide, Isoniazid, Doxorubicin. CLINICAL MANIFESTATIONS •Chest pain: most common symptom (>95%) classically sudden onset of pleuritic (sharp, worse with deep inspiration or coughing) anterior chest pain that is often persistent and positional: worse when supine & improved with the seated position or by leaning forward (sitting & leaning forward reduces pressure on the parietal pericardium). The pain may radiate to the shoulder and trapezius ridge (due to phrenic nerve irritation), back, neck, arm, or epigastric. •Pericardial friction rub: often best heard at the left parasternal area and the intensity may be increased during auscultation by having the patient sitting up and leaning forward or resting the elbows on the knees, applying firm pressure on the stethoscope diaphragm during suspended respiration. Consists of a scratchy, squeaking sound with 3 components (triphasic). DIAGNOSTIC STUDIES ECG: diagnostic test of choice - Precordial leads: widespread diffuse (typically concave up) ST segment elevation in V1-V6 with associated PR depression in those leads and without reciprocal T-wave inversions or Q waves. PR segment deviation is highly specific (less sensitive). •Lead aVR: is associated with reciprocal ST depression and PR elevation. Echocardiogram: •Useful to evaluate for an associated pericardial effusion and /or signs of cardiac tamponade. MANAGEMENT •High-dose NSAIDs or Aspirin first-line for nearly all patients with acute idiopathic or viral Pericarditis × 7-14 days (symptoms usually subside in 1-2 days). •Colchicine: second-line or first-line as monotherapy or in conjunction with Aspirin or NSAIDs. •Glucocorticoids: usually reserved for severe or refractory cases, if a regimen of NSAIDs/ASA and Colchicine is contraindicated, or in if connective tissue disease, autoreactivity, uremia, or SLE. Dresser syndrome: •Aspirin and/or Colchicine first-line agents (avoid NSAIDs other than Aspirin because they can interfere with myocardial scar formation).

For each of the following cardiovascular conditions, students should describe the underlying pathophysiology, presenting signs and symptoms, basic epidemiology, modifiable (when applicable) and non-modifiable risk factors, differential diagnosis, appropriate diagnostic studies, clinical intervention, treatment guidelines, pharmaceutical therapies, and health maintenance concerns: Endocarditis

•Infection of the endothelium/valves 2ry to colonization (eg, during transient/persistent bacteremia). •Mitral valve most common valve involved (M>A>T>P). Exception is IV drug use - tricuspid valve most common in IV drug users. •Risk factors: increased age (> 60 years of age), rheumatic heart disease, IV drug use, immunosuppression, prosthetic heart valves, congenital heart disease. Acute bacterial endocarditis: •Infection of normal valves with a virulent organism (eg, S. aureus). Subacute bacterial endocarditis: •Indolent infection of abnormal valves with less virulent organism (eg, S. viridans). IV drug-related endocarditis: •Most commonly due to S. aureus (especially MRSA). Pseudomonas aeruginosa, Candida. Prosthetic valve endocarditis: •Early (within 60 days): usually caused by S. aureus (including MRSA) & S. epidermidis. MICROBIOLOGY Staphylococcus aureus: •Most common cause of IE, including ACUTE infective endocarditis, prosthetic valve E, and IVDA infective endocarditis (especially MRSA). Often affects normal valves. Streptococcus viridans •Most common cause of SUBACUTE infective endocarditis, often affecting damaged valves. •Because it is part of the oral flora, S. viridans is associated with Endocarditis from transient bacteremia secondary to gingivitis, poor dentition, or dental procedures. Staphylococcus epidermis: •Early Prosthetic valve endocarditis (especially within 60 days of the procedure), is usually caused by S. aureus or S. epidermidis (coagulase-negative staphylococci). Enterococcus: •Seen especially in men >50 years with a recent history of gastrointestinal or genitourinary procedure. HACEK organisms •(Haemophilus aphrophilus, Actinobacillus, Cardiobacterium hominis, Eikenella corrodes, Kingella kingae) are gram- negative organisms that are hard to culture. •Suspect these organisms in patients with endocarditis & negative blood cultures. Streptococcus gallolyticus (bovis): •Commensal bacteria of the gut seen with increased incidence in patients with Colorectal cancer and Ulcerative colitis (Colonoscopy should be performed in these patients to rule out both). CLINICAL MANIFESTATIONS •Constitutional: persistent fever most common symptoms, chills, anorexia, weight loss, malaise. •New onset of a murmur or worsening of an existing murmur (85%). •Osler nodes: painful or tender raised violaceous nodules on the pads of the digits and the palms (may be seen on the thenar or hypothenar eminence). •Janeway lesions: painless erythematous macules on the palms & soles. •Splinter hemorrhages (linear reddish-brown lesions under the nail bed), petechia (skin or mucous membranes). Roth spots: retinal hemorrhages with central clearing. •Splenomegaly, septic arterial or pulmonary emboli, glomerulonephritis. •Right-sided IE: in addition to fever & constitutional symptoms, especially with tricuspid involvement, pulmonary symptoms are common (eg, cough, dyspnea, pleuritic chest pain, and hemoptysis) may mimic URI. ~90% of patients with right-sided IE are people who inject drugs. DIAGNOSTIC STUDIES •The 2 most important tests for suspected Endocarditis are (1) blood cultures and (2) echocardiography as they help to establish the 2 major Duke criteria. •Blood cultures: (before antibiotic initiation). 3 sets at least 1 hour apart if the patient is stable. •ECG: at regular intervals to assess for new conduction abnormalities (prone to arrhythmias). •Echocardiogram: obtain TTE first; consider TEE if TE is nondiagnostic or increased suspicion. Transesophageal echocardiogram (TEE) much more sensitive than TTE (>90% v 50% in NVE) (82% - 36% in PVE) so may be used in patients with suspected Prosthetic valve endocarditis. •Labs: CBC: leukocytosis, anemia (normochromic, normocytic ); TESR/Rheumatoid Factor. MODIFIED DUKE CRITERIA MAIOR MINOR • SUSTAINED BACTEREMIA 2 © blood cultures by • Predisposing condition organism known to cause endocarditis. abnormal valves, IVDA, indwelling catheters, etc. • Fever (>38° C /100.40F). • ENDOCARDIAL INVOLVEMENT: documented by • Vascular & embolic phenomena: Janeway lesions, septic arterial or pulmonary emboli, ICH. either: © echocardiogram: (vegetation, abscess, • Immunologic phenomena: - Osler's nodes, Roth spots, ® Rheumatoid factor valve perforation, prosthetic dehiscence) - Acute glomerulonephritis © Blood culture not meeting major criteria. - clearly established new valvular regurgitation (aortic or mitral regurgitation) © echocardiogram not meeting major criteria (eg, worsening of existing murmur). Clinical criteria for infective endocarditis: 2 major OR 1 major + 3 minor OR 5 minor (80% accuracy) MANAGEMENT OF INFECTIVE ENDOCARDITIS: suggested Empiric therapy: NATIVE VALVE (MSSA) • Anti-staphylococcal Penicillin eg, Nafcillin, Oxacillin. • Empiric management of suspected MSSA may include Anti- staphylococcal Penicillin plus EITHER Ceftriaxone or Gentamicin. • Penicillin allergy: Cefazolin is an alternative if non- anaphylactoid allergy; Vancomycin or Daptomycin. NATIVE VALVE (MRSA) OR • Empiric treatment includes Vancomycin plus EITHER UNKNOWN Ceftriaxone or Gentamicin. • Daptomycin is an alternative to Vancomycin in patients unable to tolerate Vancomycin. PROSTHETIC VALVE • Vancomycin + Gentamicin + Rifampin. Rifampin is efficient at killing Staphylococci that are adherent to foreign material FUNGAL • Parenteral antifungal agent (eg, Amphotericin containing product) with or without combination therapy (eg, Amphotericin B and Flucytosine). Antifungal therapy is usually given for 6 weeks or more. • Patients often need surgical intervention for fungal cases Penicillin & Vancomycin have great gram-positive coverage. Gentamicin & Ceftriaxone have great gram-negative coverage. INDICATIONS FOR SURGERY • Refractory CHF; persistent or refractory infection, invasive infection, prosthetic valve, recurrent systemic emboli, fungal infections. • In acute Endocarditis, antibiotics are started promptly after culture data is obtained. • In subacute Endocarditis, if the patient is hemodynamically stable, antibiotics may be delayed in orderto properly obtain blood culture data, especially if prior treatment with antibiotics. • Adjust the antibiotic regimen based on organism, culture & sensitivities. Fever may persist up to 1 week after appropriate antibiotic therapy has been initiated. • Duration of therapy usually 4-6 weeks (with aminoglycosides used only for the first 2 weeks). ENDOCARDITIS PROPHYLAXIS INDICATIONS Cardiac conditions 1. Prosthetic (artificial) heart valves. 2. Heart repairs using prosthetic material (not including stents). 3. Prior history of endocarditis. 4. Congenital heart disease. 5. Cardiac valvulopathy in a transplanted heart. Procedures 1. Dental: involving manipulation of gums, roots of the teeth, oral mucosa perforation. 2. Respiratory: surgery on respiratory mucosa, rigid bronchoscopy. 3. Procedures involving infected skin/musculoskeletal tissues (including abscess incision & drainage). Regimens • Amoxicillin 2g 30-60 minutes before the procedures listed above. • Clindamycin 600mg if penicillin allergic (except for dental procedures). • Macrolides or Cephalexin are other options. Prophylaxis is no longer routinely recommended for gastrointestinal or genitourinary procedures. Prophylaxis no longer routinely recommended for most types of valvular heart disease (including mitral valve prolapse, bicuspid aortic valve, acquired mitral or aortic valve disease, hypertrophic cardiomyopathy). Good oral hygiene recommended to reduce temporary episodes of bacteremia. • In May 2021, the American Heart Association updated its 2007 guidelines on antibiotic prophylaxis for prevention of streptococcal Infective endocarditis among patients with relevant cardiac risk factors undergoing dental procedures. In such patients, the preferred oral regimen is Amoxicillin; alternatives for patients with Amoxicillin allergy include Cephalexin, Azithromycin, Clarithromycin, and Doxycycline. Clindamycin is no longer a suggested alternative in patients undergoing dental procedures because it is associated with more frequent and severe adverse effects (particularly Clostridioides difficile infection) than the others.

Mobitz type II second-degree AV block

•Interruption of electrical impulse at the AV node resulting in occasional non-conducted impulses. PATHOPHYSIOLOGY: •AV node dysfunction (commonly at the bundle of HIS). ETIOLOGIES •Most common in the presence of structural heart disease: eg, myocardial ischemia, myocardial fibrosis, myocarditis (eg, Lyme disease), endocarditis, infiltrative disease (eg, Amyloid, Sarcoidosis) •Hyperkalemia, Increased vagal tone, iatrogenic (eg, AV nodal blockers, post-cardiac surgery/ablation) CLINICAL MANIFESTATIONS •Asymptomatic in most cases, especially if there are infrequent non-conducted P waves. •Symptomatic: due to bradycardia-related decreased perfusion - fatigue, dizziness, dyspnea, chest pain, syncope, or in severe cases, hypotension or altered mental status. DIAGNOSIS •ECG: constant PR interval before & after the non-conducted atrial beat (dropped QRS complexes). •If ischemia is suspected based on clinical picture, cardiac biomarkers, chest radiograph, and electrolytes should be ordered. MANAGEMENT •Symptomatic: transcutaneous pacing &/or Atropine for symptomatic bradycardia with permanent pacemaker long-term management. These patients often do not respond to Atropine. •Unstable: Atropine and in most, temporary cardiac pacing to increase pulse rate & cardiac output. •Definitive: permanent pacemaker required in many patients because it often progresses to third degree AV block and is associated with complications of hypotension, and cardiac arrest.

Constrictive pericarditis

•Loss of pericardial elasticity (thickening, fibrosis & calcification) leading to restriction of ventricular diastolic filling. •Pathophysiology: fibrosis limits ventricular filling, decreasing stroke volume and cardiac output. •Etiologies: any cause of acute Pericarditis. In the US, idiopathic and viral Tuberculosis (worldwide). CLINICAL MANIFESTATIONS •Dyspnea most common symptom, fatigue, orthopnea. •Right-sided heart failure signs: increased jugular venous distention, peripheral edema, nausea, vomiting, increased hepatojugular reflex, Kussmaul's sign (the lack of an inspiratory decline or an increase in jugular vein pressure with inspiration). •Pericardial knock: high pitched diastolic sound similar to S3 (sudden cessation of ventricular filling). DIAGNOSTIC STUDIES •Chest radiograph: pericardial calcification may be seen especially on lateral view, clear lung fields. Normal or slightly increased heart size, Square root sign on cardiac catheterization. •Echocardiography: pericardial thickening and/or calcification. Also used to rule out Restrictive cardiomyopathy. "Square root" sign - early diastolic dip followed by a plateau of diastasis. •CT scan or MRI: more sensitive than echocardiography - pericardial thickening or calcification. MANAGEMENT •Diuretics for symptom relief & reduction of edema and venous pressure. Pericardiectomy (definitive).

Stress testing in CAD

•Most useful noninvasive test in the diagnosis of Coronary artery disease. Stress ECG •Indications: most commonly used stress test. Useful only if baseline ECG is normal. •Positive findings include ECG changes (eg, ST depressions, T wave inversions, poor R wave progression) or reproduction of symptoms or signs. •Limitations: does not locate the area of ischemia. Myocardial perfusion imaging: •Uses Thallium or Technetium for imaging. •Indications: can be used if baseline ECG is abnormal. Gives information regarding the location & extent of ischemia. •Can be performed either with exercise or a pharmacologic agent if the patient cannot exercise vasodilators (eg, Adenosine or Dipyridamole). •Contraindications to vasodilators: bronchospastic disease, hypotension, AV blocks. •Theophylline and caffeine should be stopped 48 hours and 12 hours respectively. Stress echocardiogram: •Indications: can be used if baseline ECG is abnormal. Gives information regarding the location & extent of ischemia. •Can be performed with exercise or pharmacologic if patient cannot exercise with positive inotropes (eg, Dopamine or Dobutamine). •Contraindications to positive inotropes: severe LV outflow obstruction (eg, Aortic stenosis), ventricular arrhythmias, recent MI (1-3 days), or severe systemic hypertension. Other considerations: •During stress testing in patients without a history of coronary artery disease, antianginal medications (nitrates, beta blockers and calcium channel blockers) should be withheld 48 hours prior to stress testing. •Patients with known history of coronary artery disease should continue their antianginal medications prior to stress testing. This allows for the evaluation of the efficacy of the patient's current treatment regimen and also to determine the appropriate level of exercise that is safe for the patient.

Premature ventricular complexes (PVC)

•PVC: premature beat originating from the ventricle -> wide, bizarre QRS occurring earlier than expected. With a PVC, the T wave is in the opposite direction of the QRS usually. Associated with a compensatory pause = overall rhythm is unchanged (AV node prevents retrograde conduction). MANAGEMENT: Low-risk for heart disease: •No treatment is needed for PVCs in patients with low-risk of heart disease. PVCs are typically self-limiting, rarely life-threatening, and in most cases do not require treatment. •Low-risk patients include overall good health with no personal or family history of heart disease (structural or electrical), minimal to no symptoms due to PVCs, no symptoms of heart disease, normal physical examination, and normal-appearing ECG (other than PVCs). Asymptomatic PVCs: •Low PVC burden (<10%) and without apparent underlying structural heart disease, and no symptoms, observation and reassurance alone are the initial management of choice. •In asymptomatic patients with structural heart disease, management focuses on providing appropriate therapy tailored to the specific underlying condition. Often times, treatment for the underlying condition (eg, Beta blockers) may reduce the occurrence of PVCs. Symptomatic patients: •Management of patients with significant symptoms include reduction, correction, or elimination of underlying triggers and initiation of medical therapy - Beta-blockers are often first-line medical therapy. Non-dihydropyridine Calcium channel blocker may be used. •Radiofrequency catheter ablation may be indicated in cases refractory to medical therapy (or if adverse effects due to medical therapy develops) or for those who do not prefer to be on long term antiarrhythmic therapy. Successful treatment may reverse PVC-induced cardiomyopathy •Class I antiarrhythmics: eg, Flecainide or Amiodarone are suitable alternatives for patients without apparent structural heart disease in whom catheter ablation is not performed,

First degree AV block

•Prolonged AV conduction characterized by a longer than normal delay in conduction from the atrium to the ventricle (prolonged PR interval > 0.20 seconds at resting heart rates) without interruption in atrial to ventricular conduction (delayed but 1:1 conducted impulses). PATHOPHYSIOLOGY •The AV node normally delays the conduction from the atria to the ventricles briefly to allow for full ventricular filling before contraction, represented by a normal PR interval of 0.12-0.20 seconds. •In First degree AVB, the conduction delay is most frequently in the AV node but may also be in the His-Purkinje system. Morphology and size of the QRS complex will reflect the site of conduction delay. •It is not a true "block" because all of the atrial impulses are conducted to the ventricles; it is more appropriately characterized as a prolonged delay in AV conduction. ETIOLOGIES: •The two most common etiologies are (1) increased vagal tone (eg, highly conditioned endurance athletes or patients with lower resting heart rates) or (2) result of AV node-blocking medications (eg, Beta blockers, Non-dihydropyridine calcium channel blockers, Digoxin). •Often a normal variant - individuals with increased vagal tone without structural heart disease (eg, well-conditioned athletes) & patients with a slow resting heart rate may show evidence of a first degree AVB as a result of increased vagal tone & baseline lower resting heart rate. •AV nodal blocking drugs eg, Digoxin, Beta blockers, Non-dihydropyridine Calcium channel blockers. Some antiarrhythmic agents. Fibrosis in the conduction system in older patients, intrinsic AV node disease, increase in vagal tone, Acute MI, electrolyte disturbances (eg, hyperkalemia), Infiltrative diseases: eg, Amyloidosis, Sarcoidosis, Systemic sclerosis. Myocarditis (eg, Lyme disease). CLINICAL MANIFESTATIONS: •Asymptomatic in most cases and is most commonly benign in nature. Very rarely do patients present with symptoms directly related to First degree AV block. •It is rarely symptomatic, but if symptomatic, bradycardia-related decreased perfusion produces symptoms - fatigue, dizziness, dyspnea, chest pain, syncope. DIAGNOSIS •ECG: all atrial impulses are delayed but conducted to the ventricles = prolonged PR interval (> 0.20 seconds) at resting heart rate + all P waves are followed by QRS complexes (1:1 conduction). MANAGEMENT: Asymptomatic: •Asymptomatic patients with first degree AV block do not require any specific therapy beyond potential observation, surveillance, or cardiac consult if there is concern for underlying pathology. •Any reversible causes of AV block (eg, prior use of AV nodal blocking agents, ischemia) should be identified and treated (ischemia) or withdrawn (causative medications). Symptomatic: •Atropine first-line. Atropine's vagolytic properties can improve AV node conduction Definitive: Permanent pacemaker: •Most cases of First degree AV block do not require pacemaker placement but may be needed if severe.

Sinus bradycardia

•Regular cardiac rhythm originating from the sinus node with a decreased heart rate <60 beats per minute (bpm) in adults (normal resting heart rate for adults 60-100 bpm). PATHOPHYSIOLOGY: •Sinus Bradycardia is caused by different intrinsic and extrinsic factors which may increase vagal tone or compromise the integrity of the sinus node. ETIOLOGIES: •Physiologic; Healthy young adults and athletes tend to have an increased vagal tone which keeps them in sinus bradycardia at rest, patients > 65y often have Sinus bradycardia secondary to the aging of the sino-atrial node; Vasovagal reaction, vasovagal stimulation (eg, endotracheal suctioning), carotid sinus sensitivity; Increased intracranial pressure; Sleep; Nausea and/or vomiting, Carotid massage. •Pathologic: Medications: Beta blockers, Calcium channel blockers, Digoxin, Adenosine, Narcotics. Ischemic heart disease: sinoatrial node ischemia, inferior or posterior wall Myocardial infarction. Infection: Gram-negative sepsis, Lyme disease. Sick sinus syndrome, Hypothyroidism, hypoxia. CLINICAL MANIFESTATIONS •The majority of patients with Sinus bradycardia do not have symptoms. •Symptoms: fatigue, exercise intolerance, lightheadedness, dizziness, syncope or presyncope, altered mental status changes, anginal symptoms. DIAGNOSIS: ECG: Regular, slow rhythm (<60 bpm). Normal-appearing P wave; every P followed by a QRS complex. Workup: •History and physical examination is the most significant component of evaluation for a patient who presents with signs and symptoms of sinus Bradycardia - vital signs (respiratory rate, blood pressure, temperature, and heart rate) and an electrocardiogram. Hemodynamically unstable patients may develop hypotension, altered mental status, dyspnea, or worsening of anginal symptoms. •Laboratory workup: may include: basic metabolic panel (including calcium and magnesium), glucose levels, thyroid function testing, Troponin, and toxicological drug screen. MANAGEMENT: Symptomatic or unstable: •Atropine first-line treatment. Atropine is an anticholinergic drug that decreases vagal tone (increased vagal tone due to a variety of reasons is the most common cause of Sinus bradycardia). Intravenous (IV) atropine 0.5 mg push every 3-5 minutes up to 3 mg total. If the patient's symptoms and heart rate fail improve, the patient is a candidate for a temporary pacemaker. •Second-line Epinephrine or transcutaneous pacing if not responsive to Atropine. Epinephrine directly stimulates the adrenergic beta-1 receptor, leading to increased heart rate and contractility. •Permanent pacemaker: definitive management. •Hypothermic patients warmed to normothermia before making definitive determinations on treatment. Asymptomatic: •No medical treatment or specific cardiac medications needed if physiologic - observation, cardiac consult, further workup, or cardiology follow up may be needed if pathologic. •In older individuals, Sinus bradycardia may be due to sinus node abnormalities or underlying ischemic heart disease so further workup is often warranted.

Sinus tachycardia

•Regular cardiac rhythm originating from the sinus node with an increased heart rate > 100 bpm in adults (normal resting heart rate for adults is 60-100 bpm). •Can be a normal variant but the presence of tachycardia at rest could be the earliest sign of serious pathology. ETIOLOGIES: •Physiologic; Catecholaminergic triggers - eg, exercise, stress, pain, and anxiety; Young children/infants; Postural orthostatic tachycardia syndrome is also often seen in young females and is a common orthostatic disease seen after stress (eg, sepsis, pregnancy, surgery, or trauma). •Pathologic: Infectious: fever, infection, sepsis; Hypovolemia: dehydration, shock, hemorrhage; Cardiac etiologies: Myocarditis, Cardiac tamponade, Acute coronary syndrome; Respiratory: Hypoxia, Pneumonia, Pulmonary emboli, Pneumothorax; Metabolic; hypoglycemia, hyperthyroidism. electrolyte disorders (eg, calcium, potassium, magnesium disorders), anemia, pregnancy. Clonidine, alcohol, benzodiazepine, or opioid withdrawal; Sympathomimetics eg, decongestants, cocaine. CLINICAL MANIFESTATIONS •Most patients with Sinus tachycardia are commonly asymptomatic from the rhythm itself. •Depending on the underlying etiology, Sinus tachycardia may present with dyspnea, chest pain, lightheadedness, dizziness, syncope, and presyncope. PHYSICAL EXAMINATION: •Evaluation of the patient's hemodynamic status is crucial to stabilization of the patient, particularly to ensure that the patient is not on the verge of cardiovascular collapse due to shock. DIAGNOSIS: ECG: •Regular, rapid rhythm (> 100 bpm). Normal-appearing P wave; every P followed by a QRS complex. Workup: •Depending on the suspected etiology, workup may include electrocardiogram, echocardiogram, 24-hour Holter recording (can confirm the presence of inappropriate sinus tachycardia), arterial blood gas, lactic acid level, chest radiograph, D-dimer, chest CT angiography, ventilation-perfusion scan, cardiac enzymes levels, glucose level, electrolytes, complete blood count, and/or a toxicology screen. MANAGEMENT: •Identifying and treating the underlying cause of Sinus tachycardia is the cornerstone of management. •Benign causes (eg, physical activity or stress) often do not require any specific cardiac treatment •If Sinus tachycardia is due to a medical condition or the patient is at risk for worsening of clinical status (eg, sepsis, shock, hypoxia, metabolic acidosis, Acute coronary syndrome), admission and urgent evaluation is recommended with treatment of the underlying cause. •Beta blockers (eg, Metoprolol) used in the management of persistent Sinus tachycardia in the setting of Acute coronary artery syndrome.

Brugada syndrome EKG

•Right bundle branch block (RBB) pattern (often incomplete). •ST elevation V1-V3 (often downsloping pattern). •T wave inversions V1 & V2, +/-S wave in lateral leads.

Proximal Supraventricular Tachycardia (PSVT)

•SVT is an umbrella term when a more specific term can't be applied to a tachyarrhythmia originating above the ventricles (atrial or AV nodal source); usually regular ventricular response. PATHOGENESIS: •Reentry is the most common cause of narrow ORS complex tachycardia - AV nodal reentry (most common type) and AV reciprocating tachycardia. In reentry, there exists 2 distinct electrical conduction tissues or pathways with different electrophysiologic attributes that are connected both proximally and distally, creating a functional or anatomical circuit. PATHOPHYSIOLOGY: •AV node re-entrant tachycardia: (AVNRT) two pathways within the AV node or perinodal tissue (1 normal & 1 accessory pathway). Most common type. •AV reciprocating tachycardia: (AVRT) two pathways with one being an extranodal accessory pathway (1 normal and 1 accessory pathway outside of the AV node). •A narrow QRS complex (<120 milliseconds) indicates rapid activation of the ventricles via the normal His-Purkinje system (origination above or within the His bundle). Orthodromic. •A widened QRS (120 milliseconds or greater) occurs when ventricular activation is abnormally slow due to (1) origination below the His bundle in the bundle branches, Purkinje fibers, or ventricular myocardium (eg, ventricular tachycardia); (2) aberrant conduction of the supraventricular tachycardia, or (3) conduction occurs over an accessory pathway (eg, antidromic conduction). CLINICAL MANIFESTATIONS •Symptomatic: palpitations (most common), anxiety, exercise intolerance, dizziness, lightheadedness, diaphoresis, fatigue, dyspnea, presyncope, syncope (rare), chest pain or discomfort. •Unstable: symptoms are due to hypoperfusion and can include hypotension (eg, systolic BP in double digits), altered mental status, refractory chest pain, signs of Heart failure (eg, pulmonary edema). ECG •Orthodromic (95%): regular, narrow-complex tachycardia (> 100 bpm + QRS < 120 milliseconds) + no discernable P waves due to the rapid rate. •Antidromic (5%): regular, wide-complex tachycardia (> 100 bpm + QRS 120 milliseconds or greater) that mimics Ventricular tachycardia. •Heart rate > 100 bpm. •Rhythm usually regular with narrow QRS complexes. •P waves hard to discern due to the rapid rate. MANAGEMENT Stable (regular, narrow complex): •If no severe symptoms or hemodynamic collapse or asymptomatic patients, the sequential approach is Vagal maneuvers, followed by IV Adenosine (if Vagal maneuvers are ineffective). •Second-line: IV Non-dihydropyridine Calcium channel blockers (eg, Verapamil, Diltiazem) or IV Beta blockers (eg, Metoprolol, Atenolol, Esmolol); Digoxin. All are AV nodal-blocking agents. Stable (wide complex): •Antiarrhythmics: Class I or Class III - eg, IV Procainamide or IV Amiodarone. •IV Procainamide if WPW or antidromic rhythm is suspected. Unstable: •Direct current (synchronized) cardioversion should be performed urgently in most. Definitive: •Radiofrequency catheter ablation is an option if an accessory pathway is identifiable in patients with recurrent SVT.

Vasospastic (Prinzmetal's) angina

•Spontaneous episodes of angina accompanied by transient ECG ischemic ST changes due to epicardial coronary artery vasospasm, leading to transient myocardial ischemia. PATHOPHYSIOLOGY •Coronary artery vasospasm: diffuse or segmental spasm of the smooth muscle layer of epicardial coronary arterial wall may result in transient ST segment elevation & transmural ischemia. •Endothelial dysfunction and vascular smooth muscle hyper-reactivity. TRIGGERS: •Cold weather, Alpha-agonists (eg, Pseudoephedrine, Oxymetazoline, cocaine, and Amphetamines). Cocaine use, especially with concurrent cigarette smoking history; Sumatriptan. •Hyperventilation; Coronary manipulation through cardiac catheterization, Marijuana, Alcohol. Although exercise is a potential trigger, exercise doesn't usually provoke episodes. RISK FACTORS: •Females, smoking, other vasospastic disorders (eg, Raynaud phenomenon, Migraine). Compared to Stable angina, patients with Vasospastic angina are often younger (<50 years) and exhibit fewer classic cardiovascular risk factors (except for cigarette smoking). Magnesium deficiency. CLINICAL MANIFESTATIONS: •Chest pain; mainly at rest (especially midnight to early morning), usually not triggered by exertion nor relieved with rest as is typical Angina. Often, the patient is younger with few or no classical cardiovascular risk factors. Episodes last between 5-15 minutes but may be persistent. DIAGNOSIS •ECG: transient ST elevations in the pattern of the affected artery that resolve with symptom resolution (ST elevations may resolve with Calcium channel blockers or Nitroglycerin). May have ST depressions. The ECG is usually normal between anginal episodes in Vasospastic angina. •Angiography: rules out coronary artery disease and may show evidence of coronary vasospasm during angiography (>90% constriction), especially with the use of Ergonovine, hyperventilation, or Acetylcholine. Usually no evidence of high-grade coronary stenosis. MANAGEMENT: •Initial therapy includes cessation of smoking and pharmacologic therapy with Calcium channel blockers and sublingual Nitroglycerin as needed for symptoms of angina. •Calcium channel blockers mainstay of therapy (eg, Diltiazem, Amlodipine, Nifedipine) to cause vasodilation effect in the coronary vasculature and prevent vasoconstriction, alleviating symptoms. The use of a long-acting calcium antagonist is recommended to be given at night as the episodes of vasospasm are more frequent at midnight and early in the morning. •Short-acting nitrates: sublingual Nitroglycerin with the onset of each episode, can be used to decrease both the duration of symptoms and ischemia. •Long-acting Nitrates: second-line due to the occurrence of tolerance with chronic use. In patients on CCBs without an adequate response to treatment, long-acting Nitrates can be added. Nitrates produce direct endothelium vasodilatation, prevent vasospasm, & reduce preload. •Nicorandil, a nitrate and K-channel activator also suppress vasospastic attacks. •During an acute chest pain episode prior to the diagnosis, Aspirin and Heparin may be given until atherosclerotic disease is ruled out. Medications to avoid: •Beta blockers are avoided as they may lead to unopposed alpha-mediated vasoconstriction and vasospasm, especially non-selective Beta blockers.

Superficial thrombophlebitis

•Superficial phlebitis - The presence of pain and inflammation involving a superficial vein in the absence of thrombus. Great saphenous vein (60-80%) small/short saphenous vein (10-20%). •Superficial thrombophlebitis - Pain and inflammation involving a superficial vein + a thrombus apparent as a thickened cord or more accurately identified on imaging studies (eg, Ultrasound). RISK FACTORS; •Varicose veins - phlebitis and thrombosis of the lower extremity superficial veins most commonly occur in varicose veins, especially with a history of lack of physical activity or trauma. •Malignancy and hypercoagulable states malignancy, hypercoagulable states, and Thromboangiitis obliterans (eg, Burger disease). Trousseau sign; migratory thrombophlebitis associated with underlying malignancy (eg, Carcinoma of the pancreas most common). May be seen with other vasculitic disorders. Pregnancy and estrogen therapy. Vein excision/ablation. •Intravenous catheter use - phlebitis & thrombosis can occur due to a combination of endothelial injury and venous stasis. The upper extremity veins are affected much more commonly. CLINICAL MANIFESTATIONS: •Local phlebitis: tenderness, pain, induration, &/or erythema along the course of a superficial vein. Edema or pruritus may occur. •Recurrent or migratory thrombophlebitis - Phlebitis and thrombosis can be recurrent in the same vein. Migratory thrombophlebitis occurs in distinctly different vein segments over time. •Suppurative thrombophlebitis - High fever, fluctuance, erythema beyond the vein margins, and/or purulent drainage suggest infection within the vein (eg, suppurative or septic thrombophlebitis). May occur in the setting of recent venipuncture or catheterization. PHYSICAL EXAMINATION; •A palpable, sometimes nodular cord due to thrombus within the affected vein may occur. •Tenderness, pain, induration, edema, &/or erythema along the course of a superficial vein. DIAGNOSIS •Venous Duplex ultrasound: Indications: performed especially with lower extremity involvement to rule out the presence of coexistent DVT and to determine the exact location & extent of thrombosis (eg, thrombus within the axial veins) due to the high rate of concurrent deep vein thrombosis (DVT). Duplex findings -- include vein wall thickening and perivenous or subcutaneous edema. Noncompressibility of the vein if thrombosis is present. Perivenous fluid collections or air in the tissues indicated infection (Suppurative thrombophlebitis). MANAGEMENT: •Low-risk for VTE: eg, Focal SVT with axial vein involvement 5 cm or less in length, remote from the saphenopopliteal or saphenofemoral junction (eg, SVT involving the below knee), no medical risk factors for VTE: Supportive mainstay - NSAIDs (for pain and inflammation), extremity elevation (eg, waist level), warm or cool compresses, compression stocking therapy as tolerated if not contraindicated (eg, Peripheral arterial disease). •Intermediate risk: eg, SVT in proximity (3 - 5 cm) to the deep venous system, VT in the above-knee great saphenous vein, SVT 5 cm or greater, rather than focal involvement, SVT that propagates with conservative management, and the patient has no medical risk factors for VTE. Prophylactic anticoagulation with uncomplicated axial vein thrombosis (eg, SVT) at intermediate risk for thromboembolism. •High-risk for thromboembolism. eg, thrombus 5 cm or less to the deep venous system, especially involving the great saphenous vein above the knee within 10 cm of the saphenofemoral junction, affected vein segment 5 cm or greater, positive medical risk factors for VTE: Supportive + therapeutic anticoagulation (eg, subcutaneous Fondaparinux, Enoxaparin, oral Rivaroxaban).

For each of the following cardiovascular conditions, students should describe the underlying pathophysiology, presenting signs and symptoms, basic epidemiology, modifiable (when applicable) and non-modifiable risk factors, differential diagnosis, appropriate diagnostic studies, clinical intervention, treatment guidelines, pharmaceutical therapies, and health maintenance concerns: Aortic Dissection

•Tear through the innermost layer of the aorta (intima) due to cystic medial necrosis. •Ascending most common near the aortic arch or left subclavian (65%), 20% descending, 10% aortic arch. Ascending = high mortality. RISK FACTORS •Hypertension (most important), age > 50 years (20-30 years of age in patients with Marfan syndrome), men, vasculitis (rare), trauma, family history of aortic dissection, Turner's syndrome, Collagen disorders: (eg, Marfan syndrome, Ehlers-Danlos), pregnancy. PATHOPHYSIOLOGY: •Intimal tear; constant exposure to high pulsatile pressure and shear stress lead to weakening of the aortic wall in susceptible patients, resulting in an intimal tear (the primary event in Aortic dissection). Degeneration of the aortic media (cystic medial necrosis) is a predisposing factor. •Site of intimal tear: 95% occur in two places. The most common site of intimal tear is the first few centimeters of the ascending aorta (90% occur within 10 cm of the aortic valve). Second most common is just beyond the ligamentum arteriosum in the descending aorta. CLINICAL MANIFESTATIONS •Chest or back pain: most common symptom (90%) - sudden onset of severe (maximal severity at onset) chest and/or upper back pain with a sharp, ripping, knife-like, or tearing quality that may radiate between the scapulae. The pain is more often located in the anterior chest in ascending (type A) dissection, back or abdomen in descending (type B) dissection, or may migrate with propagation. •Accompanied symptoms: the chest pain may be accompanied with abdominal pain, neurological findings (eg, limb weakness or paresthesias due to spine ischemia), syncope, Acute coronary syndrome, Heart failure, or Cerebrovascular accident. PHYSICAL EXAMINATION: •Blood pressure: May be hypertensive (common) or hypotensive (ominous sign, indicating rupture). Hypertension more common in descending (type B) dissections (70%) but seen in only 25-35% of type A. Hypotension in 25% involving the ascending aorta but <5% in type B. •Pulse variation: variation or discrepancy in blood pressure - asymmetric blood pressure (eg, >20 mmHg blood pressure difference between the right & left arm). Asymmetric, decreased, or absent pulses (eg, carotid, brachial, radial, or femoral). May have wide pulse pressure. •Aortic regurgitation of new onset if ascending (blowing diastolic decrescendo murmur). •Neurological deficits present in 20% of cases - focal neurologic deficits include stroke, altered consciousness, acute paraplegia (spinal cord ischemia), Horner syndrome (compression of the superior cervical sympathetic ganglion), and hoarseness (vocal cord paralysis due to compression of the left recurrent laryngeal nerve). •Muffled heart sounds if cardiac tamponade is present. Ascending vs, descending: •Type A: Ascending aorta involvement should be suspected if anterior chest pain more so than the back or abdomen, syncope, hypotension, shock, Acute Aortic valve regurgitation, Acute coronary syndrome, Cardiac tamponade, Hemothorax, focal neurologic deficits related to cerebrovascular ischemia, and upper extremity pulse deficit. Often will have elements of descending dissections. •Type B: Descending aorta should be suspected if posterior chest/upper back pain that may radiate to the abdomen; Other clinical features include malperfusion syndromes (eg, abdominal pain from visceral ischemia, lower extremity ischemia, focal neurologic deficits related to spinal ischemia, or acute kidney injury). INITIAL EVALUATION: •ECG: part of the initial evaluation of chest pain to rule out MI. Ischemic changes in 15% & nonspecific ST and T changes in 30%. Extension of type A dissection can cause coronary ischemia. •Chest radiograph: commonly obtained to help rapidly differentiate the various causes of chest pain. Widening of the mediastinum or widened aortic silhouette (loss of normal aortic knob contour) are common. May be normal in 10% so normal CXR does not rule out dissection. Advanced vascular imaging: •CT angiogram, Transesophageal echocardiogram, & MR angiogram are all acceptable first-line imaging modalities to confirm and visualize the tear in suspected Aortic dissection and determine the appropriate management. •CT angiography: in most EDs, CT angiography with contrast is the most commonly used first vascular imaging technique if the patient is stable (especially if not strong suspicion of ascending dissection) to determine the type and extent of dissection. •Transesophageal echocardiogram performed at the bedside is the recommended initial advanced imaging if unstable, renal insufficiency, contrast allergy, or strong suspicion of an ascending dissection. Yields diagnosis in minutes, can be performed at the bedside or in the operating room, and is better compared to CT at seeing a tear, flap, or regurgitation. •Aortography: may be performed if the diagnosis is uncertain after advanced imaging. CT angiography has largely replaced Aortography. MANAGEMENT: Surgical management: •Indications; used in acute proximal (Stanford A/ DeBakey I and II) OR acute distal with complications (Stanford B/Debakey Type III with ischemia, progression, impending rupture, etc). Open repair may include excision of the intimal tear, obliteration of entry into the false lumen proximally, reconstitution of the aorta with interposition of a synthetic vascular graft, and repair or replacement of the aortic valve as needed. Endovascular repair an option. •Preoperative blood pressure control - blood pressure and heart rate may rise during induction for emergency surgery, so lower readings prior to surgery are recommended. Medical management: •Indications: acute descending dissection (Stanford B/Debakey Ill) without complications. •IV Beta blockers (eg, Esmolol, Labetalol) - Beta blockers help lower the systolic blood pressure, heart rate, prevent reflex tachycardia, and decrease left ventricular contractility (all of which reduce aortic wall stress). The vasodilators Sodium nitroprusside or Nicardipine can be added after Beta blocker to reach target blood pressure goal if additional therapy is needed (they are not used alone or prior to Beta Blockers because they can cause reflex tachycardia). •Blood pressure goal: systolic blood pressure is rapidly lowered to a goal of SBP 100-120 mmHg within 20 minutes to reduce shearing forces, propagation, and rupture. Maintain heart rate <60 bpm and systolic blood pressure between 100 and 120 mmHg.

sinus node dysfunction (sick sinus syndrome)

•The inability of the SA node to generate a heart rate that meets the physiologic needs. ETIOLOGIES: •Intrinsic causes include Degenerative fibrosis - The most common cause of sinus node dysfunction is the replacement of sinus node tissue by fibrous tissue (age-related), which may be accompanied by degeneration and fibrosis of other components of the conduction system. •Extrinsic: Medications - Non-dihydropyridine Calcium channel blockers, Beta blockers, Digoxin, Antiarrhythmics, Acetylcholinesterase inhibitors (eg, Rivastigmine). Parasympathomimetics, sympatholytics (eg, Clonidine, Methyldopa), Ivabradine. Metabolic - hypothyroidism, hyperkalemia, hypokalemia, hypocalcemia, hypoxia, and hypothermia can lead to depression of the pacing or autonomic instability. Inflammatory disease, trauma. Abnormally increased vagal tone - carotid sinus hypersensitivity, autonomic dysfunction. Infiltrative disease (Amyloidosis, sarcoidosis, scleroderma). CLINICAL MANIFESTATIONS: •Hypoperfusion: When symptoms do develop, they are usually attributed to hypoperfusion to vital organs with high oxygen demand. Cerebral hypoperfusion (syncope or near-fainting). •Nonspecific symptoms: fatigue, exercise intolerance, lightheadedness, palpitations, presyncope, syncope, dyspnea on exertion, or chest discomfort. WORKUP: •Prior to any testing beyond an ECG, the first step in evaluating Sinus node dysfunction is to exclude reversible causes - eg, electrolyte or metabolic abnormalities, uncontrolled sleep apnea, medications (eg, Beta-blockers, Calcium channel blockers, Digoxin), MI, systemic illness (eg, hypothyroidism). •Because of the transient episodic nature of SND, diagnosis on 12 lead ECG is uncommon and prolonged monitoring (eg, 24-48 h Holter) to detect and document bradycardic episodes is often indicated. Electrocardiogram: •Alternating bradycardia (eg, sinus pauses, sinus arrest) and atrial tachyarrhythmias in >50% of cases. Atrial fibrillation is most common, Atrial flutter, Paroxysmal supraventricular tachycardia. MANAGEMENT: •The initial step is identifying and correcting reversible factors, removing extrinsic factors, symptom control, when possible, and permanent pacemaker placement (definitive management) for patients without an identifiable reversible etiology. Unstable patients: •Pharmacologic management includes Atropine (first-line), Dopamine, or Epinephrine, as well as temporary cardiac pacing (either with transcutaneous or, if immediately available, transvenous pacing) to increase heart rate and cardiac output. •Signs and symptoms of hemodynamic instability include hypotension, altered mental status, signs of shock, ongoing ischemic chest pain, and evidence of acute pulmonary edema. Stable patients: •Monitoring, evaluation, and treatment of reversible causes of SA nodal depression, such as drugs (eg, Beta blockers, Calcium channel blockers, Digoxin), ischemia, and autonomic imbalance. Asymptomatic patients: •Persons with bradycardia and no symptoms due to the bradycardia are treated with intermittent examinations. Symptomatic patients: •Pacemaker placement is indicated if documented correlation between symptoms and sinus bradycardia or sinus pauses to relieve symptoms.

ACS conservative management

•Used in patients whose chest pain began >12 hours (without current/active chest pain) or low TIMI: -Aspirin (+/- Clopidogrel x 9 months), statin, beta blocker, ACE Inhibitor. Nitroglycerin as needed. •For patients with symptoms of >12 hours, fibrinolytic therapy is not indicated, but emergent PCI may be considered, particularly for patients with evidence of ongoing ischemia or high-risk.


Ensembles d'études connexes

Chapter 13 Protestant Reformation

View Set

Zybooks Chapter 16 Searching and Sorting Algorithms Participation Activity Questions

View Set